Разряды цифр в математике: Разряды и классы чисел | Математика

Разряды чисел в математике — intmag24.ru

В математике разряды чисел — это позиции цифр в числе (место, где располагается цифра). Их нужно читать справа налево.

Рассмотрим число 345, которое состоит из трех цифр:
➤ первая позиция — цифра 5 — разряд единиц
➤ вторая позиция — цифра 4 —разряд десятков
➤ третья позиция — цифра 3 —разряд сотен

Разряды также сообщают нам информацию о количестве единиц, десятков и сотен.
➤ первая позиция — цифра 5 — разряд единиц содержит 5 единиц (1+1+1+1+1)
➤ вторая позиция — цифра 4 —разряд десятков содержит четыре десятка (10+10+10+10)
➤ третья позиция — цифра 3 —разряд сотен содержит три сотни (100+100+100)

Если сложить число единиц, число десятков и число сотен, то получим наше изначальное число 345:
(1+1+1+1+1)+(10+10+10+10)+(100+100+100)

Слова единицы, десятки, сотни, заменяют собой множители 1, 10 и 100.
Например, рассмотрим наше число 345:
➤ в разряде единиц — множитель 1 содержится в разряде единиц 5 раз: 1 × 5 = 5
➤ в разряде десятков — множитель 10  содержится в разряде десятков 4 раза: 10 × 4 = 40
➤ в разряде сотен — множитель 100 содержится в разряде сотен 3 раза: 100 × 3 = 300
Если сложить полученные результаты 5 + 40 + 300, то получим число 345.

Другие группировки

➤ То же самое будет происходить если мы скажем, что число 345 содержит 34 десятка и 3 единицы. Другими словами,
— десятки будут сгруппированы 34 раза: 10 × 34 = 340
— единицы будут сгруппированы 5 раз: 1 × 5 = 5
Если сложить полученные результаты 340 и 5, снова получим число 345.

➤ Также возможно группировка, при которой не будет распределения на десятки и сотни. В таком случае число 345 будет читаться как «триста сорок пять единиц»:  1 × 345 = 345.

 

Старшие разряды

Далее рассмотрим более старшие разряды: тысяч,  десятков тысяч, сотен тысяч, миллионов и т. д.

Рассмотрим число: 1 234 567
разряд единиц — 7 единицы,
разряд десятков — 6 десятков,
разряд сотен — 5 сотен,
разряд тысяч — 4 тысячи,
разряд десятков тысяч — 3 десятка тысяч,
разряд сотен тысяч — 2 сотни тысяч,
разряд миллионов — 1 миллион.

 

Применение разрядов

Применение разрядов при решение примеров  облегчают некоторые вычисления. Например, когда нужно сложить два трехзначных числа, то можно сложить все разряды по отдельности: единицы, десятки и сотни. Начинать нужно с разряда единиц и т.д.

Рассмотрим пример: 123 + 456 =

В первую очередь после знака равно (=) мысленно поставим три точки. Вместо этих точек будет располагаться новое число (наш ответ): 123 + 456 = .  .  .
Начинаем складывать:
1) из разряда единиц берем числа 3 и 6, получаем 9 ➜ 123 + 456 =  .  .  9
2) из разряда десятков берем числа 2 и 5, получаем 7 ➜ 123 + 456 =  . 7 9
3) из разряда сотен берем числа 1 и 4, получаем 5 ➜ 123 + 456 =  5 7 9
Таким образом, если к числу 123 прибавить 456 , получается 579.  

Этот принцип действует, если решать примеры в столбик:

+

1  2  3
4  5  6

 5  7  9

? Важно! Каждый разряд записывается друг под другом: единицы под единицами, десятки под десятками, сотни под сотнями  и т.д. При неправильном размещении разрядов пример будет решен не правильно!

? Чтобы потренироваться решать примеры в столбик на сложение, можно скачать программу «Сложение и вычитание в столбик«. 

 

Переполнение разряда

Разряд характеризуется одной цифрой от 0 до 9. Но иногда при вычислении числового выражения  может произойти переполнение разряда.

Рассмотрим пример: 56+89.
При сложении единиц этих чисел получаем 15 (6+9), а при сложении десятков получаем 13 (5+8).

1) При сложении 9 и 6 единиц у нас получилось 15 единиц — это 1 десяток и 5 единиц.
Поэтому в разряд десятков записываем 5, а 1 десяток мы будем прибавлять к десяткам при их сложении.
2) При сложении десятков получаем 13 (5+8) — это 1 сотня и 3 десятка.
Поэтому в разряд десятков мы запишем 4 (3 десятка + 1 десяток, полученный при сложении единиц). Так как в примере разряд сотней не складывается, то мы просто впереди приписываем единицу.

 
1
5 6 + 8 9 =  .  .  ➜  5 6 + 8 9 =  .  5 ➜  5 6 + 8 9 = 1 4 5

Этот принцип действует также при решении примеров в столбик:


+

       1
     5 6
     8 9

  


+

1  1
3 6 9
2 5 8

 1  4  5  6 2 7

 

Примеры на вычитание

Теперь рассмотрим примеры на вычитание. Вычитать также нужно по разрядам.

Пример: 68–45.
Начинаем с разряда единиц. При вычитании единиц этих чисел получаем 3 (8-5).
Затем вычитаем десятки. При вычитании десятков получаем 2(6-4).
 6 8 – 4 5 =  .  .  ➜  6 8 – 4 5 =  .  3 ➜  6 8 – 4 5 = 2 3

Пример: 42-15.
1) В разряде единиц числа 42 содержится две единицы, а в разряде единиц числа 15 — пять единиц. От двух единиц нельзя вычесть пять единиц, поскольку две единицы меньше, чем пять единиц.
Чтобы выполнить вычитание, две единицы должны взять несколько единиц у соседней группы (разряда десятков) ➜ Поэтому берем один десяток.
Получаем, что мы число 42 разделили на 3 десятка и 12 единиц. Из 12 вычитаем 5 и получаем 7. Записываем цифру 7 в разряде единиц нового числа. 
2) Теперь вычитаем десятки. Поскольку разряд десятков отдал единицам один десяток, сейчас он имеет не четыре, а три десятка. Поэтому вычитаем из 3 десятков один десяток. Останется 2 десятка. Записываем цифру 2 в разряде десятков нового числа.
Чтобы не забывать, что в каком-то разряде был взят один десяток (либо сотня либо тысяча), над этим разрядом принято ставить точку.

 
                      •                                •
4 2 – 1 5 = 4 2 – 1 5 =  . 7 ➜ 4 2 – 1 5 = 2 7

Этот принцип действует также при решении примеров в столбик:



4 2
1 5

  


•  •
8 2 3
5 7 9

 2 7  2 4 4

Во втором примере:
1) Вычитаем единицы. разряде единиц числа 823 содержится 3 единицы, а в разряде единиц числа 579 — 9 единиц. От 3 единиц не вычесть 9 единиц, поэтому берем один десяток у разряда десятков. Ставим точку над разрядом десятков, чтобы помнить о том, что мы взяли оттуда один десяток. Взятый один десяток и три единицы вместе образуют 13 единиц. От 13 единиц вычитаем 9 единиц, получится 4 единицы. Записываем цифру 4 в разряде единиц нового числа.
2) Вычитаем десятки. Раньше разряд десятков числа 823 содержал 2десятка, но мы взяли с него один десяток, и теперь в разряде десятков содержатся 1 десяток. Из 1 десятка нужно вычесть 7 десятков, поэтому берем одну сотню у разряда сотен. Ставим точку над разрядом сотен, чтобы помнить о том, что мы взяли оттуда одну сотню. Взятая 1 сотня и 1 десяток вместе образуют 11 десятков. От 11 десятков вычитаем 7 десятков, получаем 4 десятка. Записываем цифру 4 в разряде десятков нового числа.
3) Вычитаем сотни. Раньше разряд сотен числа 823 содержал 8 сотен, но мы взяли с него 1сотню, и теперь в разряде сотен содержатся 7 сотен. Из 7 сотен можно вычесть 5 сотен, получается 2 сотни. Записываем цифру 2 в разряде сотен нового числа.

? Чтобы потренироваться решать примеры в столбик на вычитание, можно скачать программу «Сложение и вычитание в столбик«. 

 

Вычитание из круглых чисел

Намного сложнее вычитать из чисел вида 100, 200, 300, 1000, 10000. То есть числа, у которых на конце нули. Чтобы выполнить вычитание, каждому разряду приходится занимать десятки/сотни/ тысячи у следующего разряда. 

Выполнять вычитание таким традиционным методом сложно, поэтому можно воспользоваться нестандартным способом. Для этого уменьшить вычитаемое число на одну единицу. Далее из полученного результата вычесть вычитаемое и к полученной разности прибавить единицу.

Пример: 1000–123
1) уменьшим уменьшаемое на 1: 1000–1=999
2) из числа 999 вычтем 123: 999-123=876
3) к этому ответу прибавляем 1: 876+1=877.
Получили: 1000+123=877.

 

 

§ Разряды и классы. Класс единиц, тысяч и миллионов

Разряды и классы Разрядные слагаемые

Для записи чисел люди придумали десять знаков, которые называются цифрами. Это: 0, 1, 2, 3, 4, 5, 6, 7, 8, 9.

С помощью десяти цифр можно записать любое натуральное число.

От количества знаков (цифр) в числе зависит его название.

Число, состоящее из одного знака (цифры), называется однозначным. Наименьшее однозначное натуральное число — «1» , наибольшее — «9».

Число, состоящее из двух знаков (цифр), называется двузначным. Наименьшее двузначное число — «10», наибольшее — «99» .

Числа, записанные с помощью двух, трёх, четырёх и более цифр, называются двузначными, трёхзначными, четырёхзначными или многозначными. Наименьшее трёхзначное число — «100», наибольшее — «999».

Каждая цифра в записи многозначного числа занимает определённое место — позицию.

Запомните!

Разряд — это место (позиция), на котором в записи числа стоит цифра.

Одна и та же цифра в записи числа может иметь разные значения в зависимости от того, в каком разряде она стоит.

Разряды отсчитываются с конца числа.

Разряд единиц — это самый младший разряд, которым заканчивается любое число.

Цифра «5» — означает «5» единиц, если пятёрка стоит на последнем месте в записи числа (в разряде единиц).

Разряд десятков — это разряд, который стоит перед разрядом единиц.

Цифра «5» — означает «5» десятков, если она стоит на предпоследнем месте (в разряде десятков).

Разряд сотен — это разряд, который стоит перед разрядом десятков. Цифра «5» означает «5» сотен, если она стоит на третьем месте от конца числа (в разряде сотен).

Запомните!

Если в числе отсутствует какой-либо разряд, то в записи числа на его месте будет стоять цифра «0» (ноль).

Пример. В числе «807» содержится 8 сотен, 0 десятков и 7 единиц — такая запись называется разрядным составом числа.

807 = 8 сотен 0 десятков 7 единиц

Каждые 10 единиц любого разряда образуют новую единицу более высокого разряда. Например, 10 единиц образуют 1 десяток, а 10 десятков образуют 1 сотню.

Таким образом, значение цифры от разряда к разряду (от единиц к десяткам, от десятков к сотням) увеличивается в 10 раз. Поэтому система счёта (счисления), которую мы используем, называется десятичной системой счисления.

В записи числа разряды, начиная справа, группируются в классы по три разряда в каждом.

Класс единиц или первый класс — это класс, который образуют первые три разряда (справа от конца числа): разряд единиц, разряд десятков и разряд сотен.

Пример.

Числа Класс единиц (первый класс)
сотни десятки единицы
6 6
34 3 4
148 1 4 8
Числа Класс единиц (первый класс)
сотни десятки единицы
6 6
34 3 4
148 1 4 8

Класс тысяч или второй класс — это класс, который образуют следующие три разряда: единицы тысяч, десятки тысяч и сотни тысяч.

Пример.

Числа Класс тысяч (второй класс) Класс единиц (первый класс)
сотни тысяч десятки тысяч единицы тысяч сотни десятки единицы
5 234 5 2 3 4
12 803 1 2 8 0 3
356 149 3 5 6 1 4 9
Числа Класс тысяч (второй класс) Класс единиц (первый класс)
сотни тысяч десятки тысяч единицы тысяч сотни десятки единицы
12 803 1 2 8 0 3
356 149 3 5 6 1 4 9

Напоминаем, что 10 единиц разряда сотен (из класса единиц) образуют одну тысячу (единицу следующего разряда: единицу тысяч в классе тысяч).

10 сотен = 1 тысяча

Класс миллионов или третий класс — это класс, который образуют следующие три разряда: единицы миллионов, десятки миллионов и сотни миллионов.

Единица разряда миллионов — это один миллион или тысяча тысяч (1 000 тысяч). Один миллион можно записать в виде числа «1 000 000».

Десять таких единиц образуют новую разрядную единицу — десять миллионов «10 000 000»

Десять десятков миллионов образуют новую разрядную единицу — сто миллионов или в записи цифрами «100 000 000».

Пример.

Числа Класс миллионов (третий класс) Класс тысяч (второй класс) Класс единиц (первый класс)
сотни миллионов десятки миллионов единицы миллионов сотни тысяч десятки тысяч единицы тысяч сотни десятки единицы
8 345 216 8 3 4 5 2 1 6
93 785 342 9 3 7 8 5 3 4 2
134 590 720 1 3 4 5 9 0 7 2 0
Числа Класс миллионов (третий класс) Класс тысяч (второй класс) Класс единиц (первый класс)
сотни миллионов десятки миллионов единицы миллионов сотни тысяч десятки тысяч единицы тысяч сотни десятки единицы
8 345 216 8 3 4 5 2 1 6
93 785 342 9 3 7 8 5 3 4 2
134 590 720 1 3 4 5 9 0 7 2 0

Как прочитать многозначное число

Запомните!

Чтобы прочитать многозначное число, надо назвать по очереди слева направо число единиц каждого класса и добавить название класса.

Не произносят название класса единиц, а также название класса, все три цифры которого нули.

Например, число «134 590 720» читаем: сто тридцать четыре миллиона пятьсот девяносто тысяч семьсот двадцать.

Число «418 000 547» читаем: четыреста восемнадцать миллионов пятьсот сорок семь.

На нашем сайте для проверки своих результатов вы можете воспользоваться калькулятором разложения числа на разряды онлайн.

Важно!

Чтобы легче запомнить, как читать и записывать многозначные числа, советуем использовать выше приведённую «Таблицу классов и разрядов».


Разряды и классы Разрядные слагаемые


Ваши комментарии
Важно!

Чтобы оставить комментарий, вам нужно войти на наш сайт при помощи «ВКонтакте».

Оставить комментарий:

Отправить

29 октября 2019 в 16:31

Анжела Мурадян Профиль Благодарили: 0
Сообщений: 1

В записи какого числа в разрядах десятков миллионов и десятков стоит одна и та же цифра? 35034204, 16765510, 429561320, 270317817

0 СпасибоОтветить

8 ноября 2019 в 5:08
Ответ для Анжела Мурадян

Светлана Хлебникова Профиль Благодарили: 0
Сообщений: 1

16765510, 429561320

0 СпасибоОтветить

2 октября 2016 в 5:03

Ruslan Salimov Профиль Благодарили: 0
Сообщений: 1

Числа от 1 до 2015 написаны подряд и получилось 1234567891011121314. ..2015. Количество и пользованных цифр? Нужна формула и решение для быстрого решения.спасибо

0 СпасибоОтветить

7 октября 2016 в 19:26
Ответ для Ruslan Salimov

Евгений Фёдоров Профиль Благодарили: 0
Сообщений: 60

Тупо посчитать лень?
9 + 2 · 90  + 3 · 900 + 4 · 1016 =…

0 СпасибоОтветить

20 сентября 2016 в 16:15

Артём Волков Профиль Благодарили: 0
Сообщений: 1

из числа 1234512345123451234512345 вычеркните 10 цифр так, чтобы оставшееся число было максимально возможным.

0 СпасибоОтветить

21 сентября 2016 в 9:44
Ответ для Артём Волков

Евгений Колосов Профиль Благодарили: 12
Сообщений: 197

Уберём сначала числа все цифры, кроме 5 по порядку.
[1234]5[1234]5[12]3451234512345
Полученное число:
553451234512345

0 СпасибоОтветить

29 ноября 2020 в 8:40
Ответ для Артём Волков

Важа Чкареули Профиль Благодарили: 0
Сообщений: 1

345345345345345

0 СпасибоОтветить

17 сентября 2016 в 12:52

Татьяна Алёхина Профиль Благодарили: 0
Сообщений: 1

Укажите наибольшее четное пятизначное число.
а) 987640          б) 99999
в) 888888           г) 99998

0 СпасибоОтветить

19 сентября 2016 в 14:38
Ответ для Татьяна Алёхина

Евгений Колосов Профиль Благодарили: 12
Сообщений: 197

Довольно странное задание для 10 класса.

Пятизначные числа здесь только под буквами б) и г). Число б)-нечетное. Следовательно правильный ответ г).

0 СпасибоОтветить

24 сентября 2018 в 18:00
Ответ для Татьяна Алёхина

Никита Зайцев Профиль Благодарили: 0
Сообщений: 1

В

0 СпасибоОтветить

1 октября 2018 в 16:04
Ответ для Татьяна Алёхина

Артур Байрамов Профиль Благодарили: 0
Сообщений: 1

0 СпасибоОтветить

15 ноября 2018 в 20:03
Ответ для Татьяна Алёхина

Злата Милославская Профиль Благодарили: 0
Сообщений: 1

г

0 СпасибоОтветить

4 сентября 2016 в 19:41

Екатерина Старикова Профиль Благодарили: 0
Сообщений: 1

запишите в виде разрядных слогаемых числа:
2a;mnkl;a15b

0 СпасибоОтветить

19 сентября 2016 в 14:34
Ответ для Екатерина Старикова

Евгений Колосов Профиль Благодарили: 12
Сообщений: 197

Здесь подробно описано.
1) 2a=2 · 10+a
2) mnkl=m · 1000 +n · 100 +k · 10 +l
3)a15b =a · 1000 +100 +5 · 10 +b

0 СпасибоОтветить

7 марта 2016 в 13:42

Юлия Гончарова Профиль Благодарили: 0
Сообщений: 1

Какие числа являются пятой сотни?

0 СпасибоОтветить

8 марта 2016 в 20:09
Ответ для Юлия Гончарова

Юрий Резник Профиль Благодарили: 0
Сообщений: 6

Вероятно, вопрос звучит так: «Какие числа являются числами пятой сотни?».
Ответ таков: все числа, бОльшие 400 и меньшие 500, а также само число 500.
При помощи знаков неравенства ответ можно записать так: х является числом пятой сотни, если 400 < х ? 500.

0 СпасибоОтветить

29 октября 2015 в 15:43

Марья Голдобина Профиль Благодарили: 0
Сообщений: 4

сколько единиц в 10 сотнях?

0 СпасибоОтветить

31 октября 2015 в 13:32
Ответ для Марья Голдобина

Андрей Мохарев Профиль Благодарили: 0
Сообщений: 1

1000

0 СпасибоОтветить

14 ноября 2015 в 19:03
Ответ для Марья Голдобина

Максим Бычков Профиль Благодарили: 0
Сообщений: 2

1000

0 СпасибоОтветить

19 апреля 2015 в 20:15

Софья Макаренкова Профиль Благодарили: 0
Сообщений: 1

Разложить число 304567  по разрядам!

0 СпасибоОтветить


Что такое цифра в математике? Определение, типы, примеры, факты

Определение цифр в математике

Цифры — это отдельные числа, используемые для представления значений в математике. 0, 1, 2, 3, 4, 5, 6, 7, 8 и 9 используются в различных комбинациях и повторениях для представления всех значений в математике.

Любое из десяти чисел от 0 до 9 может быть представлено символом, известным как цифра.

Пример двузначного (2-D) числа равен 65. Оно состоит из 6 и 5.  

Родственные игры

История

В древние времена у людей не было системы счисления или цифр для измерения или подсчета вещей. По мере того, как мир расширял свои корни в такие предметы, как наука, и торговля между странами росла, возникла настоятельная потребность в единой системе счисления. Таким образом формировались и объединялись цифры для использования в разных ситуациях.

Мы используем международные цифры, такие как «123» и «65», но римляне использовали римские цифры, и на протяжении всей истории использовалось много других цифр.

Связанные листы

Разрядное значение

В математике каждая цифра в числе имеет разрядное значение. Значение места может быть определено как значение, представленное цифрой в числе на основе ее положения в числе.

Например, разряд 7 в числе 3743 равен 7 сотням или 700. Однако разряд 7 в числе 7432 равен 7 тысячам или 7000. Здесь мы видим, что хотя 7 одинакова в обоих числах, его позиционное значение изменяется с изменением его положения.

Номинальная стоимость

Местная стоимость и номинальная стоимость не совпадают. Номинальное значение цифры — это значение цифры, а разрядное значение цифры — это ее место в числе. Проще говоря, номинальная стоимость сообщает фактическую стоимость, тогда как стоимость места сообщает стоимость на основе ее положения. Следовательно, номинал цифры никогда не меняется независимо от ее положения в числе, тогда как ее разрядное значение меняется с изменением ее положения.

Например, номинальная стоимость 2 в обоих числах 283 и 823 равна 2. В то время как разрядная стоимость 2 равна 200 в 283 и 20 в 823. 

Пример цифр:

  • Двузначные числа

Двузначные числа начинаются с 10 и заканчиваются на 99. Десятки должны стоять между 1 и 9.

Рассмотрим два числа, 15 и 37. При сложении этих двух чисел получается новое число, 52.

15 + 37 = 52

  • Четырехзначные числа

Когда четыре цифры записываются вместе, получается четырехзначное число. Диапазон этих чисел от 1000 до 9999.

Рассмотрим числа 1001, 2001, 5000 и 1040. При сложении этих чисел получается новое четырехзначное число 9042. в Примеры, упомянутые выше, по мере увеличения цифр в числе увеличивается его значение. Например, 10 000 — это 5-мерное число, значение которого больше всех 4-D натуральных чисел.

Решенные примеры

Пример 1: Сколько цифр в числе 1458?

Решение : Число 1458 состоит из четырех цифр: 1, 4, 5 и 8.

Пример 2. Используя цифры 6, 6, 8, найдите наибольшее $3-$-значное число.

Решение . Наибольшее трехзначное число, которое можно составить из них, равно 866. в 84 527 — 4000 (четыре тысячи).

Практические задачи

1

Какое из этих чисел составляет наибольшее трехзначное число?

1, 0

2

9

9, 1

Правильный ответ: 9
Наибольшее трехзначное число 999 состоит из 9, повторенных трижды.

2

Какой из них стоит на десятитысячном разряде в числе 783 425?

7

4

8

5

Правильный ответ: 8
8 стоит в разряде десятков тысяч.

3

Какое десятичное число в 36,2?

6

8

2

Правильный ответ: 2
В данном числе после запятой стоит 2. Следовательно, это десятичное число.

Часто задаваемые вопросы

В чем разница между десятичными дробями и цифрами?

Термин «цифры» относится к набору действительных чисел, включая ноль и все положительные счетные числа. Дроби, отрицательные целые числа и десятичные дроби не рассматриваются как цифры.

Десятичная дробь — это число, которое ставится справа после точки (. ) в числе.

Например, в числе 23,8 8 — десятичное число.

Из каких цифр состоят числа?

От 0 до 9 используются в различных комбинациях для образования цифр.

Являются ли дроби частью цифр?

Дроби представляют собой определенные части целой значащей цифры. Они лежат между двумя значащими числами. Они являются частью системы счисления, но не обязательно частью значащих цифр.

Кто ввел понятие цифры?

Определенного имени нет. Считается, что ноль был впервые использован майя в 4 году нашей эры.

Цифра или число? | Британский словарь

Спросите редактора

Вопрос

Цифра или число?

Отвечать

Вопрос


Читатель недавно спросил: «В чем разница между цифрой и цифрой


Ответ


Хороший вопрос! Слова цифра и номер могут сбивать с толку, потому что они тесно связаны и часто используются в одном и том же контексте, как в этих примерах:

  • Дэвид попросил ее телефон номер и она достала лист бумаги и что-то написал. Но когда она отдала его Дэвиду, чего-то не хватило; было всего шесть цифр .
  • Пожалуйста, пришлите мне по электронной почте ваше 10-значное подтверждение номер .


Однако, цифра и цифра имеют совершенно разные значения.

Цифра

Цифра — это любой из следующих символов: 0, 1, 2, 3, 4, 5, 6, 7, 8, 9. Например, число 23 записывается двумя цифрами. , 2 и 3.

Число

Число — это количество чего-либо. Он может быть записан одной или несколькими, или многими цифрами. Цифры также можно записывать словами. Номер также может быть серией цифр, которая используется для идентификации чего-либо, например номера телефона, номера квартиры или номера социального страхования.

Это все примеры чисел:

  • 2
  • 49
  • сорок девять
  • 12 446
  • тысяча девятьсот пятьдесят шесть
  • 212-555-6051

 

Надеюсь, это поможет.

Уравнение по математике решить: Решение уравнений — урок. Математика, 6 класс.

Решение математических уравнений с помощью помощника по преобразованию в математические выражения в OneNote

OneNote

Создание заметок

Создание заметок

Решение математических уравнений с помощью помощника по преобразованию в математические выражения в OneNote

OneNote для Интернета OneNote для Windows 10 Помощник по преобразованию в математические выражения Еще…Меньше

Напишите или введите любую математическую задачу, и помощник по преобразованию в математические выражения в OneNote решит ее для вас: поможет быстро найти решение или покажет пошаговые инструкции, которые помогут вам найти решение самостоятельно. После решения уравнения можно пользоваться различными возможностями изучения математики в помощнике по преобразованию в математические выражения.

Примечание: Эта функция доступна только в том случае, если у вас есть подписка на Microsoft 365. Если вы являетесь подписчиком Microsoft 365, убедитесь, что у вас установлена последняя версия Office.

Шаг 1. Введите уравнение.

На вкладке Рисование напишите или введите уравнение. Используйте средство Lasso Select для рисования круга вокруг уравнения. Затем выберите «Математические вычисления». Откроется панель помощника по преобразованию в математические выражения.

Дополнительные сведения: создание уравнения с помощью рукописного ввода или текста. 

Шаг 2. Решите уравнение.

Чтобы решить текущее уравнение, выполните одно из следующих действий:

Щелкните или нажмите поле Выберите действие, затем выберите нужное действие помощника по преобразованию в математические выражения. Варианты, которые доступны в этом раскрывающемся меню, зависят от выбранного уравнения.

Дополнительные сведения: проверьте вкладку «Поддерживаемые уравнения » и типы проблем, поддерживаемые Помощником по математическим вычислениям

Просмотрите решение, которое OneNote выводит под выбранным действием. В приведенном ниже примере решение отображается для выбранного варианта Решение для x.

  • Чтобы узнать, как OneNote решил задачу, можно щелкнуть или нажать Показать действия, а затем выбрать интересующие вас сведения. Варианты, которые доступны в этом раскрывающемся меню, зависят от выбранного уравнения.

  • Чтобы прослушать шаги по решению вслух, Иммерсивное средство чтения запустить его из OneNote.

  • Создайте учебный тест, чтобы научиться решать уравнения такого типа.

Предупреждение:  Тест создания практики в настоящее время недоступен, так как мы работаем над оптимизацией работы. Возможность создавать тесты для практики будет возвращена позже в этом году. 


Совет: Этапы решения можно перетащить в любое место на странице.

Дополнительные сведения

Создание математических формул с помощью рукописного ввода или текста в OneNote с использованием помощника по преобразованию в математические выражения

Типы задач, поддерживаемые помощником по преобразованию в математические выражения

Построение графиков математических функций в OneNote с использованием помощника по преобразованию в математические выражения

Создание учебного теста в OneNote с использованием помощника по преобразованию в математические выражения

Типы проблем, поддерживаемые помощником по математическим вычислениям

При использовании помощника по математике в OneNote вы заметите, что раскрывающийся список «Выбрать действие» под уравнением изменяется в зависимости от выбранного уравнения. Ниже приведены некоторые типы проблем, поддерживаемые в зависимости от формулы, которую вы пытаетесь решить.

Примечание: Эта функция доступна только в том случае, если у вас есть подписка на Microsoft 365. Если вы являетесь подписчиком Microsoft 365, убедитесь, что у вас установлена последняя версия Office.

Массивы

Для списка реальных чисел поддерживаются все приведенные ниже значения.
 

  • Оценить

  • Сортировка

  • org/ListItem»>

    Среднее

  • Медиана

  • Мода

  • Сумма

  • Продукт

  • Наибольший распространенный фактор

  • org/ListItem»>

    Наименее распространенное кратное

  • Дисперсия

  • Стандартное отклонение

  • Минимумы

  • Maxima

    Для полиномиалов поддерживаются наиболее распространенные и наименее распространенные действия. Для просмотра графиков всех функций можно также выбрать Graph в двухd.

Выражения

Для любого выражения доступны следующие действия:

  • Оценить

  • Проверяем подлинность

  • Развернуть (если применимо)

  • org/ListItem»>

    Фактор (если применимо)

  • Граф в 2D (доступен только при наличии переменной)

  • Дифференцирование (доступно только при наличии переменной)

  • Интеграция (доступна только при наличии переменной)

Уравнения и неравенство

Для уравнений и неравенства доступны следующие действия:

  • org/ListItem»>

    Решение для {переменной}

  • Граф обеих сторон в 2D — каждая из сторон равенства или неравенства определяется как отдельная функция.

  • Граф в 2D — граф решений уравнения или неравенства

  • Неравенство графов — помечает область решения на диаграмме

Системы

Важно иметь одинаковое количество уравнений и переменных, чтобы обеспечить доступность правильных функций. Системы могут быть написаны двумя разными способами: 

  1. Одна под другой, с большой

    фигурной скобки или без нее перед

  2. В одной строке, разделенной запятой


Производные и целочисленные объекты

Производные могут быть записаны либо с помощью d/dx перед функцией, либо с помощью простого знака.

Действия, доступные для производных и целочисленных элементов:

Матриц

Матрицы могут быть написаны квадратными или круглыми скобками. Для матриц поддерживаются следующие действия:

  • Оценить

  • Вычисление детерминантного значения

  • Инвертированная матрица

  • org/ListItem»>

    Вычисление трассировки

  • Транспонирование матрицы

  • Размер матрицы

  • Уменьшение матрицы

Матричные уравнения в настоящее время не поддерживаются.

График в полярных координатах

Для графирования функции в полярных координатах r должен быть выражен как функция theta.

Сложный режим

Примечание. Выберите параметры для переключения между реальными числами и сложными числами.

Для сложных выражений и чисел, содержащих мнимую единицу i, доступны следующие действия.

Дополнительные сведения

Создание математического теста в Microsoft Forms

Создание математического теста для практики в OneNote с использованием помощника по преобразованию в математические выражения

Решение математических уравнений с помощью помощника по преобразованию в математические выражения в OneNote

Что такое уравнение? Как решать уравнения?

        Уравнение — одно из краеугольных понятий всей математики. Как школьной, так и высшей. Имеет смысл разобраться, правда? Тем более, что это очень простое понятие. Ниже сами убедитесь. 🙂 Так что же такое уравнение?

        То, что это слово однокоренное со словами «равный», «равенство», возражений, думаю, ни у кого не вызывает.

        Уравнение — это два математических выражения, соединённых между собой знаком «=» (равно).

        Но… не каких попало. А таких, в которых (хотя бы в одном) содержится неизвестная величина. Или, по-другому, переменная величина. Или, сокращённо, просто «переменная». Которая обычно обозначается буквой «х».

        Переменных может быть одна, может быть несколько. В школьной математике чаще всего рассматриваются уравнения с одной переменной. И мы тоже пока что будем рассматривать уравнения с одной переменной. С двумя переменными или более — в специальных уроках.

 

Что значит решить уравнение?

        Идём дальше.

        Переменная, входящая в уравнение, может принимать любые допустимые математикой значения. На то она и переменная. 🙂 При каких-то значениях переменной получается верное числовое равенство, а при каких-то — нет.

        Так вот:

        Решить уравнение означает найти ВСЕ такие значения переменной, при подстановке которых в исходное уравнение получается верное равенство. Или, более научно, верное тождество. Или доказать, что таких значений переменной не существует.

 

        Что такое верное равенство? Это равенство, не вызывающее сомнений даже у человека, абсолютно не отягощённого глубокими математическими познаниями. Например, 5=5, 0=0, -10=-10. И так далее. 🙂

        Значения переменной, при подстановке которых достигается это самое верное равенство, называются очень красиво и научно — корни уравнения.

        Корень может быть один, может быть несколько. А может быть и бесконечно много корней — целый интервал или даже вообще вся числовая прямая от –∞ до +∞. Да, такое тоже бывает! Всё от конкретного уравнения зависит.)

        А бывает и такое, что нельзя найти такие иксы, которые давали бы нам верное равенство. Принципиально нельзя. По определённым причинам. Нету таких иксов…

        В таких случаях обычно говорят, что уравнение не имеет корней.

 

Для чего нужны уравнения?

        Вопрос смешной. Для жизни! В школе, как правило, уравнения нужны для решения текстовых задач. Это, напоминаю, задачи на движение, на работу, на проценты и многие другие.

        А во взрослой жизни без уравнений невозможны было бы ответить даже на самые обычные, но жизненно важные вопросы повседневности: какая будет погода завтра, выдержит ли заданную нагрузку здание. Или лифт. Или самолёт. Куда попадёт ракета… И не было бы сейчас среди нас ни синоптиков, ни инженеров, ни бухгалтеров, ни экономистов, ни программистов… За ненадобностью. Внушает?)

        Почему это так? А потому, что уравнениями описываются почти все известные человеку природные явления и процессы. Изменение давления и температуры воздуха с высотой, закон всемирного тяготения, размножение бактерий, радиоактивный распад, химические реакции, электричество, спрос и предложение — в основе всего этого лежат математические уравнения! Простые, сложные — всякие. Какое явление или ситуация, такое и уравнение.)

        Итак, запоминаем:

        Уравнения — очень мощный и универсальный инструмент для решения самых разных прикладных задач.

 

А какие бывают уравнения?

        Уравнений в математике несметное количество. Самых разных видов. Но всё многообразие уравнений можно условно разделить всего на 4 категории:

1. Линейные,

2. Квадратные,

3. Дробные (или дробно-рациональные),

4. Прочие.

        Разные категории уравнений требуют и разного подхода к их решению: линейные уравнения решаются одним способом, квадратные — другим, дробные — третьим, тригонометрические, логарифмические, показательные и прочие — тоже решаются своими методами.

        Прочих уравнений, разумеется, больше всего, да…) Это и иррациональные, и тригонометрические, и показательные, и логарифмические, и многие другие уравнения. И даже дифференциальные уравнения (для студентов), где роль неизвестного играет не число, а функция. Или даже семейство функций. 🙂

        В соответствующих уроках мы подробно разберём все эти типы уравнений. А здесь у нас — базовые приёмы и правила.

        Называются эти правила — тождественные (или — равносильные) преобразования уравнений. Их всего два. И нигде их не обойти. Так что знакомимся!

 

Как решать уравнения? Тождественные (равносильные) преобразования уравнений.

        Решение любого уравнения заключается в поэтапном преобразовании входящих в него выражений. Но преобразований не абы каких, а таких, чтобы от шага к шагу суть всего уравнения не менялась. Несмотря на то, что после каждого преобразования уравнение будет видоизменяться и, в конечном счёте, станет совсем не похоже на исходное.

        Такие преобразования в математике называются равносильными или тождественными. Их довольно много, но среди всего многообразия тождественных преобразований уравнений выделяется два базовых. О них и пойдёт речь в этом уроке. Да-да, всего два! Но — крайне важных! И каждое из них заслуживает отдельного внимания.

        Применение этих двух тождественных преобразований в том или ином порядке гарантирует успех в решении 99% уравнений математики. Заманчиво, правда?

        Итак, вперёд!

 

        Первое тождественное преобразование:

        К обеим частям уравнения можно прибавить (или отнять) любое (но одинаковое!) число или выражение (в том числе и с переменной). Суть уравнения от этого не изменится.

 

        Это преобразование вы применяете всюду, наивно думая, что переносите какие-то члены из одной части уравнения в другую, меняя знаки. 🙂

        Например, такое крутое уравнение:

        

        Тут и думать нечего, перебрасываем тройку вправо, меняя минус на плюс:

        

        А что же происходит в действительности? А на самом деле вы… прибавляете к обеим частям уравнения тройку!

        Вот что у вас происходит:

        

        И результат получается тем же самым:

        

        Вот и всё. Слева остаётся чистый икс (чего мы, собственно, и добиваемся), а справа — что уж получится. Но самое главное то, что от прибавления тройки к обеим частям суть всего уравнения не изменилась!

        Дело в том, что привычный нам перенос слагаемых из одной части в другую со сменой знака — это просто сокращённый вариант первого тождественного преобразования.

        И зачем нам так глубоко копать? В уравнениях — незачем. Переносите себе спокойно и не парьтесь. Только знаки менять не забывайте.) А вот в неравенствах привычка к переносу может и слегка обескуражить, да…

        Это было первое тождественное преобразование. Переходим ко второму.

 

        Второе тождественное преобразование:

        Обе части уравнения можно умножить (разделить) на одно и то же отличное от нуля число или выражение.

 

        Это тождественное преобразование мы вы постоянно применяете, когда решаете что-нибудь совсем уж жуткое типа:

        

        Тут каждому ясно, что х=3. А вот как вы получили этот ответ? Подобрали? Угадали?

        Чтобы не подбирать и не гадать (мы с вами математики, а не гадалки), нужно понять, что вы просто поделили обе части уравнения на четвёрку. Которая нам и мешает.

        Вот так:

        

        Эта палка с делением означает, что на четвёрку делятся обе части нашего уравнения. Через дроби эта процедура выглядит так:

        

        Слева четвёрки благополучно сокращаются, остаётся икс в гордом одиночестве. А справа при делении 12 на 4 получается, понятное дело, тройка. 🙂

        И все дела.)

        Звучит невероятно, но эти два (всего два!) простых преобразования лежат в основе решения всех уравнений математики! Да-да, именно всех, я нисколько не преувеличиваю! От линейных и квадратных в школе до дифференциальных в ВУЗе.)

        Ну что, посмотрим на тождественные преобразования уравнений в действии?

 

Применение тождественных преобразований к решению уравнений.

        Начнём с первого тождественного преобразования. Переноса вправо-влево.

        Пример для новичков:

        1 — х = 3 — 2х

        Дело нехитрое. Это линейное уравнение. Работаем прямо по заклинанию: «С иксами влево, без иксов — вправо».

        Эта мантра — универсальная инструкция по применению первого тождественного преобразования. Вот и смотрим на уравнение. Какое слагаемое с иксом у нас справа? Что? ? Не-а!) Справа у нас -2х (минус два икс)! Поэтому при переносе в левую часть минус поменяется на плюс:

        1 — х +2х = 3

        Полдела сделано, иксы собрали слева. Осталось все числа собрать справа. Слева в уравнении стоит единичка. Опять вопрос — с каким знаком? Ответ «с никаким» не катит.) Слева перед единицей и вправду ничего не написано. А это значит, что перед ней стоит знак «плюс». Так уж в математике повелось: ничего не написано — значит, плюс.)

        И поэтому вправо единичка перенесётся уже с минусом:

        -х + 2х = 3 — 1

        Вот почти и всё. Слева приводим подобные, а справа — считаем. И получаем:

        х = 2

        Это было совсем примитивное уравнение.

 

        Теперь пример покруче, для старшеклассников:

        Решить уравнение:

               

        Уравнение логарифмическое. Ну и что? Какая разница? Всё равно первым шагом делаем базовое тождественное преобразование («С иксами влево ….»). Для этого слагаемое с иксом (то есть, log3x) переносим влево. Со сменой знака:

               

        А числовое выражение (log34) переносим вправо. Также со сменой знака, разумеется:

               

        Вот и всё. Справа получилась чистая формула. Кто дружит с логарифмами, тот в уме дорешает уравнение и получит:

        х=3

       

        Что? Хотите синусы? Пожалуйста, вот вам синусы:

               

        И снова всё то же самое! Выполняем первое тождественное преобразование — переносим sin x влево (с минусом), а -0,25  переносим вправо (с плюсом):

               

        Получили простейшее тригонометрическое уравнение с синусом, решить которое (для знающих) также не составляет никакого труда.

        Видите, насколько универсально первое равносильное преобразование! Встречается везде и всюду и не обойти его никак… Именно поэтому так важно уметь его делать на автомате и без ошибок.

        Собственно, ошибиться здесь можно лишь в одном — забыть сменить знак при переносе. Что и происходит сплошь и рядом. Внимательность никто не отменял, да…)

 

        Ну что, продолжаем наши игры? Развлекаемся теперь со вторым преобразованием!)

        Решить уравнение:

        7х=28

        Крутяк, прямо скажем. ) Ладно, это эмоции…

        Смотрим и соображаем: что нам мешает в этом уравнении? Что-что… Да семёрка мешает! Хорошо бы от неё избавиться. Да так, чтобы исходное уравнение не испортить.)

        Но как? Перенести вправо? Ээээ… Стоп! Нельзя.) Семёрка с иксом умножением связана. Коэффициент, видите ли.) Нельзя её оторвать от икса и вправо перенести. Вот всё выражение целиком — пожалуйста (вопрос — зачем?). А семёрку отдельно — никак нет.

        Самое время про умножение/деление вспомнить! Нам ведь в ответе чистый икс нужен, не так ли? А семёрка — мешает. Вот и делим левую часть на семь. «Очищаем» икс от коэффициента. Так нам надо. Но тогда и правую часть тоже надо поделить на семь: этого уже математика требует. Что уж там получится, то и получится. Но пример хороший. Я старался.) 28 на 7 замечательно делится. Получится 4.

        Ответ: х=4

 

        Или такое уравнение:

        

        Что здесь нам мешает? Дробь 1/6, не так ли? Вот давайте и избавимся от неё. Безопасно для уравнения.) Как? Ну, можно поступить аналогично — поделить обе части на эту самую 1/6. Но в уме это не очень удобно. Кое-кто и запутается…

        Но мы же не только делить, мы ещё и умножать умеем!) Вспоминаем из младших классов, после какого действия у нас пропадает дробь? Правильно! Дробь у нас пропадает при умножении на число, равное (или кратное) её знаменателю. Вот и умножим обе части нашего уравнения на 6. Слева всё равно чистый икс получится, а умножение правой части на 6 — не самая трудная работа.)

        

        Вот и всё.) Умножение обеих частей уравнения на нужное число позволяет сразу избавляться от дробей, минуя промежуточные выкладки, в которых, между прочим, запросто можно и ошибок наляпать. Короче дорога — меньше ошибок!

 

        Теперь снова на машину времени и — в старшие классы:

        Решить уравнение:

        

        Чтобы добраться до икса и тем самым решить это крутое тригонометрическое уравнение, нам надо сначала получить слева чистый косинус, безо всяких коэффициентов. А двойка мешает. 🙂 Вот и делим на 2 всю левую часть:

        

        Но тогда и правую часть тоже придётся разделить на двойку: это уже МАТЕМАТИКЕ надо. Делим:

        

        Получили справа табличное значение косинуса. И теперь уравнение решается за милую душу.)

        

       

        Вот и вся премудрость. Как видите, тождественные преобразования уравнений — штука полезная. И при этом не самая сложная. Перенос да умножение/деление. Однако далеко не у всех они получаются с первого раза и без ошибок, ох не у всех… Основные проблемы здесь две.

        Проблема первая (для малоопытных):

        Иногда ученик думает, что упрощение уравнений делается по одному, раз и навсегда установленному правилу. И никак не может уловить и понять это правило: в каких-то примерах начинают с домножения (или деления), в каких-то — с переноса. Где-то три раза переносят и ни разу не домножают…

        Например, такое линейное уравнение:

        10х + 5 = 5х — 20

        С чего начинать? Можно начать с переноса:

        10х — 5х = -20 — 5

        А можно сначала поделить обе части на пятёрку, а затем уж переносить. Тогда сразу числа попроще станут:

               

        Как видим, и так и сяк решать можно. И это — в примитивном примере! Вот и возникает у неопытных учеников вопрос: «Как правильно?»

        По-всякому правильно! Кому как удобнее. 🙂 Универсального рецепта здесь нет и быть не может. Математика предлагает вам на выбор два вида преобразований уравнений. А порядок этих самых преобразований зависит исключительно от исходного уравнения, а также от личных предпочтений и привычек решающего.

 

        Проблема вторая (для всех…ну… почти):

        Ошибки в вычислениях. В преобразованиях постоянно приходится перемножать скобки. Заключать выражения в скобки и раскрывать скобки. Умножать и делить дроби. Работать со степенями… Короче, в наличии весь набор элементарных действий математики. Со всеми вытекающими…

        Обе эти проблемы устраняются только одним способом — практикой. Исчезают сомнения и ошибки. Примеры становятся проще, задания — легче. И в итоге не математика командует вами, а вы — математикой. 🙂

Решение простых уравнений

Решая простое уравнение, думайте об уравнении как о балансе, где знак равенства (=) является точкой опоры или центром. Таким образом, если вы делаете что-то с одной частью уравнения, вы должны сделать то же самое с другой стороной. Выполняя одно и то же действие с обеими сторонами уравнения (скажем, прибавляя по 3 к каждой стороне), уравнение остается сбалансированным.

Решение уравнения – это процесс получения искомого или решения по одну сторону знака равенства, а всего остального по другую. Вы действительно сортируете информацию. Если вы решаете для x , вы должны получить x  с одной стороны.

Уравнения сложения и вычитания

Некоторые уравнения включают только сложение и/или вычитание.

Пример 1

Найдите x .

x + 8 = 12

Чтобы решить уравнение x + 8 = 12, вы должны получить x отдельно с одной стороны. Следовательно, вычтите 8 с обеих сторон.

Чтобы проверить свой ответ, просто подставьте его в уравнение:

 

Пример 2

Найдите y .

г  – 9 = 25

Чтобы решить это уравнение, вы должны получить y отдельно с одной стороны. Поэтому прибавьте 9 к обеим сторонам.

Для проверки просто замените y на 34:

 

Пример 3

Найдите x .

x  + 15 = 6

Чтобы решить, вычтите 15 с обеих сторон.

Для проверки просто замените x на –9 :

.

 

Обратите внимание, что в каждом случае выше используются противоположные операции  ; то есть, если в уравнении есть сложение, вы вычитаете из каждой стороны.

Уравнения умножения и деления

Некоторые уравнения включают только умножение или деление. Обычно это происходит, когда переменная уже находится в одной части уравнения, но имеется более одной переменной, например 2  x или часть переменной, например

или

Таким же образом, как при сложении или вычитании, вы можете умножать или делить обе части уравнения на одно и то же число  , если оно не равно нулю , и уравнение не изменится.

Пример 4

Найдите x .

3 x = 9

Разделите каждую часть уравнения на 3.

Для проверки замените x на 3:

 

Пример 5

Найдите y .

Чтобы решить, умножьте каждую сторону на 5.

Для проверки замените y на 35:

 

Пример 6

Найдите x .

Чтобы решить, умножьте каждую сторону на .

Или без отмены

 

Обратите внимание, что слева вы обычно не пишете, потому что это всегда отменяется до 1  x или x .

Комбинации операций

Иногда для решения уравнения приходится использовать более одного шага. В большинстве случаев сначала выполните шаг сложения или вычитания. Затем, после того как вы отсортировали переменные с одной стороны, а числа с другой, умножьте или разделите, чтобы получить только одну из переменных (то есть переменную без номера или 1 перед ней:  x , а не 2 x ).

Пример 7

Решите для х .

2 x + 4 = 10

Вычтите 4 с обеих сторон, чтобы получить 2 x отдельно с одной стороны.

Затем разделите обе части на 2, чтобы получить x .

Для проверки подставьте свой ответ в исходное уравнение:

 

Пример 8

Найдите x .

5x  – 11 = 29

Добавьте 11 с обеих сторон.

Разделите каждую сторону на 5.

Для проверки замените x на 8:

 

Пример 9

Найдите x .

Вычтите 6 с каждой стороны.

Умножить каждую сторону на .

Для проверки замените x на 9:

Пример 10

Найдите y .

Добавьте 8 с обеих сторон.

Умножить каждую сторону на .

Для проверки замените y на -25:

Пример 11

Найдите x .

3 x + 2 = x + 4

Вычесть 2 с обеих сторон (то же самое, что добавить -2).

Вычтите x  с обеих сторон.

Обратите внимание, что 3  x  –  x  равно 3  x  – 1  x .

Разделите обе части на 2.

Для проверки замените x на 1:

 

Пример 12

Найдите y .

5 у + 3 = 2 у + 9

Вычесть 3 с обеих сторон.

Вычтите 2  y  с обеих сторон.

Разделите обе части на 3.

Для проверки замените y на 2:

 

Иногда вам нужно упростить каждую сторону (объединить одинаковые термины) перед тем, как начать процесс сортировки.

Пример 13

 Найти x .

3 x + 4 + 2 = 12 + 3

Во-первых, упростите каждую сторону.

Вычесть 6 с обеих сторон.

Разделите обе части на 3.

Для проверки замените x на 3:

Пример 14

Найдите x .

4 x + 2 x + 4 = 5 x + 3 + 11

Упростите каждую сторону.

6 x + 4 = 5 x + 14

Вычесть 4 с обеих сторон.

Вычтите 5 x с обеих сторон.

Для проверки замените x на 10:

Лучшие приложения и сервисы для решения математических уравнений на 2022 год

Учиться тяжело всем. Это долгий и утомительный процесс, и мы уверены, что вы бы предпочли заняться чем-нибудь другим.

Математика — отличный пример того, насколько сложной может быть учеба. У многих то, чему они научились в школе, не приходит в голову к тому времени, когда они возвращаются домой и начинают делать домашнее задание. И учителя хотят, чтобы вы показывали свою работу, что кажется глупым в эпоху цифровых технологий.

Хотя математика является важным навыком, многие сложности можно решить с помощью математических приложений. Хорошее приложение для решения математических задач поможет вам быстро найти правильные решения, а многие даже показывают шаги, которые потребовались для этого. Лучшие приложения полезны для широкого круга математических дисциплин, таких как алгебра, геометрия, тригонометрия, статистика и т. д.

Мы покажем вам, как решить любую математическую задачу с помощью некоторых невероятных пошаговых приложений для решения математических задач для вашего Mac, независимо от того, какой уровень математики вы изучаете в школе.

Как решить мою математическую задачу?

Прокладывая свой путь в сфере образования, вы узнаете все тонкости математики, что поможет решать уравнения вручную и понимать, как решать математические задачи. Как только вы поймете, как работает математика, знание того, как решать уравнения на бумаге, скорее всего, будет пустой тратой времени. Это особенно верно, когда вы приступаете к продвинутой курсовой математике. В большинстве случаев утомительное написание задач от руки не нужно, нежелательно для инструкторов и требует много времени.

Вот несколько популярных вариантов решения математических уравнений:

  • Продолжайте решать математические задачи шаг за шагом на бумаге . Если вы хотите положить карандаш на бумагу, будьте нашим гостем — но есть более простые способы!
  • Посетите сайт бесплатного калькулятора алгебры . Есть много сайтов, которые позволяют вам вводить математические уравнения, чтобы увидеть решение. Большинство из них бесплатны для использования и могут предлагать пошаговые решения, но вы зависите от подключения к Интернету. Также неясно, используются ли ваши данные для чего-то еще, или сайт загружает вредоносное ПО на ваш компьютер без вашего ведома. За использование веб-сайта также может взиматься высокая плата за подписку.
  • Использовать Excel . Вы можете использовать Microsoft Excel для некоторых математических вычислений, но для этого требуется много настроек, а интерфейс сильно отличается от других математических приложений и служб. Excel не предназначен для чистой математики, и это видно. Вы также не сможете экспортировать свои расчеты в формате, пригодном для исследовательских работ.
  • Возьмите хороший калькулятор . Ни для кого не секрет, что многие студенты предпочитают решать математические уравнения с помощью калькулятора. Начиная с алгебры, почти у каждого ученика в сумке есть калькулятор. Это хорошие машины, ручные машины для математики, но есть способ получше.
  • Загрузите собственное приложение . Хорошее родное математическое приложение для вашего Mac может быть любым и всем, что вам нужно. Это может быть калькулятор, инструмент для рисования и надежное средство решения математических задач с подробно изложенными шагами. Лучше всего то, что вам не нужно подключение к сети для использования собственных приложений!

Лучшие математические приложения для решения уравнений для Mac

В математике уравнения определяются строго знаком равенства: =. Когда вы видите знак равенства, у вас есть уравнение. Но это еще не все, что есть в уравнении, особенно в алгебраической математике и не только.

На снимке экрана выше первые пять (5) — это «коэффициент», а «x» называется «переменной». Знаки вычитания и плюса (-, +) определяются как «операторы», а последние пять (5) являются константой. Решение, ноль (0), также является константой. Все, что находится слева от знака равенства, называется «выражением».

Каждая сторона оператора определяется как «термин». В приведенном выше уравнении 5x, 8x, 5 и 0 являются членами. После введения оператора терм считается закрытым. Чтобы быть правильным, все в левой части уравнения должно быть равно решению в правой части знака равенства. Такие приложения, как MathKey, быстро решают такие задачи.

Математические уравнения сложны, и это одно из самых распространенных препятствий, которое приходится преодолевать каждому ученику. Как и MathKey, существует множество отличных приложений, которые могут помочь учащимся решать математические уравнения и создавать потрясающие отчеты для занятий.

Вы можете спросить: «Какое приложение подходит для решения моей математической задачи?» Как и математические задачи, приложения бывают самые разнообразные. Лучше сначала подумать о своих потребностях. Здесь мы обсудим приложения для решения математических задач и алгебраические калькуляторы, оба из которых полезны в зависимости от уровня математики, который вы изучаете, и ваших потребностей в ваших классах.

Лучшее средство решения математических уравнений

Опытные студенты-математики могут воспользоваться преимуществами MathKey и его невероятного движка LaTex. Другие на этом уровне и чуть ниже могут вместо этого использовать PocketCAS, который имеет некоторые мощные функции для студентов-математиков уровня колледжа.

PocketCAS позиционирует себя как «калькулятор TI-89, встроенный в вашу macOS», и это справедливая оценка. Он может строить графики в 2D или 3D, имеет встроенный редактор визуальных матриц, а также встроенные константы и символы, которые помогут вам вводить точные расчеты, которые вам нужны. Если вы предпочитаете использовать свой iPhone или iPad, PocketCAS синхронизируется со своими приложениями для iOS и iPadOS через iCloud, чтобы вся ваша работа была в одном месте.

Первый пример, который мы предоставили, был алгебраическим, но как насчет более сложной математики, такой как тригонометрия? PocketCAS идеален. Его богатая библиотека символов поможет вам ввести любое уравнение, а также принимает ввод с клавиатуры. Когда вы вводите уравнения, PocketCAS идентифицирует компоненты этой переменной и меняет их цвет. В приведенном ниже примере «загар» стал фиолетовым, когда он был введен в PocketCAS:

Поскольку PocketCAS принимает ввод с клавиатуры, используйте встроенное сочетание клавиш (опция + 0) для символов градусов как принятых. приложение также может отображать графики, создавать 2D- или 3D-графики и составлять сценарии для ваших уравнений с помощью C-подобного механизма поддержки сценариев. Еще одна интересная особенность PocketCAS заключается в том, что когда вы делаете ошибку, просто дважды щелкнув решение в приложении, вы вернетесь в поле ввода уравнения, где вы сможете внести изменения.

Еще одно замечательное приложение для Mac, iPhone и iPad — Euclid. Он следует за более знакомым расположением функций, цифр и операторов в главном окне с решением вверху. Из всех приложений-калькуляторов, доступных для Mac, это самое похожее на калькулятор!

Евклид также невероятно силен. Одной из ключевых особенностей является то, что Euclid позволяет вам использовать ваши любимые формулы Excel, удобный инструмент, если вы использовали Excel для расчетов. Это также редактор LaTex, с легкостью вычисляющий даже самые трудоемкие уравнения.

Euclid также удобен для преобразования и утверждает, что может «преобразовать что угодно». Измерения, валюты, электричество — вы называете это, Евклид, вероятно, может преобразовать это. Одни только эти функции делают его отличным приложением для всех.

Лучший алгебраический калькулятор с шагами

Если вам нужно отличное приложение для алгебры для Mac, есть много отличных вариантов на выбор. Одним из наиболее популярных приложений является Numi, бесплатный калькулятор для вашего Mac, который использует ввод на естественном языке для решения уравнений.

Поскольку Numi использует естественный язык, он также позволяет вам определять свои собственные переменные. В приведенном ниже примере мы определили стоимость за единицу товара, а также скидку на заказ. Простое изменение суммы изменило окончательную стоимость; это отличный инструмент, если у вас есть определенная сумма, которую нужно потратить, и вам нужно точно знать, сколько товаров нужно заказать.

Вот как это делается:

  1. Откройте Numi на вашем Mac
  2. Введите имя переменной, а затем знак равенства
  3. Присвойте переменной значение
  4. Используйте предложение на естественном языке с именами ваших переменных, чтобы получить решение. Numi не требует изучения специальной языковой модели; просто напечатайте в обычном режиме и позвольте ему сделать все остальное

    Когда вам нужно выйти за рамки обычного языка и простого преобразования, PocketCAS, вероятно, станет шагом вперед по сравнению с Numi.

    Лучший блокнот-калькулятор

    Калькуляторы предназначены для компьютеров, но калькулятор Soulver снова делает их людьми. Вы можете вводить математические вычисления, функции, даты и преобразования единиц измерения, которые легко читать и форматировать, и приложение даст вам результаты, которые вы можете использовать.

    Это особенно полезно, если вы много работаете с различными комбинациями единиц измерения или часто используете одни и те же расчеты, но подставляете разные числа.

    Заключение

    Студенты-математики во всем мире могут извлечь пользу из упомянутых здесь замечательных приложений. Независимо от вашего уровня образования или потребностей, эти приложения помогут вам получить нужные результаты.

    Все четыре приложения — Numi, Euclid, PocketCAS и Soulver — доступны бесплатно в рамках семидневной пробной версии Setapp, самого полного в мире набора приложений для повышения производительности для вашего Mac. Наряду с этими четырьмя замечательными приложениями у вас также будет полный доступ ко всей библиотеке Setapp, включающей почти 200 других родных приложений для Mac.

Ответы по на экзамен по математике: Ответы на ЕГЭ по математике накануне экзамена

Ответы по ЕГЭ по математике: где искать

Меры, принимаемые Рособрнадзором для предотвращения слива заданий ЕГЭ в Сеть, судебные разбирательства и предупреждения об административной ответственности за копирование и распространение экзаменационных вариантов во время проведения испытания, видимо, приносят ожидаемые плоды. Поэтому с каждым годом вероятность найти в Интернете подлинные, и главное, правильные, ответы по ЕГЭ по математике накануне часа Ч стремится к нулю. А вот варианты прошлых лет, которые можно использовать для подготовки, отыскать нетрудно.

Содержание

  • Источники информации
  • Структура демоверсий
  • Что дальше
  • Видео по теме
  • Комментарии

Источники информации

Переходить на темную сторону в поисках ответов ЕГЭ по математике совсем необязательно. Стопроцентно легальные и абсолютно правильные ответы на все вопросы КИМов ЕГЭ профильного и базового уровней есть только в одном месте – в демоверсии на сайте ФИПИ. Конечно, единственный вариант даст всего лишь примерное представление о том, что будет на самом экзамене и как решать самые сложные задания, но это уже хотя бы что-то.

Чтобы получить доступ к ответам на ЕГЭ по математике не нужно даже проходить процедуру регистрации. Достаточно выполнить несколько простых действий:

  1. Установить на компьютере программное обеспечение для распаковки zip-архивов.
  2. Перейти на главную страницу официального сайта ФИПИ.
  3. В разделе меню «ЕГЭ и ГВЭ-11» выбрать вкладку «Демоверсии, спецификации и кодификаторы».
  4. Пройти по ссылке «Математика», выбрать нужный уровень.
  5. Скачать и распаковать архив.

В папке будет целый пул сопроводительных материалов, но ответы содержатся в конце демоверсии. Для сравнения можно ознакомиться еще и с вариантами предыдущих лет – они выложены, начиная с 2007 года. Структура ЕГЭ по математике не меняется только с 2016 года, так что слишком глубоко копать не стоит, ради интереса, разве что.

Зачем это делать? Хотя бы для того, чтобы иметь четкое представление, соответствует ли подлинным заданиям экзамена все то, что предлагается в Интернете и многочисленных печатных сборниках для подготовки в качестве заданий с ответами по ЕГЭ по математике.

Структура демоверсий

В базовой версии на одной позиция представлены две-три разновидности вопросов, которые могут попасться на ЕГЭ.

Ответы даны на все варианты, но в самом конце демоверсии.

Так что есть возможность прорешать официальный вариант КИМа.

В демоверсии КИМа усложненного профильного ЕГЭ по математике ответы представлены по-другому, так как задания с 13 по 19 требуют развернутых обоснований полученных результатов. Итоги, которые должны получиться после вычислений или выбора варианта в первых 12 вопросах, тоже сгруппированы в таблицу.

Ход решения и ответы самых трудных заданий изложены более обстоятельно.

Что дальше

Теперь, если в интернете попался сайт с тренировочными заданиями и ответами ЕГЭ по математике, как две капли воды похожими на официальные прототипы, смело можно браться за их решение.

Сайту с таким вариантом можно доверять.

Конечно же, актуальные для текущего года версии КИМов ЕГЭ по математике, тем более ответы к ним, – тайна за семью печатями вплоть до начала экзамена. Но, решая десятки, а возможно, и сотни вариантов заданий, скажем, на все виды неравенств, волей-неволей придется повторить все формулы, которые могут понадобиться для вычислений. Тогда одна измененная цифра или переставленные местами слова в условии задачи не собьют с толку на экзамене, и решение придет, если и не само собой, то без мучительных терзаний и лишней потери времени.

Правильные ответы по математике появились в сети за два дня до экзаменов

Комсомольская правда

Результаты поиска

ОбществоОбразованиеСКАНДАЛЕГЭ-2013

Александр МИЛКУС

3 июня 2013 15:15

Несмотря на заверения организаторов, что уж обязательный ЕГЭ по математике пройдет честно, «Комсомолка» получила множество свидетельств, прямо противоположных [видео]

«Комсомолка» получила множество свидетельств о «нечестной игре» во время сдачи ЕГЭФото: Иван ВИСЛОВ

После скандала на ЕГЭ по русскому, когда выяснилось — ответы на задания оказались в Интернете за несколько часов до начала испытания, — министр образования и науки Дмитрий Ливанов пообещал, что ЕГЭ по математике пройдет по другому регламенту. Что за регламент — не объяснялось. Но фраза звучала грозно, и можно было подумать, что организаторы экзамена так извернутся, что в этот раз на ЕГЭ и муха без санкции в аудиторию не пролетит.

Увы. Незадолго до времени «Ч» — то есть 3 июня в 10 утра по местному времени, когда выпускники должны были приступить к написанию экзаменационной работы, правильные ответы по математике уже благополучно лежали в Сети. Оставалось пронести их в аудиторию, найти свой вариант и проставить в нужных квадратиках аккуратные галочки черной ручкой.

— Совершенно ясно, что у неких лиц есть все варианты заданий ЕГЭ по всем предметам, — утверждает «Учитель года России-2007», преподаватель математики из Питера Дмитрий Гущин. — Причем эта база была у них на руках еще в апреле. На ней неплохо заработали. Технология такая. Сначала задания ЕГЭ продавали. Затем по мере приближения экзамена — когда деньги уже были получены и стало понятно, что больше уже ничего не «снять», варианты стали появляться на бесплатных сайтах и в соцсети.

— А откуда известно, что задания подлинные? Образовательные чиновники из Приморского края уверяют: задания, выложенные в Интернете, неправильные и не соответствуют тем вариантам, которые писал Дальний Восток.

— Я не говорю, что все задания были подлинные. Схема каждый раз перед экзаменом одна и та же. Сначала появляются фальшивки. И любой опытный учитель или эксперт, который в теме, их запросто вычисляет: задания в них компилируются из вопросов прошлых лет, тасуются… Потом появляется что-то более близкое к правде. А за пару дней до экзамена всплывают правильные варианты.

— Это точно?

— Абсолютно! По математике варианты появились вечером 1 июня. Дальше тоже все отработано. Задания растаскивают по Сети (их же много). Участники группы их быстро решают. И кто-то сводит их в табличку. Как правило, в первых таких табличках есть ошибки — кто-то неправильно решил задачу или не в ту графу занес правильный ответ. Их корректируют. В соцсетях идет бурное обсуждение. И в конце концов накануне экзамена — правильная табличка. Мало того — для каждого часового пояса.

— То есть некие добрые люди сливают не ответы, а сами задания?

— Да, а «коллективный разум» в Интернете их прорешивает.

— Трудно отличить вбрасываемые фальшивки от реальных заданий?

— Человек опытный это может сделать, посмотрев, каким шрифтом набраны задания, «плавает» ли колонтитул…

— «КП» на прошлой неделе опубликовала путь, который проделывают задания из Федерального института педагогических измерений (ФИПИ) через Федеральный центр тестирования в регионы. Как вы считаете, на каком этапе могли быть утечки?

— И на уровне ФИПИ, и в регионах. Не думайте, что своровали задания только один раз. Вероятно, источников, из которых ­ КИМы попадают в Сеть, несколько.

— Каков процент ребят, которые списывают на ЕГЭ?

— Думаю, 40-50 процентов. Может быть, не все задания, может, не всегда пользуются ответами из Сети, а банальными шпаргалками — но списывают. Есть пункты проведения экзамена, где организаторы такой практике явно потворствуют.

У Дмитрия Гущина нет сомнений — точно так же пройдут и остальные экзамены — с выбросами правильных ответов и списыванием.

Напомню: ребятам осталось сдать экзамены по выбору — иностранный язык, химия, литература, обществознание и география.

P. S. Рособрнадзор (служба, отвечающая за проведения ЕГЭ в стране) опубликовала обращение к журналистам с просьбой не накалять обстановку, не нервировать участников экзамена и не вселять в них ложные надежды — «специалисты пока говорят, что в Сети не обнаружено настоящих заданий предстоящего ЕГЭ по математике». Обращение пришло в редакцию «КП»31 мая…

ПИСЬМО В «КП»

«Каюсь, я решала задания для выпускников»

«27 мая 2013 года примерно в 11.30 утра, то есть прямо во время проведения ЕГЭ по русскому языку, ко мне попали реальные КИМы с экзамена. Некая женщина, предположительно из Дагестана, известная мне только по переписке в социальной сети, переслала мне их для того, чтобы я их решила. Я это сделала, переслав ей готовые решения. То же самое выполняли еще несколько человек, так как эта женщина разместила объявления повсюду в социальной сети.

На пересланных мне листках есть индивидуальный штрих-код и номер, а также две фамилии сдававших экзамен.

Все это, вместе взятое, означает, что в Дагестане (точное место мне неизвестно) организовано массовое мошенничество прямо при проведении экзамена. КИМы пересылаются, очевидно, по телефону прямо во время ЕГЭ, решаются посторонними лицами, а потом пересылаются в экзаменационную аудиторию, где сдающие их списывают.

Высылаю вам пересланные мне листы ­КИМов с штрих-кодом и фамилиями сдающих.

С уважением

Марина Васильевна Петрова».

Мы дозвонились главному специалисту Центра обработки информации Дагестана Аиде Сайдуллаевой и попросили прокомментировать ситуацию:

— Они (школьники. — Ред.) каким-то образом их проносят. Есть акты от наблюдателей — у кого обнаружили телефон, кто нарушал, кто вставал, выходил. После каждого экзамена акты о нарушениях мы отправляем в Министерство образования. Собирается государственная экзаменационная комиссия. Они решают, кому аннулировать экзамены.

По словам министра образования и науки Дагестана Артура Далгатова, в министерство поступил 61 акт о нарушении процедуры проведения ЕГЭ по русскому языку.

ЦИФРА

ЕГЭ по математике в этом году писали 816 тысяч человек, из них 750 тысяч выпускников текущего года.

Скандальный ЕГЭ: как российские школьники сдавали выпускной экзамен

КП-ТВ

ЧИТАЙТЕ ТАКЖЕ

Все выпускники, получившие 100 баллов на ЕГЭ в Ставрополье, — дети чиновников — В регионе — аномальное количество высокобалльников по ряду предметов, — отметил полпред президента в СКФО Александр Хлопонин на недавнем совещании, посвященном проведению ЕГЭ в округе. — Например, из восьми стобалльных работ по русскому языку в России семь — ставропольских. Требует внимательного изучения и тот факт, что среди досрочников, предоставлявших медицинские справки, — в основном дети местных чиновников (далее).

Все выпускники, получившие 100 баллов на ЕГЭ в Ставрополье, — дети чиновников — В регионе — аномальное количество высокобалльников по ряду предметов, — отметил полпред президента в СКФО Александр Хлопонин на недавнем совещании, посвященном проведению ЕГЭ в округе. — Например, из восьми стобалльных работ по русскому языку в России семь — ставропольских. Требует внимательного изучения и тот факт, что среди досрочников, предоставлявших медицинские справки, — в основном дети местных чиновников (далее).

— В регионе — аномальное количество высокобалльников по ряду предметов, — отметил полпред президента в СКФО Александр Хлопонин на недавнем совещании, посвященном проведению ЕГЭ в округе. — Например, из восьми стобалльных работ по русскому языку в России семь — ставропольских. Требует внимательного изучения и тот факт, что среди досрочников, предоставлявших медицинские справки, — в основном дети местных чиновников (далее).

Читайте также

Возрастная категория сайта 18+

Сетевое издание (сайт) зарегистрировано Роскомнадзором, свидетельство Эл № ФС77-80505 от 15 марта 2021 г.

ГЛАВНЫЙ РЕДАКТОР — НОСОВА ОЛЕСЯ ВЯЧЕСЛАВОВНА.

ШЕФ-РЕДАКТОР САЙТА — КАНСКИЙ ВИКТОР ФЕДОРОВИЧ.

АВТОР СОВРЕМЕННОЙ ВЕРСИИ ИЗДАНИЯ — СУНГОРКИН ВЛАДИМИР НИКОЛАЕВИЧ.

Сообщения и комментарии читателей сайта размещаются без предварительного редактирования. Редакция оставляет за собой право удалить их с сайта или отредактировать, если указанные сообщения и комментарии являются злоупотреблением свободой массовой информации или нарушением иных требований закона.

АО «ИД «Комсомольская правда». ИНН: 7714037217 ОГРН: 1027739295781 127015, Москва, Новодмитровская д. 2Б, Тел. +7 (495) 777-02-82.

Исключительные права на материалы, размещённые на интернет-сайте www.kp.ru, в соответствии с законодательством Российской Федерации об охране результатов интеллектуальной деятельности принадлежат АО «Издательский дом «Комсомольская правда», и не подлежат использованию другими лицами в какой бы то ни было форме без письменного разрешения правообладателя.

Приобретение авторских прав и связь с редакцией: [email protected]

Вопросы по математике с ответами

Представлен набор математических вопросов с несколькими вариантами ответов. Ответы предоставлены и расположены в нижней части страницы. Вопросы были разработаны, чтобы проверить глубокое понимание математических концепций. Также даны подробные пояснения и решения этих вопросов.

Домашняя страница

Вопросы 1

Если Log x (1/8) = — 3/2, то x равно
А. — 4
Б. 4
С. 1/4
Д. 10

Вопросы 2

20 % от 2 равно
А. 20
Б. 4
С. 0,4
Д. 0,04

Вопросы 3

Если Log 4 (x) = 12, то log 2 (x / 4) равен
А. 11
Б. 48
С. -12
д. 22

Вопросы 4

Население страны увеличивалось в среднем на 2% в год с 2000 по 2003 год. Если население этой страны на 31 декабря 2003 г. тысяча была бы
А. 1 846 000
Б. 1 852 000
К. 1 000 000
Д. 1 500 000

Вопросы 5

f — квадратичная функция, график которой представляет собой параболу, открывающуюся вверх и имеющую вершину на оси x. График новой функции g, определяемой соотношением g(x) = 2 — f(x — 5), имеет диапазон, определяемый интервалом
А. [-5, + бесконечность)
Б. [ 2 , + бесконечность)
С. (- бесконечность, 2]
д. (- бесконечность, 0]

Вопросы 6

f — функция такая, что f(x) < 0. График новой функции g, определяемой формулой g(x) = | е (х) | является отражением графика f
А. по оси Y
Б. по оси х
C. на линии y = x
D. на линии y = — x

Вопросы 7

Если график y = f(x) преобразовать в график 2y — 6 = — 4 f(x — 3), точка (a, b) на графике y = f(x) станет точкой (A, B) на графике 2y — 6 = — 4 f(x — 3), где A и B задаются формулой
А. А = а — 3, В = б
Б. А = а — 3, В = б
С. А = а + 3, В = -2 б
Д. А = а + 3, В = -2 б +3

Вопросы 8

Когда парабола, представленная уравнением y — 2x 2 = 8 x + 5 переносится на 3 единицы влево и на 2 единицы вверх, новая парабола имеет вершину в
А. (-5 , -1)
Б. (-5 , -5)
С. (-1 , -3)
Д. (-2 , -3)

Вопросы 9

Графики двух линейных уравнений a x + b y = c и b x — a y = c, где ни один из коэффициентов a, b, c не равен нулю,
А. параллельны
Б. пересекаются в точке (0,0)
С. пересекаются в двух точках
Д. перпендикулярно

Вопросы 10

Графики двух уравнений y = a x 2 + b x + c и y = A x 2 + B x + C, такие, что a и A имеют разные знаки и что величины b 2 — 4 a c и B 2 — 4 A C равны отрицательный,
А. пересекаются в двух точках
Б. пересекаются в одной точке
С. не пересекаются
D. ничего из вышеперечисленного

Вопросы 11

Для x больше или равного нулю и меньше или равного 2 π, sin x и cos x оба уменьшаются на интервалах
А. (0 , π/2)
Б. (π/2, π)
С. (π, 3 π/2)
Д. (3 π / 2 , 2 π)

Вопросы 12

Три решения уравнения f (x) = 0 равны -2, 0 и 3. Следовательно, три решения уравнения f (x — 2) = 0 равны
А. — 4, -2 и 1
Б.-2, 0 и 3
С. 4, 2 и 5
Д. 0, 2 и 5

Вопросы 13

Три решения уравнения f (x) = 0 равны — 4, 8 и 11. Следовательно, три решения уравнения f (2 x) = 0 равны
А. — 2, 4 и 11/2
Б. — 8, 16 и 22
С. — 4, 8 и 11
Д. 2, 19/2 и 7/2

Вопросы 14

Школьный комитет состоит из 2 учителей и 4 учеников. Количество различных комитетов, которые могут быть сформированы из 5 преподавателей и 10 студентов, равно
А. 10
Б. 15
С. 2100
д. 8

Вопросы 15

Пять разных книг (A, B, C, D и E) должны быть расставлены на полке. Книги C и D должны располагаться первыми и вторыми, начиная справа от полки. Количество различных порядков, в которых могут быть расположены книги A, B и E, равно
А. 5!
Б. 3!
С. 2!
Д. 3! * 2!

Вопросы 16

Среднее значение набора данных равно 10, а его стандартное отклонение равно 1. Если мы добавим 5 к каждому значению данных, то среднее значение и стандартное отклонение станут
A. среднее значение = 15, стандартное отклонение = 6
B. среднее = 10, стандартное отклонение = 6
C. среднее значение = 15, стандартное отклонение = 1
D. среднее = 10, стандартное отклонение = 1

Вопросы 17

Экзаменационные баллы всех 500 студентов были записаны, и было установлено, что эти баллы были распределены нормально. Если балл Джейн на 0,8 стандартного отклонения выше среднего, то сколько с точностью до единицы учащиеся набрали больше баллов, чем Джейн?
А. 394
Б. 250
С. 400
Д. 106

Вопросы 18

Если f(x) — нечетная функция, то | е (х) | является
А. нечетная функция
B. четная функция
C. ни нечетный, ни четный
Д. четные и нечетные

Вопросы 19

Период | грех (3x) | является
А. 2 π
Б. 2 π/3
С. №/3
Д. 3 π

Вопросы 20

Когда металлический шарикоподшипник помещается внутрь цилиндрического сосуда радиусом 2 см, высота воды внутри сосуда увеличивается на 0,6 см. Радиус шарикоподшипника с точностью до десятых долей сантиметра
А. 1 см
Б. 1,2 см
С. 2 см
Д. 0,6 см

Вопросы 21

Период 2 sin x cos x равен
А. 4 π 2
Б. 2 π
С. 4 π
Д. π

Вопросы 22

Вероятность того, что электронное устройство, произведенное компанией, не будет работать должным образом, равна 0,1. Если куплено 10 устройств, то вероятность с точностью до тысячной того, что 7 устройств будут исправно работать, равна
А. 0,057
Б. 0,478
С. 0,001
Д. 0

Ответы на вышеуказанные вопросы

1б, 2в, 3г, 4а, 5в, 6б, 7г, 8а, 9г, 10в
11б, 12г, 13а, 14в, 15б, 16в, 17д, 18б, 19в, 20б
21д, 22а.

Подробнее Ссылки и ссылки на вопросы и задачи по математике

вопросы и задачи по математике с подробными решениями.

404—Страница не найдена

Перейти к основному содержанию

НУЖНА ИНФОРМАЦИЯ

Найти школу

Чат

Свяжитесь с нами

Зарегистрироваться Родительский портал

Как зарегистрироваться

Требуется дополнительная информация

К сожалению, запрошенная вами страница не найдена. Пожалуйста, проверьте правильность написания URL-адреса и, если у вас по-прежнему возникают проблемы, попробуйте воспользоваться поиском на этой странице или по одной из ссылок ниже: 

  • K12 Home
  • Записаться в школу
  • Найти школу
  • Посетить мероприятие

* Обязательно.

* Имя *

* Фамилия *

* Электронная почта *

* Телефон *

Состояние *

* Выберите штат. .. МеждународныйАлабамаАляскаАризонаАрканзасКалифорнияКолорадоКоннектикутОкруг КолумбияДелавэрФлоридаГрузияГавайиАйдахоИллинойсИндианаАйоваКанзасКентуккиЛуизианаМэнМэрилендМассачусетсМичиганМиннесотаМиссисипиМиссуриМонтанаНебраскаНевадаНовый H АмпширНью-ДжерсиНью-МексикоНью-ЙоркСеверная КаролинаСеверная ДакотаОгайоОклахомаОрегонПенсильванияРод-АйлендЮжная КаролинаЮжная ДакотаТеннессиТехасЮтаВермонтВирджинияВашингтонЗападная ВирджинияВисконсинВайоминг

* Почтовый индекс *

* Какие марки вас интересуют? Выберите все подходящие варианты.

Да, я заинтересован в подготовке к карьере

У меня есть вопросы по следующим темам.

Виртуальное/онлайн-обучение

Участие учителя

Добавить личное примечание

Начать писать

Мой ребенок/дети уже зачислены в школу K12.

Предоставляя эту информацию, вы соглашаетесь получать звонки/текстовые сообщения от K12 или представителя школы или устройства, которое будет автоматически набирать предоставленный номер. Могут применяться тарифы на передачу сообщений и данных.

Требуется дополнительная информация *Обязательные поля

* Имя *

* Фамилия *

* Адрес электронной почты *

* Номер телефона *

Состояние *

* Выберите штат… МеждународныйАлабамаАляскаАризонаАрканзасКалифорнияКолорадоКоннектикутОкруг КолумбияДелавэрФлоридаГрузияГавайиАйдахоИллинойсИндианаАйоваКанзасКентуккиЛуизианаМэнМэрилендМассачусетсМичиганМиннесотаМиссисипиМиссуриМонтанаНебраскаНевадаНовый H АмпширНью-ДжерсиНью-МексикоНью-ЙоркСеверная КаролинаСеверная ДакотаОгайоОклахомаОрегонПенсильванияРод-АйлендЮжная КаролинаЮжная ДакотаТеннессиТехасЮтаВермонтВирджинияВашингтонЗападная ВирджинияВисконсинВайоминг

* Почтовый индекс *

* Какие марки вас интересуют?

Выберите все подходящие варианты.

Да, я заинтересован в подготовке к карьере

У меня есть вопросы по следующим темам.

Виртуальное/онлайн-обучение

Участие учителя

Добавить личную заметку

Мой ребенок/дети уже зачислены в школу K12

Предоставляя эту информацию, вы соглашаетесь получать звонки/текстовые сообщения от K12 или представителя школы или устройства, которое будет автоматически набирать предоставленный номер. Могут применяться тарифы на передачу сообщений и данных.

Мы хотим услышать от вас. Отправьте электронное письмо или позвоните нам по телефону 866.968.7512. *Обязательные поля

* Имя *

* Фамилия *

* Адрес электронной почты *

* Номер телефона *

* Сообщение *

Мой ребенок/дети уже зачислены в школу K12

Состояние *

* Выберите штат. .. МеждународныйАлабамаАляскаАризонаАрканзасКалифорнияКолорадоКоннектикутОкруг КолумбияДелавэрФлоридаГрузияГавайиАйдахоИллинойсИндианаАйоваКанзасКентуккиЛуизианаМэнМэрилендМассачусетсМичиганМиннесотаМиссисипиМиссуриМонтанаНебраскаНевадаНовый H АмпширНью-ДжерсиНью-МексикоНью-ЙоркСеверная КаролинаСеверная ДакотаОгайоОклахомаОрегонПенсильванияРод-АйлендЮжная КаролинаЮжная ДакотаТеннессиТехасЮтаВермонтВирджинияВашингтонЗападная ВирджинияВисконсинВайоминг

* Почтовый индекс *

* Какие марки вас интересуют?

Выберите все подходящие варианты.

Да, я заинтересован в подготовке к карьере

У меня есть вопросы по следующим темам.

Виртуальное/онлайн-обучение

Участие учителя

Предоставляя эту информацию, вы соглашаетесь получать звонки/текстовые сообщения от K12 или представителя школы или устройства, которое будет автоматически набирать предоставленный номер.

Решебник сборник задач по математике для втузов ефимов демидович: Ефимов А.В., Демидович Б.П. Сборник задач по математике для втузов. Часть 1-2 — Книги — ГУМФ — Каталог файлов

Демидович решебник pdf

скачать антидемидович бесплатно

 



 


※ Download: Демидович решебник pdf

 


 

Оборудование: таблицы, рисунки, фотографии, запись песни «Праздник приближается», карточки с заданиями. Разумовская 2014 Русский язык, 5 класс Т. Может у кого-то есть такие учебники для 7 класса на укр.


 

Его часто приглашали в состав Оргкомитетов как научных конференций, так и школьных олимпиад. Интегрирование тригонометрических функций 192 5.


 

Решебник к книге Демидовича задач и упражнений по математическому Предел функции. №471- №480 — Геометрический смысл производной 117 § 4.


 

Координаты центра тяжести Каждой задаче отведен pdf решебниик, содержащий общую постановку задачи, план ее решения с необходимыми теоретическими пояснениями и решение конкретного. Ко всем задачам даны ответы дн. Сборник задач по математике для втузов — Ефимов Pdv Теория статистики с основами теории вероятностей. В учебнике предложен новый п. Часть 3 Your browser does not seem to support iframes. И в мещерский решебник по теоретической Готовое домашнее задание по алгебре и математике. Производная функции, заданной в неявном виде 3. Наряду с минимальными теоретическими сведениями в нем содержится более семисот детально разобранных примеров. Решебник к сборнику задач по математическому анализу — Бермана Онлайн решебник по решебник за 10 класс, Всё задачи направлены на развитие логического мышления. Гдз по истории учебник россия 11 класс автор а. Кагановича, а с февраля 1936 г. Книга для демидович, кто испытывает проблемы с пониманием математического анализа, но стремится понять. Дифференциальное исчисление функций нескольких 318 переменных 1. Изложены алгоритмы и примеры решения задач статики, кинематики решебник динамики из курса теоретической механики, изучаемого в технических вузах. Your browser does not seem to демидович iframes. Демидович решебник скачать pdf Построение графиков функций по характерным точкам Задачник предназначен для студентов втузов, а такж. Вычисление объемов Сборник задач и упражнений по математическому анализу. Признаки сходимости знакопеременных рядов Мы не показываем назойливой pdf непристойного содержания, а небольшой доход от простого показа рекламных блоков помогает в развитии сайта. Решебник к сборнику задач по математическому анализу Бермана Г.

Ejercicio No7 Libro Demidovich «Problemas y ejercicios de Análisis Matemático»

imgur.com

Imgur: The magic of the Internet

Category Entertainment News News & Media Journey

Гдз сборник задач по матиматике для средних специальных :: retucaret

12. 11.2016 02:24

 

 

 

 

 

 

 

 

 

 

 

 

 

 

 

 

 

 

 

 

Проведения письменного экзаме по математике в девятых классах общеобразовательных школ СССР. Ефимов А. В., Демидович Б. П. Сборник задач по математике для втузов. Сборник задач и заданий для тематического оценивания для русских школ. Заданиями на задания из специального сборника по Математике для 5 класса, преподаватели, производящие процесс преподавания предмета в пятых классах средних. Проблема обучения математике в средних специальных учебных. Богомолов Н. В. Сборник задач по математике.— 496 с. Содержание книги соответствует программе по математике для средних специальных учебных заведений на базе классов средней школы. Книга предназначена для учащихся средних специальных учебных заведений. Картина сСборник задач по математике для техникумов соловейчик. Решебник с ГДЗ. Ко всем задачам даны ответы, а ко многимуказания и решения. Название: Сборник.

Анализа, гдз сборник задач по математике богомолов спо. Сборник задач по математике для техникумов. Сборник задач по математике для поступающих в вузы под редакцией М. И. Сканави. Математика. Учебники, ГДЗ, решебники, ЕГЭ, ГИА, экзамены, книги. Каждая глава. Примеров и задач. Для студентов средних специальных учебных заведений. Сборник задач по элементарной математике для техникумов, Рудник А. ФЕНИКС Филимонова Математика для сред.спец.учеб.завед.:учеб.пособ.дп Филимонова Е. В. Физика и математика. Для преподавателей и студентов средних специальных учебных заведений. Решебник Сборник задач по математике Богомолов Н В — : 7:54. Учебное пособие для средних специальных учебных заведений. Сборник индивидуальных заданий по высшей математике. Часть 1. Сборники задач, заданий, самоучители. Часть 4. Содержит задачи по специальным курсам математики: теории. Для студентов средних специальных учебных заведений. Гдз по сборнику для.

Задач по математике. Учебник физики 9 класс Автор: Перышкин Для студентов средних специальных учебных заведений. Сборник содержит задачи по основным разделам курса физики, изучаемого в. В сборник включены задачи по всем разделам курса математики, изучаемого в средних специальных учебных заведениях. Вы здесь: Главная сайта ГДЗ Сборник задач. Он может быть полезно аспирантам и научным указанных специальных. Сборник задач по математике для поступающих во ВТУЗы. Физика для средних специальных учебных заведений. Сборник Задач По Математике Богомолов Решебник Скачать. Математика. Физика для средних специальных учебных заведений, Жданов. Краткие теоретические сведения, снабженные большим количеством разобранных примеров, позволяют использовать сборник для всех видов обучения. ГДЗ по математике. Задачники и решебники. Книга: Математика: Учебное пособие для средних специальных учебных.

Скачать Сборник задач по математике. Учебники, ГДЗ, решебники, ЕГЭ, ГИА, экзамены, книги. Книга предназначена для дополнительного чтения по физике в средних специальных учебных. Сборник задач по математике богомолов гдз Гдз по математике богомолов н. Название:. Математика для техникумов на базе среднего образования О. Н. Афанасьева, Я. С. Бродский, А. Л. Справочники, энциклопедии. Пособие содержит задачи по всем разделам курса математики, изучаемого в средних специальных учебных заведениях. Решебник ГДЗ по учебнику Сборник заданий для проведения письменного экзамена по математике курс А, алгебре и началам анализа курс В за курс средней школы: 11 класс. Высшая школа. Математика: учебное пособие для средних специальных учебных заведений. В учебнике рассмотрены основные разделы математики, охватываемые действующими программами для техникумов: алгебра, начала.

 

Вместе с Гдз сборник задач по матиматике для средних специальных часто ищут

 

практические занятия по математике богомолов решебник гдз.

гдз богомолов сборник задач по математике.

сборник задач по математике богомолов pdf решебник.

практические занятия по математике богомолов гдз высшая школа.

Сборник задач по высшей математике лунгу 1 курс решебник: Высшая математика Лунгу К.Н. 1, 2 курс

Сборник задач по высшей математике с контрольными работами1 курс (Дмитрий Письменный)

1 349 ₽

+ до 202 бонусов

Купить

Цена на сайте может отличаться от цены в магазинах сети. Внешний вид книги может отличаться от изображения на сайте.

В наличии

10

Цена на сайте может отличаться от цены в магазинах сети. Внешний вид книги может отличаться от изображения на сайте.

Сборник содержит свыше трех с половиной тысяч задач по высшей математике. Ко всем разделам книги даны необходимые теоретические пояснения. Детально разобраны типовые задачи, приведено изрядное количество разнообразных заданий различных уровней сложности для самостоятельного решения. Наличие в сборнике контрольных работ, устных задач и «качественных» вопросов позволит студенту подготовиться к экзаменационной сессии. Книга охватывает материал по линейной алгебре, аналитической геометрии, основам математического анализа и комплексным числам. Книга будет полезна студентам младших курсов и преподавателям вузов.

Описание

Характеристики

Сборник содержит свыше трех с половиной тысяч задач по высшей математике. Ко всем разделам книги даны необходимые теоретические пояснения. Детально разобраны типовые задачи, приведено изрядное количество разнообразных заданий различных уровней сложности для самостоятельного решения. Наличие в сборнике контрольных работ, устных задач и «качественных» вопросов позволит студенту подготовиться к экзаменационной сессии. Книга охватывает материал по линейной алгебре, аналитической геометрии, основам математического анализа и комплексным числам. Книга будет полезна студентам младших курсов и преподавателям вузов.

Айрис-пресс

Как получить бонусы за отзыв о товаре

1

Сделайте заказ в интернет-магазине

2

Напишите развёрнутый отзыв от 300 символов только на то, что вы купили

3

Дождитесь, пока отзыв опубликуют.

Если он окажется среди первых десяти, вы получите 30 бонусов на Карту Любимого Покупателя. Можно писать неограниченное количество отзывов к разным покупкам – мы начислим бонусы за каждый, опубликованный в первой десятке.

Правила начисления бонусов

Если он окажется среди первых десяти, вы получите 30 бонусов на Карту Любимого Покупателя. Можно писать неограниченное количество отзывов к разным покупкам – мы начислим бонусы за каждый, опубликованный в первой десятке.

Правила начисления бонусов

Оценка товара

Плюсы

Подача материала , качество бумаги , гармоничное сочетание теории и практических заданий.

Минусы

Отсутствуют

Книга «Сборник задач по высшей математике с контрольными работами1 курс» есть в наличии в интернет-магазине «Читай-город» по привлекательной цене. Если вы находитесь в Москве, Санкт-Петербурге, Нижнем Новгороде, Казани, Екатеринбурге, Ростове-на-Дону или любом другом регионе России, вы можете оформить заказ на книгу Дмитрий Письменный «Сборник задач по высшей математике с контрольными работами1 курс» и выбрать удобный способ его получения: самовывоз, доставка курьером или отправка почтой. Чтобы покупать книги вам было ещё приятнее, мы регулярно проводим акции и конкурсы.

Издательство ФИЗМАТЛИТ — физико-математическая и техническая литература

Издательство ФИЗМАТЛИТ — физико-математическая и техническая литература

  • Абрамовский В.А., Архипов Г.И., Найда О.Н. «Дифференциальное и интегральное исчисление функции одной переменной»
  • Абрамовский В.А., Белов В.Н., Найда О.Н. «Аналитическая геометрия и линейная алгебра. Ряды и интегралы, зависящие от параметра. Ряды и интегралы Фурье»
  • Акивис М. А., Гольдберг В.В. «Тензорное исчисление»
  • Алексеев В.М., Галеев Э.М., Тихомиров В.М. «Сборник задач по оптимизации. Теория. Примеры. Задачи»
  • Артамонов В.А., Бахтурин Ю.А., Винберг Э.Б., Голод Е.С. и др. «Сборник задач по алгебре» Под ред. А.И. Кострикина
  • Арутюнов А.В. «Лекции по выпуклому и многозначному анализу»
  • Афанасьев В.И., Зимина О.В., Кириллов А.И., Петрушко И.М. и др. «Решебник. Высшая математика. Специальные разделы» Под ред. А.И. Кириллова
  • Баврин И.И. «Краткий курс высшей математики для химико-биологических и медицинских специальностей»
  • Беклемишев Д.В. «Решение задач »
  • Бесов О. В. «Лекции по математическому анализу»
  • Битюков Ю.И., Ильина А.Н., Мартюшова Я.Г. «Математический анализ» Под. ред. Кибзуна
  • Бочаров П.П., Печинкин А.В. «Теория вероятностей. Математическая статистика»
  • Бугров Я.С., Никольский С.М. «Сборник задач по высшей математике»
  • Вагин В.Н. «Знания и убеждения в интеллектуальном анализе данных»
  • Вагин В.Н., Головина Е.Ю., Загорянская А.А., Фомина М.В. «Достоверный и правдоподобный вывод в интеллектуальных системах» Под ред. В.Н. Вагина, Д.А. Поспелова
  • Васильева А.Б., Тихонов Н.А. «Интегральные уравнения»
  • Владимиров В. С., Михайлов В.П., Михайлова Т.В., Шабунин М.И. «Сборник задач по уравнениям математической физики»
  • Волковыский Л.И., Лунц Г.Л., Араманович И.Г. «Сборник задач по теории функций комплексного переменного»
  • Гаврилов Г.П., Сапоженко А.А. «Задачи и упражнения по дискретной математике»
  • Гантмахер Ф.Р. «Теория матриц»
  • Геворкян П.С. «Высшая математика. Основы математического анализа»
  • Геворкян П.С. «Высшая математика. Интегралы, ряды, ТФКП, дифференциальные уравнения»
  • Геворкян П.С. «Высшая математика. Линейная алгебра и аналитическая геометрия»
  • Геворкян П. С., Потемкин А.В., Эйсымонт И.М. «Теория вероятностей и математическая статистика» Под ред. П. С. Геворкяна
  • Герасимчук В.С., Васильченко Г.С., Кравцов В.И. «Курс классической математики в примерах и задачах»
  • Герасимчук В.С., Васильченко Г.С., Кравцов В.И. «Курс классической математики в примерах и задачах»
  • Герасимчук В.С., Васильченко Г.С., Кравцов В.И. «Курс классической математики в примерах и задачах»
  • Гладков Л.А., Курейчик В.В., Курейчик В.М. «Дискретная математика» Под ред. В.М. Курейчика
  • Головешкин В.А., Ульянов М.В. «Теория рекурсии для программистов»
  • Гордин В.А. «Математика, компьютер, прогноз погоды и другие сценарии математической физики»
  • Горелик Г. С. «Колебания и волны»
  • Дураков Б.К. «Краткий курс высшей алгебры»
  • Егоров А.И. «Классификация решений обыкновенных дифференциальных уравнений первого порядка»
  • Ершов Ю.Л., Палютин Е.А. «Математическая логика»
  • Ефимов Н.В. «Высшая геометрия»
  • Ефимов Н.В. «Квадратичные формы и матрицы»
  • Ефимов Н.В. «Краткий курс аналитической геометрии»
  • Ефимов Н.В., Розендорн Э.Р. «Линейная алгебра и многомерная геометрия»
  • Закревский А.Д., Поттосин Ю.В., Черемисинова Л.Д. «Логические основы проектирования дискретных устройств»
  • Зельдович Я. Б. «Высшая математика для начинающих и ее приложения к физике» Под общ. ред. С.С. Герштейна
  • Зимина О.В., Кириллов А.И., Сальникова Т.А. «Решебник. Высшая математика» Под ред. А.И. Кириллова
  • Злобина С.В., Посицельская Л.Н. «Математический анализ в задачах и упражнениях»
  • Измаилов А.Ф. «Чувствительность в оптимизации»
  • Измаилов А.Ф., Солодов М.В. «Численные методы оптимизации»
  • Ильин В.А., Позняк Э.Г. «Линейная алгебра» Под ред. В.А. Ильина
  • Ильин В.А., Позняк Э.Г. «Основы математического анализа» Под ред. В.А. Ильина
  • Ильин В.А., Позняк Э.Г. «Основы математического анализа» Под ред. В.А. Ильина
  • Кадомцев С.Б. «Аналитическая геометрия и линейная алгебра»
  • Кибзун А.И., Горяинова Е.Р., Наумов А.В. «Теория вероятностей и математическая статистика. Базовый курс с примерами и задачами»
  • Кирсанов М.Н. «Задачи по теоретической механике с решениями в Maple 11»
  • Колмогоров А.Н., Фомин С.В. «Элементы теории функций и функционального анализа»
  • Кострикин А.И. «Введение в алгебру»
  • Крянев А.В., Лукин Г.В. «Математические методы обработки неопределенных данных»
  • Крянев А.В., Лукин Г.В., Удумян Д.К. «Метрический анализ и обработка данных»
  • Кудрявцев В. Б., Гасанов Э.Э., Долотова О.А., Погосян Г.Р. «Теория тестирования логических устройств» Под ред. В.А. Садовничего
  • Кудрявцев Л.Д. «Краткий курс математического анализа»
  • Кудрявцев Л.Д. «Краткий курс математического анализа»
  • Кудрявцев Л.Д., Кутасов А.Д., Чехлов В.И., Шабунин М.И. «Сборник задач по математическому анализу» Под ред. Л.Д. Кудрявцева
  • Кудрявцев Л.Д., Кутасов А.Д., Чехлов В.И., Шабунин М.И. «Сборник задач по математическому анализу» Под ред. Л.Д. Кудрявцева
  • Кудрявцев Л.Д., Кутасов А.Д., Чехлов В.И., Шабунин М.И. «Сборник задач по математическому анализу» Под ред. Л.Д. Кудрявцева
  • Кузнецова Т.А., Мироненко Е. С., Розанова С.А., Сирота А.И. и др. «Высшая математика» Под ред. С. А. Розановой
  • Лавров И.А., Максимова Л.Л. «Задачи по теории множеств, математической логике и теории алгоритмов»
  • Лебедев В.И. «Функциональный анализ и вычислительная математика»
  • Лизунова Н.А., Шкроба С.П. «Матрицы и системы линейных уравнений»
  • Лунгу К.Н. «Линейное программирование. Руководство к решению задач»
  • Лунгу К.Н., Макаров Е.В. «Задачи по математике»
  • Лунгу К.Н., Макаров Е.В. «Основные методы решения задач по элементарной математике»
  • Лунгу К.Н., Макаров Е.В. «Высшая математика. Руководство к решению задач»
  • Лунгу К.Н., Макаров Е.В. «Высшая математика. Руководство к решению задач»
  • Макоха А.Н., Сахнюк П.А., Червяков Н.И. «Дискретная математика»
  • Марченков С.С. «Конечные автоматы»
  • Марченков С.С. «Функциональные уравнения дискретной математики»
  • Марченков С.С. «Основы теории булевых функций»
  • Миллер Б.М., Панков А.Р. «Теория случайных процессов в примерах и задачах»
  • Михеев В.И., Павлюченко Ю.В. «Высшая математика, краткий курс»
  • Мищенко А.С., Фоменко А.Т. «Краткий курс дифференциальной геометрии и топологии»
  • Мышкис А. Д. «Прикладная математика для инженеров. Специальные курсы»
  • Ольшанский М.А. «Лекции и упражнения по многосеточным методам»
  • Петров И.Б. «Вычислительная математика для физиков»
  • Печинкин А.В., Разумчик Р.В. «Системы массового обслуживания в дискретном времени»
  • Посицельская Л.Н. «Теория функций комплексной переменной в задачах и упражнениях»
  • Прудников В.В., Вакилов А.Н., Прудников П.В. «Фазовые переходы и методы их компьютерного моделирования»
  • Пугачев В.С. «Теория вероятностей и математическая статистика»
  • Ракитин В.И. «Руководство по методам вычислений и приложения MATHCAD»
  • Редькин Н. П. «Дискретная математика»
  • Розендорн Э.Р. «Задачи по дифференциальной геометрии»
  • Розендорн Э.Р., Соболева Е.С., Фатеева Г.М. «Уравнения с частными производными» Под ред. Э.Р. Розендорна
  • Рябенький В.С. «Введение в вычислительную математику»
  • Сабитов К.Б. «Уравнения математической физики»
  • Семаков С.Л. «Элементы теории вероятностей и случайных процессов»
  • Сидельников В.М. «Теория кодирования»
  • Сизый С.В. «Лекции по теории чисел»
  • Сизый С.В. «Лекции по дифференциальной геометрии»
  • Сизый С. В. «Математические задачи»
  • Сизый С.В. «Лекции по аналитической геометрии»
  • Сухарев А.Г., Тимохов А.В., Федоров В.В. «Курс методов оптимизации»
  • Тихонов А.Н., Васильева А.Б., Свешников А.Г. «Дифференциальные уравнения»
  • Треногин В.А. «Обыкновенные дифференциальные уравнения»
  • Треногин В.А., Недосекина И.С. «Уравнения в частных производных»
  • Треногин В.А., Писаревский Б.М., Соболева T.С. «Задачи и упражнения по функциональному анализу»
  • Турчак Л.И., Плотников П.В. «Основы численных методов»
  • Тыртышников Е. Е. «Основы алгебры»
  • Ульянов П.Л., Бахвалов А.Н., Дьяченко М.И., Казарян К.С. и др. «Действительный анализ в задачах»
  • Успенский В.А., Верещагин Н.К., Плиско В.Е. «Вводный курс математической логики»
  • Федорчук В.В., Филиппов В.В. «Общая топология. Основные конструкции»
  • Федоткин М.А. «Модели в теории вероятностей»
  • Федоткин М.А. «Лекции по анализу случайных явлений»
  • Филиппов В.А. «Основы геометрии поверхностей оболочек пространственных конструкций»
  • Фихтенгольц Г.М. «Курс дифференциального и интегрального исчисления» Пред. и прим. А.А. Флоринского
  • Шафаревич И.

Интересные задачки по математике: занимательные текстовые задачки, примеры и другие задания с ответами и решением

Сложные задачи с цифрами, задания на логику, математику — 27 сентября 2021

Все новости

В центре Перми отключили светофоры на двух перекрёстках

«Смерти моей ждут?»: пенсионерку вогнали в миллионные долги по чужим кредитам

В Пермском крае на острове нашли истощенного медвежонка. Детёныша спасли и назвали Машенькой

Силовики задержали первого замглавы Индустриального района Перми Дмитрия Безденежных

В России хотят ввести шестидневную рабочую неделю

Садись, два! Попробуйте ответить на реальные вопросы из ЕГЭ по литературе

В Прикамье на пожаре погибли двое детей и их бабушка с дедушкой

+31 °С и грозы: прогноз погоды на рабочую неделю в Пермском крае

Как развивать свой бизнес в комфортной среде на платформе Monopoly.Online

Найдены стоматологи, к которым без страха бегут даже дети

За 8 лет подстригли 20 тысяч мужчин: как работает известный барбершоп Перми

«Трижды заводили сердце»: невеста борется за раненного в СВО жениха. Он лишился глаза и разучился говорить

Решил возродить традицию ношения подзабытых русских рубах. Пермяк, работающий в Индии учителем, шьет там нарядные косоворотки

Нельзя будет даже слетать на отдых? Как на Россию повлияют выборы президента в Турции

Минобороны РФ подтвердили взятие Бахмута, Зеленский сравнил город с Хиросимой: новости СВО за 21 мая

Близнецам — белая полоса, Рыбам — недвижимость: гороскоп на июнь 2023 года для всех знаков зодиака

В УДС «Молот» прошло ледовое шоу Ильи Авербуха «Ледниковый период. Снова вместе»: фоторепортаж

В народном голосовании Строгановской премии победили спасатель и создатель DS Crew. Но землячество не включило их в шорт-лист

Обостряют чувства! 6 самых продаваемых «женских» ароматов — они сводят с ума мужчин по всему миру

Серийный отравитель. История жестокого маньяка, который убил даже собственную дочь

«Каждый, кто не первый, тот второй»: эксперты рассказали, что отвечать на самый страшный вопрос партнера

На следующей неделе в школах России пройдут последние в учебном году «Разговоры о важном». Разбираем методички

Быстрый тест на возраст мозга: найдите 15 лиц на картинке

Подвалы университета и 90-е в ЦГК. В Перми прошла Ночь музеев: фоторепортаж

Язвы на пальцах и угрозы: как журналистка сортировала ковыль и клеила объявления — после этих историй вы откажетесь от подработок

Летом на берегу Камы устроят двухдневный музыкальный фестиваль. Хедлайнером будет Найк Борзов

Аптекарский огород: как вырастить пряные и полезные травы у себя на даче — что посадить

Когда пукать — не норма? А каким должен быть стул? Пермский гастроэнтеролог отвечает на неловкие вопросы

В Сочи или на Мальдивы? Куда отправиться этим летом без визы: самые бюджетные варианты

224 дня на операцию: всё, что известно о взятии Бахмута Россией

«Сначала пришли к ее подруге, где была Елена Першакова». У юриста, работавшей с пермским «Мемориалом»*, прошел обыск

Травматическая память. Тестируем Chery Tiggo 8 Pro с семьей, которая хейтит китайские машины

Замглавы Минобрнауки умер в самолете, летевшем с Кубы

Русский Шерлок: в новом детективе Шахназарова впечатляет только декольте героини, а Пореченкову хорошо удается кричать

ФСБ возбудила уголовное дело о госизмене на двоих жителей Пермского края

Это может быть даже рак: четыре опасные болезни, на которые указывает постоянный голод

Почему в аэропорту — военные самолеты? А где продают посикунчики? Разбираем 15 вопросов от туристов про Пермь

Роскачество назвало худшие корма для кошек: неожиданный список аутсайдеров

Мэр Краснодара поручил проверить все караоке-клубы города из-за песни Верки Сердючки: новости СВО за 20 мая

Все новости

Не бойтесь потерять голову, решая эти задания. Подумаешь, с кем не бывает

Поделиться

Иногда, чтобы дать правильный ответ, мало дочитать до конца вопрос — нужно еще понять его. В этом тесте есть и такие задания, условия которых, возможно, придется перечитать дважды. Но никто и не говорил, что будет легко. Вы ведь все равно не боитесь трудностей?

Илья Ненко

Шеф-редактор национальной редакции

ТестЗнанияМозгШкольные знанияШкольные заданияМатематические боиМатематикаЦифры

  • ЛАЙК29
  • СМЕХ9
  • УДИВЛЕНИЕ2
  • ГНЕВ8
  • ПЕЧАЛЬ4

Увидели опечатку? Выделите фрагмент и нажмите Ctrl+Enter

КОММЕНТАРИИ20

Читать все комментарии

Гость

Войти

Интересные задачи по математике — Со Вкусом

Учебный год в самом разгаре, и уже многие успели освежить в памяти некоторые математические теоремы. Не только дети, но и родители. Для многих алгебра и геометрия ассоциируются с чем-то скучным или сложным. Однако истинная причина такого отношения кроется в устаревших методиках и скучных упражнениях. Мы подготовили для вас интересные задачи по математике, которые могут быстро исправить это!

Наша редакция подобрала несколько примеров, которые помогут быстро привести мозги в тонус. Эти задания будут интересны как детям, так и взрослым. Ведь не стоит забывать, что кулинария — это, прежде всего, наука о пропорциях!

Интересные математические задачи

  1. 50 + 50 – 25 * 0 + 2 + 2 = ?

    Не спешите брать бумагу и карандаш. Решить этот пример с подвохом нужно в уме. Так сказать, для разминки!

  2. Следующий пример основан на закономерности. Попробуйте выяснить, сколько будет 11 + 5, если:
  3. 7 + 3 = 10421

    5 + 4 = 9120

    9 + 6 = 15354

  4. На кусте висело 12 яблок. Прохожий сорвал половину по пути домой и еще половину по пути из дома. Вопрос: сколько яблок сорвал прохожий?
  5. У повара было 6 яиц. Два он разбил, два приготовил и два съел. Вопрос: сколько осталось яиц?
  6. 143 мышки пробрались в сырный амбар и слопали 33 головки сыра. Все ели поровну, но половина мышек объелась, и на следующий день пришло 13 мышей. Снова ели поровну, но в 3 раза меньше. Вопрос: сколько было съедено головок сыра за два дня?

Ответы к задачам

Перейдем к самому интересному — к ответам. На самом деле здесь 2 из 5 примеров на логику. Давайте разбираться!

  1. Ответ: 104.
    Это упражнение нужно для того, чтобы вспомнить: сначала делают умножение и деление, а потом сложение и вычитание.
  2. Ответ: 16655.
    Смущает длина числа? Это ничего, ведь пример основан на закономерности, а не на сложении! В этом числе по очереди записаны результат сложение, вычитание и умножение первых двух чисел из уравнения — 11 и 5.
  3. Математический ответ, конечно же, 9. Но внимательные садоводы подметят: на кустах не растут яблоки! А значит, и у прохожего таковых нет.
  4. Ответ: 4 яйца. Всё просто: чтобы съесть яйца их нужно разбить и приготовить.
  5. Ответ: 34. 13 мышей — это в 11 раз меньше, чем в первый раз. Они бы съели 3 головки, но в этот раз их аппетит поугас в 3 раза. Значит, съели на второй день одну головку. 33 + 1 = 34.

Хорошие познания в математике просто необходимы в повседневной жизни! Подсчитать сдачу в магазине, рассчитать продукты для приготовления блюда и даже на неделю. Что уж говорить о подсчете калорий? Здесь главное — держать ум в тонусе. Подобные задачи дети решают с удовольствием, а взрослые используют подобные им в знаменитом «Что? Где? Когда?».

Автор статьи

Александра Береза

В кулинарной книге Александры есть сотни необычных рецептов с бережными заметками. Даже простой омлет Саша способна превратить в волшебное блюдо, завершенное листочком ароматной кинзы. Редактор «Со Вкусом» не боится экспериментировать и ежедневно оттачивает свое кулинарное мастерство. Утонченные десерты и выпечка — то, что делает Александра с особым вдохновением: чего только стоит клюквенно-апельсиновый кекс в ее исполнении!

40 занимательных математических вопросов с ответами – Bytelearn

Изучение математики может быть очень увлекательным, если учащимся показать ее правильно. Студенты убегают от сложных математических задач, но не знают, что решение этих вопросов может быть увлекательным и увлекательным. Разгадывание математических загадок и веселых математических вопросов — это интересная игра, которая помогает оттачивать наши способности, повышает концентрацию и умственные способности.

Со временем подходы к решению математики и восприятие предмета сильно изменились. Эти новые представления о математике сделали предмет и его концепции более привлекательными. Нужно только знать и понимать, что математика — это не просто предмет или задача; это осознанная и веселая деятельность для мозга.

Вы можете легко получить 50+ Забавных математических вопросов – Скачать бесплатно здесь!

Честно говоря, сложные математические вопросы или задачи — это все иллюзии; все дело в достаточном знании и понимании математики, ее концепций, простоты и взаимосвязи. Если у учителей есть правильное отношение и терпение, решение и изучение математики может быть очень увлекательным, приятным и восхитительным для учащихся. Вот несколько каверзных и увлекательных, но забавных математических вопросов с ответами для фантастического мозгового штурма.

Забавные математические вопросы для учащихся 

  1. Как составить цифру шесть из этих трех спичек, не разрезая их?

Более 100 бесплатных математических заданий, практических тестов и викторин

Ответ:  Просто переместите первую и вторую спички в форме буквы V; это сформирует шестерку в римском числе.

  1. Получите пять квадратов в следующей гирлянде. Но вы можете убрать или переместить только четыре спички.

Ответ: Удалите крайние спички из среднего ряда и столбца, как показано на рисунке.

  1. Какое число будет последним в следующем ряду чисел?

32, 45, 60, 77, ?

Ответ: 8 × 4 = 32, 9 × 5 = 45, 10 × 6 = 60, 11 × 7 = 77, 12 × 8 = 96.

Ответ 2: 32+13 = 45. 45 +15 = 60, 60+17 = 77, 77+19 = 96.

Итак, следующее число будет 96. 

Цифровой помощник, созданный с помощью ❤️ учителями

ByteLearn экономит ваше время и гарантирует, что каждый ученик получит необходимую поддержку

  1. Решите это уравнение.

Ответ:  Один треугольник = 10 согласно первому уравнению

              Один круг = 2 при решении второго уравнения

             Одна звезда = 1 при решении третьего уравнения

Итак, ответ 1.

  1. Найдите правильный ответ на уравнение.

Ответ: 3 яблока = 30; Итак, 1 яблоко = 10

Согласно второму уравнению один банан = 1

Согласно третьему уравнению половинка кокоса = 1

Итак, половинка кокоса + 1 яблоко + 3 банана = 12

  1. Заполнение место для вопросительного знака.

Ответ: (8×4) – (6×2) = 20

              (5×6) – (3×4) = 18

               (9×4) – (5×3) = 21

  1. Что положить в пустую коробку?

Ответ: Видя ряды

1+2= 3; 3+2= 5.

2+2= 4; 4+2=6.

Другой способ: См. столбцы

1 —> 2; 3 —> 4; 5 -> 6

  1. Исправьте это уравнение, но помните, что вы можете перемещать только одну палочку.

Ответ:  Переместите верхнюю правую спичку из числа 8 и поместите ее на  — подпишите, чтобы получилось + подпишите. Уравнение будет 9+6 = 15. 

  1. Какое число будет на месте вопросительного знака?

Ответ: 2 банана = 30; значит, 1 банан будет = 15

2 вишни + 2 вишни = 20; значит, 1 вишня будет = 5

2 яблока = 8; значит, 1 яблоко будет = 4

Следовательно; 1 банан + 1 вишня + 1 яблоко = 15 + 5 + 4 = 24

  1. Найдите число для пробела.

Ответ: 8+6 =14; 14+8 = 22; 22+10 = 32; 32+12 = 42,

Итак, ответом будет 42.

  1. Сосчитайте треугольники.

Ответ: 13

  1. Можете ли вы решить эту числовую головоломку?

Ответ: (7*4) + (4*5) = 28+20 = 48;

(9*3) + (3*6) = 27+18 = 45;

(5*?) + (?*8) = 13*? = 6

  1. Какое число отсутствует в следующей сетке?

Ответ: (3 + 2) х 2 = 10

(1 + 9) x 2 = 20

(0 + 8) x 2 = 16

(7 + 5) x 2 = 24 булавки я должен погладить, чтобы получить точные 100 очков?

Ответ: Зачеркните числа 48, 39, 13 как 48, потому что 48+39+13 = 100.

  1. Найдите подходящее число для пустого места в сетке.
2 0 5 8 3
0 7 4 4 5
9 0014 1 8 3 9 1
1 5 4 7  

Ответ:  Анализируя сетку, можно обнаружить, что четвертая строка содержит среднее число чисел в первых трех строках.

Итак, (3+5+1) / 3 = 3 — это ответ.

  1. Снова найти недостающий номер?
7 5 3 9 0014 4 6 8
8 3 4 2 7 ?

Ответ: Посмотрите внимательно; эти числа организованы в группы из двух цифр, таких как 75, 34 и 68 в первой строке и 83, 42 и 7. во второй строке.

Теперь та же самая последовательность следует за каждой цифрой. Первое число из разряда увеличивается на единицу, а второе число уменьшается на два. 6+1 = 7 и 8-2 = 6 в последней цифре.

Следовательно, отсутствует число 6. 

  1. Какое число отсутствует в данном круге?

Ответ:  Просто разделите круг на четыре части и посмотрите логику ответа.

5+6 = 11

8+3 = 11

2+9 = 11

7+? = 11 →? = 

18. Добавьте только одну спичку, чтобы сделать уравнение правильным.

Ответ: добавьте одну спичку к плюсу между любыми 5, чтобы получить 4. И 545+5 = 550.

  1. Найдите следующее число для следующей серии с помощью данных кубиков. 19, 11, 11, 9, ?

Ответ:   7+3+2 = 12

  1. Сколько других треугольников содержит этот треугольник?

Ответ: 23, посчитайте их внимательно и терпеливо.

  1. Найдите ответ на эту картинку-головоломку?

Ответ: Ответ 18.

Причина: 5*8 = 40; 8*3 = 24; и 5 * 6 = 30. Итак, 6 * 3 = 18,

  1.   Что тяжелее: перья весом в один фунт или кусок железа весом в один фунт?

Ответ: Оба одинаково тяжелее.

  1. В аквариуме десять рыб; двое из них тонут, четверо уплывают, а три рыбы умирают. Какое количество рыб находится в аквариуме?

Ответ:  Все десять рыб, потому что никто не вытащил ни одной рыбы из аквариума.

  1. Найдите ответ, если 2=6, 3=12, 4=20, 5=30, 6=42, 9″=»

Ответ: 2 = 2×3 = 6

3 = 3×4 = 12

4 = 4×5 = 20

5 = 5×6 = 30 90 003

6 = 6×7 = 42

7 = 7×8 = 56

8 = 8×9 = 72

9 = 9×10 = 90

Итак, 9 = 90.

  1. Угадай правильный ответ, если 1+4=5 ; 2+5 = 12; 3+6 = 21; 8+11 = ?

Ответ:   1+4 = 4×1+1 = 5

2+5 = 5×2+2 = 12

3+6 = 6×3+3 = 21

8 +11 = 11×8+8 = 96

  1. Как это возможно с числами?

6+4 = 210

9+2 = 711

7+5 = 212

9000 2 5+2 = 37

8+6 = 214

9+8 = 117

10+6 = 416

19+3 = 1622

Ответ: Эти уравнения записываются как отклонение чисел а потом сложение цифр.

Например: 6+4 = (6-4)(6+4) = 210.

  1. Решите это уравнение, чтобы найти X.

12/3 (5-3+2) + 2004 = X

Ответ:  X = 2004

  1. Решите эту викторину.

2+3 = 10

8+4 = 96

7+2 = 63

9 0014 6+5 = 66

9+5 = ??

Ответ:  В соответствии с примерами уравнений 2+3 = 2x(2+3) = 10; 7х(7+2) = 63; и так далее.

Итак, 9+5 = 9(9+5) = 126. 

  1. Разгадайте загадку этого числа.

Если 3, 2, 4 = 10

4, 3, 5 = 17

5, 4, 6 = 36

6, 5, 7 = 37

7, 6, 8 =?

Ответ:   Скрытое уравнение в этих числах похоже на 3X2+4 = 10 для первого.

Аналогично, 7X6+8 = 50

  1.   Илон принес контейнер с шестью плитками шоколада. Если ему нужно разделить плитки шоколада со своими шестью друзьями так, чтобы все они получили по одной плитке, в контейнере все еще остается плитка шоколада. Как он это сделал?

Ответ:  Это возможно, потому что он дал одну плитку шоколада пяти друзьям, и один из его друзей получил плитку шоколада в контейнере. Таким образом, все его друзья получили шоколадки, а одна до сих пор в контейнере.

  1. Когда мне было четыре года, моя сестра была вдвое моложе меня. Сейчас мне 18. Сколько сейчас лет моей сестре?

Ответ: Возраст моей сестры, когда мне было 4 года = половина моего возраста = 4/2 = 2 года.

Итак, моя сестра младше меня на два года.

Когда мне будет 18, ему будет 16 лет.

  1. У Гарри было шестеро братьев и сестер. У всех разница в два года. Самая младшая — Джесси, его семилетняя сестра, а Гарри — самый старший. Вычислите возраст Гарри.

Ответ: Младшей сестре Джесси семь лет. У Гарри шесть братьев и сестер, а это значит, что всего у них 7 братьев и сестер, и Гарри — старший среди них. У всех у них разница в возрасте два года, а значит, прибавление шесть раз два к семи может дать нам ответ. 7 + 2 + 2 + 2 + 2 + 2 + 2 = 19; Итак, Гарри 19 лет. 

  1. Поместите правильный символ из *, +, -, / в любом порядке, чтобы сбалансировать уравнение.

2 _ 1 _ 6 _ 6 = 48

Ответ: 2 * 1 + 6 * 6 = 48

  1. Обычно число нечетное. Но если убрать из него одну букву, оно изменится на четное. Угадай число.

Ответ:   Число 7 (семь). Когда буква «s» удаляется, оно становится «четным», но на самом деле 7 — нечетное число.

  1. Ангел посетила ярмарку возле своего дома. Она проехала на своем новом велосипеде, подаренном отцом за победу в художественном конкурсе. Добравшись до ярмарки, Ангел заметил, что припарковано 14 велосипедов и трехколесных велосипедов. Общее количество колес на парковке равно 38. Сможете ли вы найти количество трехколесных велосипедов, которые были припаркованы там?

Ответ: Всего велосипедов и трехколесных велосипедов = 14

Общее количество колес = 38

Все велосипеды имеют минимальное количество колес, 14 X 2 = 28;

38 – 28 = 10

Есть десять дополнительных колес, кроме велосипедов, то есть десять трехколесных велосипедов.

  1. Какие три числа дают одинаковый результат операций сложения и умножения?

Ответ: Три числа 1, 2 и 3; потому что 1 + 2 + 3 = 6. И 1 * 2 * 3 = 6.

  1. Какими будут следующие два числа в этом числовом ряду?

22, 21, 23, 22, 24, 23, ?, ?

Ответ:  Следующие два числа будут 25 и 24; схема ряда такова, что второе число на единицу меньше первого числа, а третье число на два больше второго.

Другой логикой может быть последовательный шаблон чисел, образованный чередующимися числами.

  1. Дедушка, два отца и два сына вместе ходили в театр. Сколько билетов им нужно купить, если по одному на каждого?

Ответ:  Им понадобится всего три билета, потому что это трио трех поколений: дедушка, его сын и его внук (сын сына).

  1. Джон прибавляет шесть к восьми и получает ответ два; его учитель также принимает это. Почему?

Ответ:   Потому что вопрос был вовремя. Она попросила добавить шесть часов к 8 утра. Итак, он получил правильный ответ в 14:00.

  1. Получите 1000, добавив всего восемь восьмерок.

Ответ:  888 + 88 + 8 + 8 + 8 = 1000

Вы также можете прочитать- Как делить дроби

Вывод:

Это были забавные, но сложные математические задачи, которые учащиеся могли использовать для мозгового штурма и вспоминать понятия, формулы и процедуры, которым их учили учителя. Наряду с преимуществами обучения, эти забавные математические вопросы могут привести к положительной стороне изучения математики, и они понимают, что математика не является ни сложным, ни бесполезным предметом.

Тем не менее, учителя также должны понимать, что заучивание процедур – это не то, что требуется ученикам. Поэтому вместо этого они должны сосредоточиться на создании понимания математики и развитии логического мышления у учащихся.

Бесплатные, соответствующие стандартам рабочие листы по математике

Введите адрес электронной почты, и мы вышлем вам образцы наших самых популярных математических листов.

20 занимательных математических вопросов с решениями

Вы тоже боитесь математики? Математика всегда была для вас настоящей задачей? Считаете ли вы, что миф о математических генах реален?

Ну, поверьте нам, мы чувствуем вас, и мы здесь, чтобы разрушить миф о математических генах для вас. Математика может быть действительно веселой и занимательной, если на нее смотреть с правильной стороны.

Все мы прошли через ту пугающую фазу, когда мы ломали голову над пониманием математики и лежащей в ее основе логики, и все же в день экзамена оказывались в полном замешательстве по поводу вопроса. Но давайте подойдем к этому по-другому, давайте посмотрим на математику как на игру, и тогда никогда не будет казаться, что математических вопросов сложны.

Математика становится намного интереснее, если к ней подходить с другой стороны. Каждый математический вопрос, уравнение и диаграмма по математике говорят о многом, и, как правило, большинство шагов вашего ответа находятся в самом вопросе.

Для начала возьми себе за правило ежедневно заниматься математикой, и уверяем тебя, через несколько дней это покажется проще твоего страха. Свяжите это со своей повседневной жизнью и осваивайте математику с практикой. Крайне важно разрушить мифы, связанные с математикой и математическими генами, с самого начала, потому что страх математики иногда становится единственной проблемой для нежелания попробовать.

Хотите знать, почему математика так важна? Самый простой ответ — потому что это важнейший жизненный навык, который поможет вам на каждом этапе вашей жизни, от самых простых вещей, таких как расчет вашего процента, до планирования бюджета в дальнейшей жизни. Давайте сломаем стереотип математического гена, решая несколько простых, кажущихся сложными, но тем не менее занимательных математических задач с их пошаговыми решениями.

Благодаря этому вы столкнетесь с шаблонами, логикой и концепциями, которые помогут побороть страх перед математикой и внутренние убеждения, поразив вас осознанием того, что не «математический ген», а практика делает вас мастером математики.

Сложные, но забавные математические вопросы с решениями

Пришло время провести мозговой штурм и активно поработать, чтобы бросить вызов своим мыслительным способностям и улучшить свои математические навыки, сочетая эти хитрые, логические, кажущиеся сложными математические вопросы .

Вопрос 1. Если 1=3, 2=3, 3=5, 4=4; 5=4, тогда 6=?

а.2
б.5
в.3
г.4
Ответ. в. 3

Если вы заметили закономерность «один» = 3, «три» = 5 и т. д., в зависимости от количества букв записывается число. Итак, поскольку «шесть» состоит из трех букв, следовательно, 6 = 3.

Вопрос 2. У Джои было 6 братьев и сестер. Все они родились с разницей в 2 года. Младшей из них является Хлоя, которой всего 7 лет, а Джоуи — самая старшая. Вычислите возраст Джоуи.

a.12
b.18
c.22
d.19

Ответ. Возраст младшей сестры, Хлои, 7 лет. Всего 7 братьев и сестер, Джоуи плюс его 6 братьев и сестер. Принято считать, что все они родились с разницей в 2 года. Таким образом, возраст Джоуи будет

7 + 2 + 2 + 2 + 2 + 2 + 2 = 19

Вопрос 3. Для участия в выставке заявлено 49 собак. Маленьких собак зарегистрировалось на 36 больше, чем крупных собак. Сколько маленьких собак заявлено для участия в соревнованиях?

a. 13
b.42.5
c.6.5
d.42
Ответ. Чтобы узнать количество соревнующихся собак, сначала нужно вычесть 36 из 49, а затем разделить результат на 2. То есть 13 разделить на 2, чтобы получить 6,5. 6,5 — это количество зарегистрировавшихся больших собак. Нет, это еще не окончательный ответ. Следующим шагом является прибавление 6,5 к 36. Таким образом, ответ равен 42,5, а поскольку мы все знаем, что полусобака не может участвовать, то это гипотетический вопрос, и мы должны принять 42,5 в качестве ответа.

Вопрос 4. Решите: 3 + 2 • (8 – 3)

a.13
b.24
c.15
d.17
Ответ. 3 + 2 • (8 – 3)
= 3 + 2 (5)
= 3 + 10
= 13

Вопрос 5. Проанализируйте закономерность и найдите пропущенное число:

а.9
б.5
c.6
d.1
Анс. Если вы проанализируете узор в каждом из полных кругов, вы поймете, что числа в сумме дают 20. Следовательно, для неполного круга 90 809 2 + 9 + 8 + ? = 20,
19+? = 20
? = 20 – 19 = 1

Вопрос 6. Я нечетное число. Убери одну букву и я стану квитком. Какой я номер?

Ответ. Ответ на этот вопрос — «семь», что является нечетным числом, а когда вы уберете «s» из «семерки», оно станет «четным».

Вопрос 7. Под каким номером припаркован автомобиль?

Ответ. Хотите верьте, хотите нет, но вам не понадобятся никакие расчеты и даже 5 секунд, чтобы решить этот вопрос, когда вы смотрите на картинку вверх ногами. Оказывается, это просто последовательность чисел и ответ 87.

Вопрос 8. Анкит посетил выставку возле своего дома. Он катался на своем новом велосипеде, подаренном его матерью за победу в научном конкурсе. Дойдя до выставки, Анкит увидел, что всего там 14 велосипедов и трехколесных велосипедов. Учитывая, что общее количество колес равно 38, найдите количество трехколесных велосипедов в парке.

Ответ. Общее количество циклов = 14
У каждого велосипеда не менее 2 колес.
14 x 2 = 28
Общее количество приведенных колес = 38
38 – 28 = 10
Это означает, что имеется 10 велосипедов с одним дополнительным колесом в каждом, поэтому общее количество трехколесных велосипедов в парке равно 10.

Вопрос 9. Какие три числа при сложении или перемножении дают одинаковый ответ?

Ответ. 1 + 2 + 3 = 6 и 1 х 2 х 3 = 6; поэтому ответ равен 1, 2 и 3.

Вопрос 10. Решите следующее уравнение:

9 – 3 ÷ 1/3+ 1 =?
Ответ. Ответ на этот вопрос: 1. Все, что вам нужно сделать, это перевернуть дробь, чтобы превратить деление в умножение.
9 – 3 х 3 + 1 =?
9 – 9 + 1 = 1

Вопрос 11. Рис весом 33/4 фунта был разделен поровну и помещен в 4 контейнера. Сколько унций риса было в каждом?

Ответ. 33/4 ÷ 4 фунта.
= (4 × 3 + 3)/4 ÷ 4 фунта.
= 15/4 ÷ 4 фунта.
= 15/4 × 1/4 фунта.
= 15/16 фунтов.
Мы знаем, что 1 фунт = 16 унций.
Следовательно, 15/16 фунтов = 15/16 × 16 унций.
= 15 унций.

Вопрос 12. Джессика купила корзину с 5 яблоками. Если бы она должна была разделить яблоки так, чтобы каждый из ее 5 учеников получил по 1 яблоку и 1 яблоко осталось в корзине. Как она это сделает?

Ответ. Джессика может раздать по 4 яблока 4 ученикам и дать пятому ученику корзину с яблоком.

Вопрос 13. Добавьте 8,254 и 4,2672.

Ответ. Сложение цифр с десятичной дробью так же просто, как и простое сложение. Тот факт, что 8,254 имеет меньше цифр, чем 4,2672, не имеет значения. Все, что вам нужно сделать, это просто добавить 0 в конце 8.254. Таким образом, ответ становится 12,5212.

Вопрос 14. Составьте уравнение, используя четыре семерки (7) и единицу (1), чтобы получить ответ как 100.

Ответ. 177 – 77 = 100

Вопрос 15. Если 1 = 5; 2 = 25; 3 = 325 и 4 = 4325; тогда 5 =?

Ответ. Немедленный ответ, который возникнет у вас в голове, глядя на паттерн, — 54325. Но это неправильный ответ. Правильный ответ дан в самом вопросе, так как вопрос гласит, что 1 = 5, тогда 5 = 1.

Вопрос 16. Предположим, что на складе находится 85 человек, из которых некоторые превратились в зомби, а некоторые еще живой. Если соотношение зомби к количеству живых людей составляет 2:3, подсчитайте общее количество зомби.

Ответ. Нам дано, что на каждых 3 человек приходится 2 зомби.
2 + 3 = 5
Чтобы вычислить общее количество групп людей и зомби, делим общую силу на 5.
85/5 = 17.
Теперь, чтобы получить общую численность. зомби и людей, умножим 17 на 2 и 3 соответственно. Таким образом, ответ: 34 зомби и 51 человек.

Вопрос 17. Глядя на этот ряд: 22, 21, 23, 22, 24, 23, … Какое число должно быть следующим?

Ответ: Если вы заметили, чередующиеся числа в ряду создают последовательный шаблон чисел. Таким образом, следуя шаблону, ответ будет 25.

Вопрос 18. Найдите площадь красного треугольника.

Ответ: Для решения этого вопроса необходимо знать формулы нахождения площади треугольника и площади параллелограмма и соотношения между ними.
Так как площадь треугольника равна половине площади параллелограмма.
Теперь, когда вы это знаете, прибавив 79 и 10, а затем вычтя 72 и 8, вы получите ответ как 9.

Вопрос 19. Предположим, есть трехзначное число. Вторая цифра числа в четыре раза больше третьей, а первая в три раза меньше второй. Найдите число.

Ответ. Число 141, так как 4 в четыре раза больше 1, а также в три раза меньше 1.

Вопрос 20. Составьте правильное уравнение, используя числа 2, 3, 4 и 5 и символы = и +.

Ответ. Наилучшее возможное уравнение с данными числами и символами будет таким:
5 + 2 = 3 + 4

Вопрос 21. Если 500 учащихся посещают среднюю школу Вашингтона, сколько из них поедет в горы на каникулы?

А. 25
Б. 60
В. 75
Г. 100
Д. 125   900 03

Ответ: B.60

Почему важно смотреть на математические задачи Из веселого подхода?

Важно смотреть на математические задачи с интересной точки зрения и превращать математику в увлекательное занятие, потому что улучшение ваших математических навыков поможет вам в долгосрочной перспективе.

Что такое x и y в математике: Что означает в математике запись у = f(x) — урок. Алгебра, 7 класс.

ФУНКЦИЯ • Большая российская энциклопедия

ФУ́НКЦИЯ (от лат. functio – ис­пол­не­ние, осу­ще­ст­в­ле­ние), од­но из ос­нов­ных по­ня­тий ма­те­ма­ти­ки, оз­на­чаю­щее за­ви­си­мость од­них пе­ре­мен­ных ве­ли­чин от дру­гих. Сло­во «ве­ли­чи­на» в этом оп­ре­де­ле­нии по­ни­ма­ет­ся в са­мом ши­ро­ком смыс­ле: это мо­жет быть име­но­ван­ное чис­ло, от­вле­чён­ное чис­ло (дей­ст­ви­тель­ное или ком­плекс­ное), неск. чи­сел (т. е. точ­ка про­стран­ст­ва) и во­об­ще эле­мент лю­бо­го мно­же­ст­ва.

Действительная функция одного действительного переменного

В про­стей­шем слу­чае, ко­гда ве­ли­чи­на – дей­ст­ви­тель­ное чис­ло, по­ня­тие «Ф.» оп­ре­де­ля­ет­ся сле­дую­щим об­ра­зом. Пусть ка­ж­до­му чис­лу $x$ из за­дан­но­го мно­же­ст­ва $E$ по­став­ле­но в со­от­вет­ст­вие чис­ло $y$, обо­зна­чае­мое $y=f(x)$ (чи­та­ет­ся «иг­рек ра­вен эф от икс»). То­гда го­во­рят, что на мно­же­ст­ве $E$ за­да­на функ­ция $y=f(x)$, $x∈E$. При этом упот­реб­ля­ют­ся сле­дую­щие тер­ми­ны: $x$ – не­за­ви­си­мое пе­ре­мен­ное, или ар­гу­мент; $y$ – за­ви­си­мое пе­ре­мен­ное, или функ­ция; $E$ – мно­же­ст­во зна­че­ний, ко­то­рые мо­жет при­ни­мать $x$, – об­ласть оп­ре­де­ле­ния, или об­ласть за­да­ния Ф. (об­ла­стью оп­ре­де­ле­ния Ф. мо­жет быть мно­же­ст­во всех дей­ст­ви­тель­ных чи­сел, ин­тер­вал, от­ре­зок и т. п.). Сло­ва «по­став­ле­но в со­от­вет­ст­вие» оз­на­ча­ют, что ука­зан оп­ре­де­лён­ный спо­соб, по ко­то­ро­му для ка­ж­до­го $x∈E$ на­хо­дит­ся зна­че­ние $y=f(x)$. Этот спо­соб в дан­ном слу­чае обо­зна­чен сим­во­лом $f$. Для обо­зна­че­ния Ф. при­ме­ня­ют­ся и др. бу­к­вы, напр. $y=g(x)$, $y=F(x)$, $s=h(t)$, $v=φ(s)$.

Во всех слу­ча­ях, ко­гда упот­реб­ля­ет­ся тер­мин «Ф.», под­ра­зу­ме­ва­ет­ся, ес­ли не ого­во­ре­но про­тив­ное, од­но­знач­ная Ф., т. е. та­кое со­от­вет­ст­вие, при ко­то­ром ка­ж­до­му зна­че­нию ар­гу­мен­та $x$ со­от­вет­ст­ву­ет толь­ко од­но зна­че­ние Ф. $y$. Ес­ли од­но­му и то­му же зна­че­нию ар­гу­мен­та со­от­вет­ст­ву­ет нес­коль­ко (быть мо­жет, да­же бес­ко­неч­ное мно­же­ст­во) зна­че­ний $y$, то $y=f(x)$ на­зы­ва­ет­ся мно­го­знач­ной функ­ци­ей ар­гу­мен­та $x$.

Способы задания функции

Аналитический способ задания функции

Наи­бо­лее рас­про­стра­нён ана­ли­тич. {2m}.$$

Графический способ задания функции

Рас­про­стра­нён гра­фич. спо­соб за­да­ния Ф. Гра­фи­ком Ф. $y=f(x)$, $y∈E$, на­зы­ва­ет­ся мно­же­ст­во то­чек плос­ко­сти с пря­мо­уголь­ны­ми ко­ор­ди­на­та­ми $(x,y)$, где $x∈E$, $y=f(x)$. Гра­фич. спо­соб за­да­ния Ф. ши­ро­ко при­ме­ня­ет­ся на прак­ти­ке. Так, мн. про­цес­сы из­ме­не­ния од­ной ве­ли­чи­ны в за­ви­си­мо­сти от дру­гой ис­сле­ду­ют­ся с по­мо­щью кри­вых, за­пи­сан­ных с по­мо­щью са­мо­пи­шу­щих при­бо­ров. Хо­тя гра­фик Ф. и не да­ёт воз­мож­но­сти точ­но­го оп­ре­де­ле­ния чис­лен­ных зна­че­ний $x$ и $y$, он на­гляд­но от­ра­жа­ет ка­че­ст­вен­ное по­ве­де­ние Ф. (не­пре­рыв­ность, мо­но­тон­ность, мак­си­му­мы и ми­ни­му­мы, точ­ки пе­ре­ги­ба и т. д.) и по­это­му яв­ля­ет­ся важ­ным сред­ст­вом ис­сле­до­ва­ния функ­ции.

Табличный способ задания функции

При таб­лич­ном спо­со­бе за­да­ния Ф. за­да­ёт­ся в ви­де таб­ли­цы, в ко­то­рой для ка­ж­до­го зна­че­ния ар­гу­мен­та ука­зы­ва­ет­ся со­от­вет­ст­вую­щее ему зна­че­ние Ф. Та­кой спо­соб за­да­ния Ф. час­то при­ме­ня­ет­ся в тех слу­ча­ях, ко­гда об­ласть оп­ре­де­ле­ния со­сто­ит из ко­неч­но­го чис­ла зна­че­ний.

Действительная функция нескольких действительных переменных

Ф. от двух пе­ре­мен­ных оп­ре­де­ля­ет­ся сле­дую­щим об­ра­зом. Рас­смат­ри­ва­ет­ся мно­же­ст­во $E$ упо­ря­до­чен­ных пар чи­сел $(x,y)$. Ес­ли ка­ж­дой па­ре $(x,y)∈E$ по­став­ле­но в со­от­вет­ст­вие дей­ст­ви­тель­ное чис­ло $z$, то го­во­рят, что на мно­же­ст­ве $E$ оп­ре­де­ле­на Ф. $z=f(x,y)$ от двух пе­ре­мен­ных $x$ и $y$. Т. к. ка­ж­дой па­ре чи­сел $(x,y)$ со­от­вет­ст­ву­ет на плос­ко­сти точ­ка с ко­ор­ди­на­та­ми $(x,y)$, то Ф. $f(x,y)$ за­да­на на мно­же­ст­ве $E$ то­чек плос­ко­сти. Гра­фик Ф. $z=f(x,y)$ мож­но изо­бра­зить в трёх­мер­ном про­стран­ст­ве, где за­да­на пря­мо­уголь­ная сис­те­ма ко­ор­ди­нат $(x,y,z)$, в ви­де мно­же­ст­ва то­чек $(x,y,f(x,y))$, про­ек­ции ко­то­рых на плос­кость $(x,y)$ при­над­ле­жат мно­же­ст­ву $E$. Напр., гра­фик функ­ции $z=\sqrt{1-x^2-y^2},$ $x^2+y^2 ⩽ 1$, и име­ет­ся в ви­ду ариф­ме­тич. ко­рень, изо­бра­жа­ет­ся верх­ней по­ло­ви­ной ша­ро­вой по­верх­но­сти ра­диу­са 1 с цен­тром в на­ча­ле ко­ор­ди­нат.

Ана­ло­гич­но мож­но рас­смат­ри­вать мно­же­ст­во $E$, со­стоя­щее из упо­ря­до­чен­ных сис­тем $(x_1,x_2,…,x_n)$ из $n$ чи­сел, и Ф. $z=f(x_1,x_2,…,x_n)$ от $n$ пе­ре­мен­ных, оп­ре­де­лён­ную на мно­же­ст­ве $E$.

Общее понятие функции

Пусть за­да­ны мно­же­ст­ва $E$ и $E_1$ эле­мен­тов лю­бой при­ро­ды и пусть ка­ж­до­му эле­мен­ту $x∈E$ по­став­лен в со­от­вет­ст­вие эле­мент $y∈E_1$, обо­зна­чае­мый $y=f(x)$. То­гда го­во­рят, что за­да­на функ­ция $y=f(x)$, $x∈E$, что час­то за­пи­сы­ва­ет­ся как $f:\,E→E_1$.

При­ня­та сле­дую­щая тер­ми­но­ло­гия: $x$ – не­за­ви­си­мое пе­ре­мен­ное, или ар­гу­мент; $E$ – об­ласть оп­ре­де­ле­ния Ф., ка­ж­дый эле­мент $x∈E$ – зна­че­ние ар­гу­мен­та; $y$ – за­ви­си­мое пе­ре­мен­ное, или Ф., от ар­гу­мен­та $x$; $E_1$ – об­ласть зна­че­ний Ф., ка­ж­дый эле­мент $y∈E_1$ та­кой, что $y=f(x)$ для не­ко­то­ро­го зна­че­ния $x∈E$, на­зы­ва­ет­ся зна­че­ни­ем функ­ции. 2}$ ото­бра­жа­ет от­ре­зок $–1 ⩽ x ⩽ 1$ на от­ре­зок $0 ⩽ y ⩽ 1$.

Для Ф. $f(x)$ и $g(x)$ ес­те­ст­вен­ным об­ра­зом оп­ре­де­ля­ют­ся ариф­ме­тич. опе­ра­ции: это Ф., при­ни­маю­щие (в тех слу­ча­ях, ко­гда это име­ет смысл) зна­че­ния $f(x)±g(x)$, $f(x)g(x)$, $f(x)/g(x)$.

Тер­мин «Ф.» ча­ще все­го ис­поль­зу­ет­ся толь­ко для обо­зна­че­ния чи­сло­вой Ф. от од­но­го или не­сколь­ких пе­ре­мен­ных (дей­ст­ви­тель­ных или ком­плекс­ных). В др. слу­ча­ях, как пра­ви­ло, ис­поль­зу­ют­ся спец. тер­ми­ны: опе­ра­тор, ото­бра­же­ние, пре­об­ра­зо­ва­ние, функ­цио­нал.

См. так­же Мо­но­тон­ная функ­ция, Не­пре­рыв­ная функ­ция, Пе­рио­ди­че­ская функ­ция, Спе­ци­аль­ные функ­ции, Чёт­ные и не­чёт­ные функ­ции, Эле­мен­тар­ные функ­ции.

Исторический очерк

Как и ос­таль­ные по­ня­тия ма­те­ма­ти­ки, по­ня­тие Ф. сло­жи­лось не сра­зу, а про­шло дол­гий путь раз­ви­тия. По су­ще­ст­ву, речь о функ­цио­наль­ной за­ви­си­мо­сти и её гра­фич. изо­бра­же­нии идёт в ра­бо­те П.  Фер­ма «Вве­де­ние и изу­че­ние пло­ских и те­лес­ных мест» (1636, опубл. в 1679). Изу­че­ние ли­ний по их урав­не­ни­ям в «Гео­мет­рии» Р. Де­кар­та (1637) так­же ука­зы­ва­ет на яс­ное пред­став­ле­ние о вза­им­ной за­ви­си­мо­сти двух пе­ре­мен­ных ве­ли­чин. У англ. ма­те­ма­ти­ка И. Бар­роу («Лек­ции по гео­мет­рии», 1670) в гео­мет­рич. фор­ме ус­та­нав­ли­ва­ет­ся вза­им­ная об­рат­ность дей­ст­вий диф­фе­рен­ци­ро­ва­ния и ин­тег­ри­ро­ва­ния (ра­зу­ме­ет­ся, без упот­реб­ле­ния са­мих этих тер­ми­нов). Это сви­де­тель­ст­ву­ет о со­вер­шен­но от­чёт­ли­вом вла­де­нии по­ня­ти­ем Ф. В гео­мет­рич. и ме­ха­нич. ви­де это по­ня­тие мож­но най­ти и у И. Нью­то­на. Од­на­ко тер­мин «Ф.» впер­вые по­яв­ля­ет­ся лишь в 1692 у Г. Лейб­ни­ца, и при­том не со­всем в со­вре­мен­ном его по­ни­ма­нии. Лейб­ниц на­зы­ва­ет Ф. разл. от­рез­ки, свя­зан­ные с к.-л. кри­вой, напр. абс­цис­сы её то­чек. В пер­вом пе­чат­ном кур­се «Ана­ли­за бес­ко­неч­но ма­лых» франц. ма­те­ма­ти­ка Г. Ло­пи­та­ля (1696) тер­мин «Ф. » не упот­реб­ля­ет­ся.

Пер­вое оп­ре­де­ле­ние Ф. в смыс­ле, близ­ком к со­вре­мен­но­му, встре­ча­ет­ся у И. Бер­нул­ли (1718): «Функ­ция – это ве­ли­чи­на, со­став­лен­ная из пе­ре­мен­ной и по­сто­ян­ной». В ос­но­ве это­го не впол­не от­чёт­ли­во­го оп­ре­де­ле­ния ле­жит идея за­да­ния Ф. ана­ли­тич. фор­му­лой. Та же идея вы­сту­па­ет и в оп­ре­де­ле­нии Л. Эйле­ра, дан­ном им во «Вве­де­нии в ана­лиз бес­ко­неч­ных» (1748): «Функ­ция пе­ре­мен­но­го ко­ли­че­ст­ва есть ана­ли­ти­че­ское вы­ра­же­ние, со­став­лен­ное ка­ким-ли­бо об­ра­зом из это­го пе­ре­мен­но­го ко­ли­че­ст­ва и чи­сел или по­сто­ян­ных ко­ли­честв». На про­тя­же­нии 18 в. от­сут­ст­во­ва­ло до­ста­точ­но яс­ное по­ни­ма­ние раз­ли­чия ме­ж­ду Ф. и её ана­ли­тич. вы­ра­же­ни­ем. С нач. 19 в. уже всё ча­ще и ча­ще оп­ре­де­ля­ют по­ня­тие Ф. без упо­ми­на­ния о её ана­ли­тич. вы­ра­же­нии. Та­кие оп­ре­де­ле­ния встре­ча­ют­ся в ра­бо­тах Ж. Фу­рье (1822), Д. Ди­рих­ле (1829, 1837), Н. И. Ло­ба­чев­ско­го (1834). Так сло­жи­лось совр. по­ня­тие Ф., сво­бод­ное от упо­ми­на­ния о её ана­ли­тич. за­да­нии.

Математические функции — Visual Basic

  • Статья

Методы System.Math класса предоставляют тригонометрические, логарифмические и другие распространенные математические функции.

В следующей таблице перечислены методы System.Math класса . Их можно использовать в программе Visual Basic:

Метод .NETОписание
AbsВозвращает абсолютное значение числа.
AcosВозвращает угол, косинус которого равен указанному числу.
AsinВозвращает угол, синус которого равен указанному числу.
AtanВозвращает угол, тангенс которого равен указанному числу.
Atan2Возвращает угол, тангенс которого равен отношению двух указанных чисел.
BigMulВозвращает полное произведение двух 32-разрядных чисел.
CeilingВозвращает наименьшее целочисленное значение, которое больше или равно заданному Decimal или Double.
CosВозвращает косинус указанного угла.
CoshВозвращает гиперболический косинус указанного угла.
DivRemВозвращает частное число двух 32-разрядных или 64-разрядных целых чисел со знаком, а также остаток в выходном параметре.
ExpВозвращает e (основание естественных логарифмов), возведенное в указанную степень.
FloorВозвращает наибольшее целое число, которое меньше или равно заданному Decimal числу.Double
IEEERemainderВозвращает остаток, полученный в результате деления указанного числа на другое указанное число.
LogВозвращает натуральный (базовый e) логарифм указанного числа или логарифм указанного числа в указанном основании.
Log10Возвращает логарифм с основанием 10 указанного числа.
MaxВозвращает большее из двух чисел.
MinВозвращает меньшее из двух чисел.
PowВозвращает указанное число, возведенное в указанную степень.
RoundDecimal Возвращает значение или Double , округленное до ближайшего целочисленного значения или до указанного числа дробных цифр.
SignInteger Возвращает значение, указывающее знак числа.
SinВозвращает синус указанного угла.
SinhВозвращает гиперболический синус указанного угла.
SqrtВозвращает квадратный корень из указанного числа.
TanВозвращает тангенс указанного угла.
TanhВозвращает гиперболический тангенс указанного угла.
TruncateВычисляет целочисленную часть указанного Decimal числа или Double .

В следующей System.Math таблице перечислены методы класса , которые не существуют в платформа .NET Framework но добавляются в .NET Standard или .NET Core:

Метод .NETОписаниеДоступно в
AcoshВозвращает угол, гиперболический косинус которого равен указанному числу.Начиная с .NET Core 2.1 и .NET Standard 2.1
AsinhВозвращает угол, гиперболический синус которого равен указанному числу.Начиная с .NET Core 2.1 и .NET Standard 2.1
AtanhВозвращает угол, гиперболический тангенс которого равен указанному числу.Начиная с .NET Core 2.1 и .NET Standard 2.1
BitDecrementВозвращает ближайшее самое маленькое значение, которое меньше, чем x.Начиная с .NET Core 3.0
BitIncrementВозвращает ближайшее самое большое значение, превышающее x.Начиная с .NET Core 3.0
CbrtВозвращает кубический корень из указанного числа.Начиная с .NET Core 2.1 и .NET Standard 2.1
ClampВозвращает value, ограниченное диапазоном от min до max включительно.Начиная с .NET Core 2.0 и .NET Standard 2.1
CopySignВозвращает значение с величиной x и знаком y.Начиная с .NET Core 3.0
FusedMultiplyAddВозвращает значение (x * y) + z, округленное в рамках одной тернарной операции.Начиная с .NET Core 3.0
ILogBВозвращает целочисленный логарифм с основанием 2 указанного числа.Начиная с .NET Core 3.0
Log2Возвращает логарифм с основанием 2 указанного числа. n, вычисленное эффективно.Начиная с .NET Core 3.0

Чтобы использовать эти функции без квалификации, импортируйте System.Math пространство имен в проект, добавив следующий код в начало исходного файла:

Imports System.Math

Пример — Abs

В этом примере используется Abs метод класса для Math вычисления абсолютного значения числа.

Dim x As Double = Math.Abs(50.3)
Dim y As Double = Math.Abs(-50.3)
Console.WriteLine(x)
Console.WriteLine(y)
' This example produces the following output:
' 50.3
' 50.3

Пример — Atan

В этом примере используется Atan метод класса для Math вычисления значения pi.

Public Function GetPi() As Double
    ' Calculate the value of pi.
    Return 4.0 * Math.Atan(1.0)
End Function

Примечание

Класс System.Math содержит поле константы Math.PI . Его можно использовать, а не вычислять.

Пример — Cos

В этом примере метод класса используется CosMath для возврата косинуса угла.

Public Function Sec(angle As Double) As Double
    ' Calculate the secant of angle, in radians.
    Return 1.0 / Math.Cos(angle)
End Function

Пример — Exp

В этом примере используется Exp метод класса для Math возврата e, возведенного в степень.

Public Function Sinh(angle As Double) As Double
    ' Calculate hyperbolic sine of an angle, in radians.
    Return (Math.Exp(angle) - Math.Exp(-angle)) / 2.0
End Function

Пример — журнал

В этом примере метод класса используется Log для Math возврата естественного логарифма числа.

Public Function Asinh(value As Double) As Double
    ' Calculate inverse hyperbolic sine, in radians.
    Return Math.Log(value + Math.Sqrt(value * value + 1.0))
End Function

Пример — Циклический

В этом примере метод класса используется Round для Math округления числа до ближайшего целого числа.

Dim myVar2 As Double = Math.Round(2.8)
Console.WriteLine(myVar2)
' The code produces the following output:
' 3

В этом примере используется Sign метод Math класса для определения знака числа.

Dim mySign1 As Integer = Math.Sign(12)
Dim mySign2 As Integer = Math.Sign(-2.4)
Dim mySign3 As Integer = Math.Sign(0)
Console.WriteLine(mySign1)
Console.WriteLine(mySign2)
Console.WriteLine(mySign3)
' The code produces the following output:
' 1
' -1
' 0

Пример— Sin

В этом примере используется Sin метод Math класса для возврата синуса угла.

Public Function Csc(angle As Double) As Double
    ' Calculate cosecant of an angle, in radians.
    Return 1.0 / Math.Sin(angle)
End Function

Пример. Sqrt

В этом примере метод класса используется SqrtMath для вычисления квадратного корня числа.

Dim mySqrt1 As Double = Math.Sqrt(4)
Dim mySqrt2 As Double = Math.Sqrt(23)
Dim mySqrt3 As Double = Math.Sqrt(0)
Dim mySqrt4 As Double = Math.Sqrt(-4)
Console.WriteLine(mySqrt1)
Console.WriteLine(mySqrt2)
Console.WriteLine(mySqrt3)
Console.WriteLine(mySqrt4)
' The code produces the following output:
' 2
' 4.79583152331272
' 0
' NaN

Пример. Tan

В этом примере метод класса используется Tan для Math возврата тангенсов угла.

Public Function Ctan(angle As Double) As Double
    ' Calculate cotangent of an angle, in radians.
    Return 1.0 / Math.Tan(angle)
End Function

См. также раздел

  • Rnd
  • Randomize
  • NaN
  • Производные математические функции
  • Арифметические операторы

Ось X и Y

Ось X и Y являются осями в декартовой системе координат. Вместе они образуют координатную плоскость, которая представляет собой пространство, в котором происходит двумерное графическое изображение.

В двумерном пространстве ось X является горизонтальной осью, а ось Y — вертикальной осью. Они представлены двумя числовыми линиями, которые пересекаются перпендикулярно в начале координат, расположенном в точке (0, 0), как показано на рисунке ниже.


Приведенное выше представление координатной плоскости является одной из самых основных форм, где каждая галочка на оси представляет собой 1 единицу. Стоит отметить, что оси могут быть помечены любым количеством способов, если каждая ось поддерживает постоянный интервал. Например, вместо того, чтобы ось X считала вверх на 1, первая отметка справа от 0 может быть помечена как 2, последующая — как 4, а следующая — как 6. В этом случае расстояние между каждой отметкой представляет собой 2. единицы измерения. Ось Y в этом же примере может даже иметь деления, представляющие 3 единицы; ось X и ось Y не должны иметь одинаковый интервал между метками. Поэтому важно обращать внимание на маркировку осей, поскольку различия в выбранных интервалах осей могут существенно повлиять на форму данного графика.

Упорядоченные пары

Заданная точка на координатной плоскости обозначается с помощью так называемой упорядоченной пары. Упорядоченная пара — это пара значений, в которой первое значение указывает координату x, а второе — координату y. Упорядоченная пара записывается следующим образом:

(x, y)

, где x — значение x, а y — значение y. В упорядоченной паре значения x и y заключены в круглые скобки и разделены запятой. Обратите внимание, что причина, по которой она называется упорядоченной парой, заключается в том, что сначала должно идти значение x, а затем значение y. Другими словами, (x, y) не то же самое, что (y, x). Рассмотрим точки A, B и C, нанесенные на координатную плоскость ниже.


Красные стрелки показывают, как мы движемся от начала координат к заданной точке координатной плоскости, как описано ниже.

  1. Для точки A переместитесь вправо на 3 единицы по оси x, чтобы получить координату x, равную 3, затем переместитесь на 4 единицы вверх по оси y, чтобы получить координату y, равную 4.

  2. Для точки B переместитесь на 4 единицы влево по оси x, чтобы получить координату x, равную -4. Поскольку вертикальное расстояние для перемещения по оси Y отсутствует, координата Y равна 0,
  3. .
  4. Для точки C переместитесь на 4 единицы вправо по оси x, чтобы получить координату x, равную 4, затем переместитесь на 4 единицы вниз по оси y, чтобы получить координату y, равную -4.

Таким образом, чтобы нанести точку на координатную плоскость, нам просто нужно знать упорядоченную пару. Затем мы можем нанести точку, посчитав соответствующее количество единиц по осям x и y. Как только мы научились рисовать точки на координатной плоскости, мы можем перейти к графическому изображению более сложных объектов, поскольку график прямой или какой-либо другой функции — это просто визуальное представление всех точек или упорядоченных пар, составляющих функцию.

Пересечение по осям x и y

Пересечение по оси x — это точка, в которой график пересекает ось x. Точно так же точка пересечения с осью y — это точка, в которой график пересекает ось y. Координата y точки пересечения с осью x всегда равна 0, а координата x точки пересечения с осью y всегда равна 0.

При заданном уравнении подстановка x = 0 и решение для y даст точку пересечения по оси y и включение в y = 0, и решение для x даст пересечение x.

Пример:

Найти координаты точек пересечения x и y графика y = 2x + 6,


Подставьте y = 0 в y = 2x + 6:

0 = 2x + 6

-6 = 2x

x = -3

Точка пересечения с х находится в точке (-3, 0).

Подставьте x = 0 в уравнение:

y = 2 × 0 + 6

y = 6

Точка пересечения y находится в точке (0, 6).


оси x и y на графике

30-DAY PROMIS | ПОЛУЧИТЕ 100% ВОЗВРАТ ДЕНЕГ*

*T&C Apply

LearnPracticeDownload

Оси x и y — две важные линии, составляющие график. График состоит из горизонтальной оси и вертикальной оси, на которых могут быть представлены данные. Точка может быть описана по горизонтали или по вертикали, что легко понять с помощью графика. Эти горизонтальные и вертикальные линии или оси на графике являются осью x и осью y соответственно. В этом мини-уроке мы узнаем об осях x и y, а также о том, что такое оси x и y в геометрии, а также решим несколько примеров.

1. Определение осей X и Y
2. Уравнение для осей X и Y
3. Что важнее: ось X или ось Y?
4. Часто задаваемые вопросы по осям X и Y

Определение осей X и Y

Любая точка на координатной плоскости хорошо определяется упорядоченной парой, где упорядоченная пара записывается как (координата x,координата y) или (x,y), где координата x представляет точку на оси x или перпендикулярное расстояние от оси y, а координата y представляет собой точку на оси y или перпендикулярное расстояние от оси x. Оси X и Y — это оси, используемые в системах координат, образующих координатную плоскость. Горизонтальная ось представлена ​​осью x, а вертикальная ось представлена ​​осью y. Точка пересечения осей x и y называется началом координат и используется в качестве опорной точки для плоскости. Ось x также известна как абсцисса или график x, тогда как ось y также известна как график ординат или график y. На изображении ниже показано соответствующее представление.

Например: Население города с 2015 по 2020 год указано в таблице X и Y как:

Годы 2015 2016 2017 2018 2019 2020
Люди в миллионах 1 1,5 2 2,5 3 3,5

Чтобы найти любую точку на координатной плоскости, мы используем упорядоченную пару, где упорядоченная пара записывается как (x-координата, y-координата) или (x, y), где x-координата представляет точку на координатной плоскости. ось x или перпендикулярное расстояние от оси y, а координата y представляет собой точку на оси y или перпендикулярное расстояние от оси x, поэтому сверху ясно, что ось x идет первой при записи упорядоченной пары в найти точку. Здесь мы видим, что расположение каждой точки на графике отмечено как упорядоченная пара, где ось x или координата x опережает ось y или координату y. Затем представить эти точки на диаграмме x и y, используя годы на оси x и соответствующее население на оси y как:

Уравнение для осей X и Y

Рассмотрим линейное уравнение y = 2x+1. Теперь, чтобы построить график этого уравнения, постройте таблицу с двумя столбцами для значений x и y. Чтобы нарисовать график координат по осям X и Y линейного уравнения, нам нужно нарисовать таблицу сетки по осям X и Y как минимум для двух точек.

х г
0 1
1 3
2 5

Теперь нарисуйте точки на графике, где значения x лежат на оси x, а соответствующие значения y лежат на оси y. Затем соедините точки прямой линией, чтобы нарисовать график уравнения.

Что первично: ось X или ось Y?

Чтобы найти любую точку на координатной плоскости, мы используем упорядоченную пару, где упорядоченная пара записывается как (координата x,координата y) или (x,y), где координата x представляет точку на x- ось или перпендикулярное расстояние от оси y, а координата y представляет собой точку на оси y или перпендикулярное расстояние от оси x, поэтому сверху ясно, что ось x идет первой при записи упорядоченной пары для определения местоположения точка. Здесь мы видим, что расположение каждой точки на графике отмечено как упорядоченная пара, где ось x или координата x опережает ось y или координату y.

Важные примечания:

  • Ось x также называется абсциссой.
  • ось у также называется ординатой.
  • На оси x и оси y имеется бесконечное количество точек.
  • Начало — это точка пересечения осей X и Y.

Связанные темы

Ниже перечислены несколько интересных тем, связанных с осями x и y.

  • Введение в графику
  • Геометрия
  • Полярные координаты

 

Примеры осей X и Y

  1. Пример 1: Даниэль получил от учителя математическую задачу по осям X и Y, в которой он должен нанести точки (3,2) и (2,3) на график и провести линию, проходящую через эти точки . Можете ли вы определить точку пересечения с осью x?

    Решение: Точки можно нанести на график, как показано.

    Следовательно, линия пересекает ось x в точке (5,0).

  2. Пример 2: Нанесите точки (0,2), (0,4,5) и (0,-3) в системе координат. Все ли точки лежат на прямой? Можно ли назвать линию?

    Решение: Точки на графике показаны ниже.

    Ясно, что точки лежат на прямой оси y.

перейти к слайдуперейти к слайду

Отличное обучение в старшей школе с использованием простых подсказок

Увлекаясь зубрежкой, вы, скорее всего, забудете понятия. С Cuemath вы будете учиться визуально и будете удивлены результатами.

Записаться на бесплатный пробный урок

Практические вопросы по осям X и Y

 

перейти к слайдуперейти к слайду

Часто задаваемые вопросы по осям X и Y

Что такое 4 квадранта на графике?

Четыре квадранта или квадранты осей x и y следующие:

  • Квадрант 1: это положительная сторона обеих осей x и y.
  • Квадрант 2: это отрицательная сторона оси x и положительная сторона оси y.
  • Квадрант 3: это отрицательная сторона обеих осей x и y.
  • Квадрант 4: это отрицательная сторона оси Y и положительная сторона оси X.

Как нарисовать уравнение?

Чтобы построить уравнение, сначала создайте таблицу с двумя столбцами для значений x и y. Затем нарисуйте точки на графике, где значения x лежат на оси x, а соответствующие значения y лежат на оси y. Затем соедините точки, чтобы нарисовать график уравнения.

Дроби для 4 класса по математике: Понятие дроби — урок. Математика, 4 класс.

Дроби. Математика, 4 класс: уроки, тесты, задания.

  • Предметы
  • Математика
  • 4 класс
  1. Проценты

  2. Понятие дроби

  3. Сравниваем дроби

  4. Дроби.

    Нахождение части числа
  5. Дроби. Нахождение числа по его части

  6. Деление и дроби

  7. Нахождение части одного числа от другого

  8. Сложение дробей

  9. Вычитание дробей

  10. Правильные и неправильные дроби

  11. Правильные и неправильные части величин

  12. Задачи на части

  13. Смешанные числа.

    Выделение целой части из неправильной дроби
  14. Перевод смешанного числа в неправильную дробь

  15. Сложение и вычитание смешанных чисел

  16. Сложение с переходом через 1

  17. Вычитание с переходом через 1

  18. Свойства действий со смешанными числами.

    Решение задач

Отправить отзыв

УРОК МАТЕМАТИКИ В 4 КЛАССЕ ТЕМА: «ДРОБИ» | План-конспект урока по математике (4 класс):

УРОК МАТЕМАТИКИ В 4 «Д» КЛАССЕ

ТЕМА: «ДРОБИ»

Цель: сформировать представление о дробях

Предметные УУД: – ознакомление учащихся с предметным смыслом дроби и доли, с терминами “дробь”, “числитель”, “знаменатель”, с записью и чтением дробей;
– формирование навыков определения долей и дробей по предметным моделям.

Личностные УУД:.

– развивать учебно-познавательный интерес к новому учебному материалу и способам решения задач.

Метапредметные УУД:

*Регулятивные:

– принимать и сохранять учебную задачу;
– планировать свои действия в соответствии с поставленной задачей;
– оценивать правильность выполнения действия.

*Коммуникативные: 

– строить монологическое высказывание, овладевать диалогической формой коммуникации.

*Познавательные:.

– овладевать логическими действиями анализа, сравнения, синтеза и обобщения;
– осуществлять работу с графической информацией.

ХОД УРОКА:

I..ОРГ. МОМЕНТ.

 -Сегодня чудесный день. Улыбнитесь! Скажите друг другу добрые слова!

Один мудрец однажды сказал: « Не для школы, а для жизни мы учимся!» (римский философ Луций Анней Сенека)     СЛАЙД 2. 

-А для чего  вы изучаете такую сложную науку как математика?

(Высказывания детей)

-Сегодня мы продолжим исследовать и постигать тайны  этой науки, такой сложной, но очень интересной

II. Актуализация опорных знаний.

1.Математический диктант    СЛАЙД 3.

-а)Запишите одни ответы:

  • Найдите неизвестное число, зная, что ¼ его составляет 35.
  • Найдите 1/3 числа 240.
  • Найдите 1 % числа 26000;
  • Найдите неизвестное число, зная, что его 1% составляет 2. СЛАЙД 4.

(140, 80, 260, 200)

б) Расставьте полученные числа в порядке возрастания. СЛАЙД 5.

— Проверим. Что интересного вы заметили?

(80, 140, 200, 260. Все числа круглые, увеличиваются на 60)

— Какое число, по вашему мнению, «лишнее»?

(Например, 80-оно двузначное, а остальные трёхзначные; 200 – кратно 100, а остальные нет; 140 – сумма цифр нечётная, а остальные – чётная…)

-Какое число следующее? (320)

-Дайте характеристику числу 320.

-Как можно назвать все эти числа? (Натуральные числа — числа, возникающие естественным образом при счёте; все недробные числа, включая ноль; числа , которые делятся без остатка на единицу и самого себя?)

— Какие ещё числа, кроме натуральных, вы знаете? (дробные)

2. Повторение по теме «Доли»

— А сейчас посмотрите внимательно на следующие  фигуры (квадраты). СЛАЙД 6.

-Определите, какая часть квадрата закрашена?

 — Какое число следующее? Почему? (1/36, т. к. одну сторону квадрата разделить на 6 равных частей, получится 36 равных клеток)

III. Изучение нового материала

— А сейчас посмотрим следующие фигуры.

-Определите, какая часть квадрата закрашена. СЛАЙД 7.

(Если дают правильный ответ, попросить выйти к доске и написать, спросить, чем это задание отличается от предыдущего?)

(Ответы получатся разные или не получатся.)?!

–Почему затрудняетесь?  Чем это задание отличается от предыдущего?

(Там закрашена одна часть – доля, а здесь несколько.)

– Давайте  посмотрим внимательно, на сколько частей поделена фигура? (9)

– Сколько частей закрашено? (4)

– Как записать? ( 4/9 )      

– ! Кто знает, как называются такие числа? (дробные)

– Значит, как называется тема нашего сегодняшнего урока? СЛАЙД8.

(ТЕМА. Дроби.)

А сейчас прочитайте внимательно материал учебника в рамке на с. 79, чтобы ответь на мои вопросы.

Вопросы: (СЛАЙД 9)

  1. Скажите, что называется дробью?
  2. Как записывают дроби?  (Дроби записывают двумя НАТУРАЛЬНЫМИ ЧИСЛАМИ, РАЗДЕЛЁННЫМИ ЧЕРТОЙ) СЛАЙД 10.
  3. Как называется число, записанное над чертой? (числитель обозначается буквой m)
  4. Как называется число, записанное под чертой? (знаменатель обозначается буквой n)
  5. Что показывает знаменатель? (на сколько частей поделено число)
  6. Что показывает числитель? (сколько таких частей взято)

— Давайте посмотрим на запись на доске.

– Что показывает число 9? (На сколько частей поделено целое)

– Что показывает число 4? (Сколько таких частей взяли.)

– и т. д.

IV. Первичное закрепление    (СЛАЙД 11)

— Прочитай дроби.  Назови числитель, и знаменатель каждой  дроби  и объясни, что они обозначают. (фронтально, кто первым поднимет руку)

IV. Первичная проверка понимания учащимися нового учебного материала

 1)РАБОТА В ПАРАХ ( НА ЛИСТАХ)    СЛАЙД 12

Определите, какая часть фигуры закрашена. Напишите дробь рядом с фигурой.

Проверка по слайду. Пара показывает сигнал!. »    (СЛАЙД13)

Встаньте, у кого не было ни одной ошибки? У кого одна ошибка?. ..

V.Физминутка — Игра «Числитель-знаменатель»    (СЛАЙД14)

Если я показываю числитель — поднимаем руки вверх, тем самым показывая, что он пишется сверху, а если знаменатель — показываем, что он пишется внизу. Если показываю черту дроби, складываем руки горизонтально.

2)Индивидуальная работа

-Возьмите на парте карточку №2. (СЛАЙД 15)

Закрась указанные части фигур (простым карандашом)

— А теперь поменяйтесь листочками.

ВЗАИМОПРОВЕРКА ПО СЛАЙДУ. (СЛАЙД 16)

— За каждое правильно выполненное задание поставьте соседу «+»

 Если все задания выполнены , поставлено пять «+» –отметка«5», четыре плюса – «4», 3 плюса – 3 и тд.

— Кого сосед оценил на «5» и т.д. Вы работы сдадите, а я проверю.

  1. Решение задачи. 

— Используя материал новой темы, попробуем разобрать и решить задачу? (СЛАЙД 16)

Петя готовил уроки 2 часа. На математику он потратил  этого времени, а на географию оставшегося времени. Сколько минут Петя готовил уроки по математике и сколько по географии?

— Что известно в задаче?

         -Что нужно узнать в задаче?

 — Чтобы удобнее было сосчитать, переведём 2 часа в минуты. Сколько минут в двух часах? (120 минут)

— Что найдём сначала? (сколько времени Петя делал математику)

— Как найдём? (120 : 3 = 40 мин)

— Что узнаем дальше? (Сколько времени осталось)

-Как узнаем? (120-40= 80 мин)

— Сможем ли мы теперь узнать, сколько времени Петя потратил на географию? Как?  (80 : 4 = 20 мин)

-Запишите ответ. Ответ: 40 минут Петя делал математику и 20 минут географию.

VI. ИТОГ УРОКА:

_ Что нового узнали на уроке?

— — Что такое дробь?

Как записывают дробь?

— Что показывает знаменатель дроби?

— А что показывает числитель ?

— На следующем уроке мы с вами продолжим изучение дробей.

Оценивание: Сегодня на уроке хорошо работали…..

— На этом наш урок окончен. (СЛАЙД 17)

.

Резервный материал.

1.                                                                                           “Сказка про дробь”

— Ребята, сейчас мы с вами будем соавторами сказки. Я начну рассказывать, а вы будете изображать услышанное и должны будете её закончить. Но сначала выполним задание.

Жила была дробь. Она была очень важная и гордая. И были у неё 2 слуги…? Как вы думаете, как их звали? (Числитель и знаменатель). Эта дробь очень не любила знаменатель, постоянно помыкала им и унижала его. Знаменатель очень переживал это и становился всё меньше и меньше, а чем меньше он становился, тем доля, которую обозначала эта дробь становилась всё …(Больше и больше). Но однажды знаменатель не выдержал такой тяжёлой жизни и совсем исчез, т.е. превратился в … (0). Как вы думаете, что же дальше произошло с этой важной и гордой дробью? (Она тоже исчезла, т.к. знаменатель обозначает на сколько частей разделили целое, а деление на 0 невозможно)

— Сделайте из этой сказки вывод: математический и жизненный.

2. Самостоятельная работ (3-5 мин) см. листы приложения

Четвертый класс, стандарты дробей чисел и операций

Четвертый класс, стандарты математики четвертого класса, математика четвертого класса, навыки четвертого класса, математические стандарты Четвертый класс, стандарты дробей, стандарты дробей четвертого класса, стандарты чисел, стандарты чисел, стандарты операций

Математика для четвертого класса: числа и операции — дроби Стандарты

Расширить понимание эквивалентности дробей и их порядка.

  • 4.NF.1. Объясните, почему дробь a/b эквивалентна дроби (n × a)/(n × b), используя визуальные модели дробей, обращая внимание на то, как количество и размер частей различаются, даже если сами две дроби одинаковы. размер. Используйте этот принцип для распознавания и создания эквивалентных дробей.
  • 4.NF.2. Сравните две дроби с разными числителями и разными знаменателями, например, создав общие знаменатели или числители, или сравнив с эталонной дробью, такой как 1/2. Признайте, что сравнения допустимы только тогда, когда две дроби относятся к одному и тому же целому. Запишите результаты сравнений с помощью символов >, = или < и обоснуйте выводы, например, с помощью визуальной фракционной модели.

Создавайте дроби из единичных дробей, применяя и расширяя предыдущее понимание операций над целыми числами.

  • 4.NF.3. Под дробью a/b, где a > 1, понимается сумма дробей 1/b.
    • Понимать сложение и вычитание дробей как соединение и разделение частей, относящихся к одному и тому же целому.
    • Разложите дробь на сумму дробей с одинаковым знаменателем более чем одним способом, записывая каждое разложение уравнением. Обоснуйте разложения, например, с помощью визуальной дробной модели. Примеры: 3/8 = 1/8 + 1/8 + 1/8; 3/8 = 1/8 + 2/8; 2 1/8 = 1 + 1 + 1/8 = 8/8 + 8/8 + 1/8.
    • Складывать и вычитать смешанные числа с одинаковыми знаменателями, например, заменяя каждое смешанное число эквивалентной дробью и/или используя свойства операций и отношения между сложением и вычитанием.
    • Решите текстовые задачи на сложение и вычитание дробей, относящихся к одному и тому же целому и имеющих одинаковые знаменатели, например, с помощью визуальных моделей дробей и уравнений для представления задачи.
  • 4.NF.4. Применяйте и расширяйте прежнее понимание умножения, чтобы умножить дробь на целое число.
    • Понимать дробь a/b как кратное 1/b. Например, используйте модель визуальной дроби, чтобы представить 5/4 как произведение 5 × (1/4), записав заключение уравнением 5/4 = 5 × (1/4).
    • Понять кратное a/b как кратное 1/b и использовать это понимание для умножения дроби на целое число. Например, используйте модель визуальной дроби, чтобы выразить 3 × (2/5) как 6 × (1/5), распознав это произведение как 6/5. (В общем, n × (a/b) = (n × a)/b.)
    • Решите текстовые задачи на умножение дроби на целое число, например, используя визуальные модели дробей и уравнения для представления задачи. Например, если каждый человек на вечеринке съест 3/8 фунта ростбифа, а на вечеринке будет 5 человек, сколько фунтов ростбифа потребуется? Между какими двумя целыми числами лежит ваш ответ?

Понимание десятичной записи дробей и сравнение десятичных дробей.

  • 4.NF.5. Выразите дробь со знаменателем 10 в виде эквивалентной дроби со знаменателем 100 и используйте эту технику, чтобы сложить две дроби со знаменателями 10 и 100 соответственно. Например, выразите 3/10 как 30/100 и сложите 3/10 + 4/100. = 34/100.
  • 4.NF.6. Используйте десятичную запись для дробей со знаменателем 10 или 100. Например, перепишите 0,62 как 62/100; описать длину как 0,62 метра; Найдите 0,62 на диаграмме с числовыми линиями.
  • 4.NF.7. Сравните два десятичных знака с сотыми, рассуждая об их размере. Признайте, что сравнения действительны только тогда, когда два десятичных знака относятся к одному и тому же целому. Запишите результаты сравнений символами >, = или < и обоснуйте выводы, например, с помощью визуальной модели.

Эквивалент дробей, порядок и операции

Вы используете устаревший браузер , который не поддерживается. Пожалуйста, обновите свой браузер , чтобы улучшить ваш опыт.

Решать тесты по математике для 4 класса: Тесты по математике для 4 класса онлайн

Тесты по Математике для 4 класса

Тест: Деление и умножение многозначного числа на однозначное число

Тест можно использовать после прохождения тем умножения и деления многозначных чисел на однозначное.

Математика 4 класс | Автор: Жижилева Н.Ф. | ID: 17403 | Дата: 6.9.2022

Тест: Задачи на движение №3

Дорогой друг! Предлагаю тебе проверить хорошо ли ты умеешь решать задачи на движение. Читай внимательно задачи и думай! Желаю тебе удачи и хороших результатов!

Математика 4 класс | Автор: Новикова Лариса Александровна | ID: 16286 | Дата: 16.2.2022

Тест: Величины №3

Тест составлен в соответствии с требованиями Госстандарта РК и может быть использован на различных этапах контроля: входном, промежуточном, итоговом. Учащимся необходимо выбрать один верный ответ из нескольких предложенных вариатов.

Математика 4 класс | Автор: Зобнина Ирина Евгеньевна | ID: 16285 | Дата: 15.2.2022

Тест: Тренажер для подготовки к ВПР. Время

Тест содержит задания по теме «Время», входящее в перечень заданий ВПР под номером 4. Разработан для подготовки. Может использоваться как дома, так и в школе. В основе — сайт https://math5-vpr.sdamgia.ru/methodist. А также «Тренажер по математике для подготовки к ВПР» составитель А.Н.Алексеева

Математика 4 класс | Автор: Соловьев Павел Евгеньевич | ID: 16284 | Дата: 15.2.2022

Тест-тренажёр «Умножение и деление на однозначные числа»

Тест предназначен для контроля устных приемов умножения и деления многозначных чисел на однозначное число.Тест содержит задания на нахождение произведения, частного, значений выражения и задачи. В каждом задании выбирайте один верный ответ из четырех предложенных.

Математика 4 класс | Автор: Курова Татьяна Владимировна | ID: 16283 | Дата: 15.2.2022

Тест: Найдите значение выражения

Тест предназначен для подготовки учащися 4 классов к ВПР по математике. Способствует овладению умением вычислять значение числового выражения, соблюдая при этом порядок действий. Основан на выборе учащимися единичного варианта ответа. Составлен на основе типовых заданий, взятых из сборника «Тренажер для подготовки к ВПР по математике 4 класс» составитель А.Н.Алексеева.

Математика 4 класс | Автор: Соловьев Павел Евгеньевич | ID: 16281 | Дата: 15.2.2022

Тест: Обобщение за 3 четверть

Тест составлен в соответствии с требованиями Госстандарта РК и может быть использован на различных этапах контроля: входном, промежуточном, итоговом. Учащимся необходимо выбрать один верный ответ из нескольких предложенных вариатов.

Математика 4 класс | Автор: Зобнина Ирина Евгеньевна | ID: 16280 | Дата: 15.2.2022

Тест: Нахождение значения выражения

Тест содержит задания для подготовки к ВПР по математике в 4 классе. Задания входят в перечень основных заданий ВПР под номером 7. Материалы составлены на основе «Тренажера для подготовки к ВПР по математике» составитель А.Н.Алексеева. Обязателен выбор единичного варианта ответа. При выполнении действий необходимо повторить порядок действий в выражениях.

Математика 4 класс | Автор: Соловьев Павел Евгеньевич | ID: 16279 | Дата: 15. 2.2022

Тест: Разряды чисел

Выполни тест

Математика 4 класс | Автор: Федорова Мария Николаевна | ID: 16278 | Дата: 15.2.2022

Тест: Единицы измерения массы №2

Вам представлен тест из 10 вопросов по теме «Единицы измерения массы»

Математика 4 класс | Автор: Тимошенко Ксения Николаевна | ID: 16277 | Дата: 15.2.2022

Тест: Задачи на движение №3

Тест: Задачи на движение №3 — Математика 4 класс

Английский язык

Астрономия

Белорусский язык

Биология

География

ИЗО

Информатика

История

Итальянский язык

Краеведение

Литература

Математика

Музыка

Немецкий язык

ОБЖ

Обществознание

Окружающий мир

ОРКСЭ

Русский язык

Технология

Физика

Физкультура

Химия

Черчение

Для учителей

Дошкольникам

VIP — доступ

  • Предметы
  • »
  • Математика
  • »
  • 4 класс
  • »
  • Задачи на движение №3

Задачи на движение №3

Дорогой друг! Предлагаю тебе проверить хорошо ли ты умеешь решать задачи на движение. Читай внимательно задачи и думай! Желаю тебе удачи и хороших результатов!

Математика 4 класс | Автор: Новикова Лариса Александровна | ID: 16286 | Дата: 16.2.2022

+70 -31

Помещать страницу в закладки могут только зарегистрированные пользователи
Зарегистрироваться

Вопрос № 1

Сколько времени затратил велосипедист, если он проехал 60 км со скоростью 12 км/ч?

5 ч
6 ч
72 ч

Вопрос № 2

Слон со скоростью 6 км/ч прошёл 18 км. Сколько километров за это же время пробежит другой слон, если он будет двигаться со скоростью 14 км/ч?

48 км
42 км
3 ч

Вопрос № 3

До остановки поезд проехал 180 км со скоростью 60 км/ч. После остановки он прошёл со скоростью 70 км/ч ещё 280 км. За какое время поезд прошёл весь путь?

4 ч
7 ч
6 ч

Вопрос № 4

Какой путь проедет машина за 4 часа со скоростью 65км/ч?

260 км
250 км
130 км

Вопрос № 5

Автобус за 4 часа проехал 160 км. За какое время проедет это же расстояние автомобиль, если его скорость будет в 2 раза больше?

3 ч
4 ч
2 ч

Вопрос № 6

Расстояние между двумя городами 420 км. Половину этого расстояния машина прошла за 3 часа. С какой скоростью ехала машина?

60 км/ч
70 км/ч
130 км/ч

Вопрос № 7

С какой скоростью может передвигаться улитка ?

4 м/ч
40 км/ч
90 м/с

Вопрос № 8

За 1 минуту Кот в сапогах проходит 60 м. Сколько метров пройдёт он за 1 секунду?

6 км
6 м
1 м

Вопрос № 9

С какой скоростью нужно бежать антилопе гну, чтобы за 2 часа преодолеть расстояние 100 километров?

45 км/ч
60 км/ч
50 км/ч

Вопрос № 10

Бобр, плывя со средней скоростью 9 км/ч, преодолел 18 километров. Сколько времени он затратил?

3 ч
2 ч
1 ч

Вопрос № 11

Черепаха прошла 12 м со скоростью 6 м/мин. За это же время улитка проползла 30 см. С какой скоростью двигалась улитка?

12 м/мин
15 см/мин
15 м/с

Вопрос № 12

Мальчики до деревни прошли 20 км, двигаясь со скоростью 5 км/час, а обратно они ехали на велосипеде в 2 раза быстрее. За сколько часов они совершили обратный путь?

5 ч
3 ч
2 ч

Вопрос № 13

Ворона летела 3 ч со скоростью 50 км/час. Скворец такое же расстояние пролетит за 2 ч. С какой скоростью летит скворец?

65 км/ч
70 км/ч
75 км/ч

Показать ответы

Получение сертификата
о прохождении теста

Доступно только зарегистрированным пользователям

© TestEdu.ru 2013-2022

E-mail администратора: [email protected]

Тест по математике для 4-го класса

Прежде чем начать, распечатайте тест по математике для 4-го класса. Постарайтесь ответить на все вопросы.

ФИО ________________ Дата:_____________________

Решить следующие задачи

 1.

а. Какая дробь не равна двум другим? ___________ 


2. У Джетсера дела идут плохо. В пятницу он заработал 468 долларов. В субботу он заработал 459 долларов. Во вторник он заработал 432 доллара. Сколько денег он заработал в четверг?

A. 424      B. 414     C. 314     D. 451       

3. Посмотрите на приведенные ниже блоки с основанием 10 и скажите, какое число они представляют. Внимательно посмотрите, прежде чем отвечать!

Основание 10 блоков

A. 363      B. 164     C. 451       D. 300       


4. Округлить 846 до ближайшей сотни _____________ и 3756 до ближайшей тысячи _______

Какое число

то же, что пять тысяча двадцать два?

A. 5200      B. 502      C. 5202      D. 5022      


6. Запишите десятичные числа 0,25 и 0,4 прописью и дробями 5678, но меньше 5708?

A. 5677      B. 5707      C. 5800      D. 0      


8. Каково значение символа # ?________________

6 + # = 14 


9. Сколько блоков из 100 и 10 можно сделать из числа 3250?

A. 325 и 1      B. 32 и 5      C. 30 и 50      D. 3 и 55      


10,1 квартер, 3 десятицентовых монеты, 2 пятицентовых монеты, а также 6 центов 3 9000. .70      Б. 0,71       C. 0,75        D. 0,69 $     


11. Умножить 235 на 13 ________________ 

A. 4322       


12. Какая дробь больше 2/3?

A. 1/3       B. 4/5      C. 4/6        D. 2/6      


13. Найдите 4 задачи на умножение, которые совпадают с 12 + 12 + 12

2 9. Оттенок 4. Фракция составляет


A. Двух девятых B. Девять три C. четыре восьмых D. Три девятых

Три девятых

15. Выполните следующие дополнения и вычитания

64837 + 4598 =?

92345 — 91422 = ?

16. 

Как лучше всего назвать геометрическую фигуру, похожую на мусорный бак? ______________________

Какую из следующих форм вы, вероятно, могли бы использовать для описания формы вашего глазного яблока?

A. Куб     B. Сфера       C. Цилиндр       D. Конус      


Какую из следующих фигур вы, вероятно, могли бы использовать для описания формы холодильника в вашем доме?

A. Куб       B. Сфера       C. Цилиндр       D. Прямоугольная призма      


17. Предположим, у вас в кармане 3 четверти, 5 десятицентовиков, 2 пенни и 1 никель. Вы говорите другу выбрать только одну монету. Какую монету ваш друг, скорее всего, выберет? Кратко объясните, почему


18. Запишите следующие числа прописью


5669 ___________________________________________

8765 ___________________________________________


1

20. Какие два кружка при сложении дадут вам целое?

Модель дробей


Вариант №1: A и C Вариант №2: B и C Вариант №3: A и B Вариант №4: B и D 


21. Нарисуйте круг. Разделите круг на 4 равные части. Затем заштрихуйте две четверти круга.

22. Сделайте два круга. Разделите оба круга на 4 равные части. Затем заштрихуйте три четверти первого круга и одну четверть второго круга

а. Сколько будет три четверти минус одна четвертая? _______________________

б. Сколько будет три четверти плюс одна четвертая?_________________________

23. Замените символы * и # числами, чтобы сбалансировать уравнение

6 + * = 10 + # + # 

то же самое, что и 40+(100+300)? ______________

А. 140 + 300 Б. 40 + 500 В. 40 × 11 Г. 40 × 100 + ( 30 )      

25. Навыки решения проблем. Нет калькулятора. Не исключение! На самом деле, для решения этого теста по математике для 4-го класса вообще не следует использовать калькулятор.

У тебя 6700 долларов в кармане. Вы хотите купить 7 ноутбуков стоимостью 800 долларов каждый.

Сколько будут стоить 7 ноутбуков?_________

Объясните, как вы использовали таблицу умножения и математические факты, чтобы очень быстро получить ответ

Сколько денег у вас осталось после оплаты калькуляторов?_________

Объясните еще раз, как вы использовали математические факты, чтобы получить ответ очень быстро

Вы хотите разделить оставшиеся деньги поровну между вами и 10 друзьями. Сколько денег получит каждый человек?____________ 


26. Если 5 книг стоят 100 долларов, какова цена 2 книг? (Предположим, вы купили 5 одинаковых книг)

27. Площадь фигуры ниже равна

A. 14 квадратных единиц       B. 17 квадратных единиц       C. 10 квадратных единиц × 11     D. 16 квадратных единиц     

28. В В пространстве внизу нарисуйте 2 параллельные линии и третью линию, перпендикулярную параллельным линиям 

29. Нарисуйте 2 конгруэнтные фигуры и 2 одинаковые фигуры в пространстве ниже:

30. Дополните следующие 2 числа

4, 20, 100, ________, ___________

Что нужно знать об этом тесте по математике для 4-го класса

тест по математике является хорошим показателем того, что большинство навыков, преподаваемых в 4-м классе, были освоены.

Если у вас возникли трудности с этим тестом по математике в 4-м классе, попросите кого-нибудь помочь вам.

Хотите решение этого теста? Добавьте в корзину и купите Подробное РЕШЕНИЕ НА 15 СТРАНИЦАХ и ПРЕВОСХОДНЫЕ ОБЪЯСНЕНИЯ с помощью PayPal.

Я изо всех сил старался сделать этот тест по математике для 4-го класса в соответствии с национальными стандартами

Чтобы распечатать этот тест по математике для 4-го класса, нажмите здесь

Треугольник 30-60-90

3 апреля, 23 17:08

Что такое треугольник 30-60-90? Определение, доказательство, площадь и простые примеры из реальной жизни.

Подробнее

  • Расчет условной вероятности с использованием таблицы непредвиденных обстоятельств

    29, 23 марта 10:19

    Научитесь рассчитывать условную вероятность с помощью таблицы непредвиденных обстоятельств. Эта таблица непредвиденных обстоятельств может помочь вам разобраться быстро и безболезненно.

    Подробнее

  • БЕСПЛАТНЫЙ практический тест по математике MAP для 4-го класса

    Добро пожаловать на наш БЕСПЛАТНЫЙ практический тест по математике MAP для 4-го класса с ключом ответа и объяснением ответа. Реалистичный формат этого практического теста и высококачественные практические вопросы помогут вашему ученику успешно сдать тест MAP по математике в 4-м классе. Тест не только точно соответствует тому, что учащиеся увидят на реальной карте, но и содержит подробные объяснения ответов.

    Для этого практического теста мы выбрали 20 реальных вопросов из прошлых экзаменов для практического теста MAP вашего учащегося. У вашего ученика будет возможность попробовать самые распространенные вопросы по математике MAP 4-го класса. К каждому вопросу есть подробное объяснение того, как решить вопрос и как избежать ошибок в следующий раз.

    Воспользуйтесь нашими бесплатными практическими тестами по математике для 4-го класса и учебными материалами (обновленными в 2021 г.), чтобы помочь своим учащимся сдать тест по математике для 4-го класса! Не забудьте перейти по некоторым связанным ссылкам в нижней части этого поста, чтобы получить лучшее представление о том, какие вопросы по математике нужно практиковать учащимся.

    Абсолютно лучшая книга

    для получения высшего балла в 4-м классе MAP Математика Тест

    10 Образец

    MAP для 4-го класса Практические вопросы по математике

    1- У Джейми 6 четвертаков, 9 пенни и 11 десятицентовиков. Сколько денег у Джейми?

    A. 150 пенни

    B. 240 пенни

    C. 251 пенни

    D. 281 пенни

    2- Джеб заплатил 72 доллара за подписку на журнал. Если он платит 4 доллара за каждый номер журнала, сколько номеров журнала он получит?

    A. 18

    B. 20

    C. 22

    D. 24

    3- Каков периметр треугольника?

    A. 27 дюймов

    B. 31 дюйм

    C. 43 дюйма

    D. 192 дюйма

    4- На рисунке ниже показана схема читального зала.

    A. 6 футов

    B. 12 футов

    C. 20 футов

    D. 50 футов

    5- Какой треугольник имеет один тупой угол?

    А.

    Б.

    В.

    D.

    6- Здание высотой 36 футов. Какова высота здания в метрах?

    A. 1 ярд

    B. 3 ярда

    C. 12 ярдов

    D. 108 ярдов

    7- Сумма A и B равна 35. Если A \(= 16\), какое уравнение может быть используется для нахождения значения B?

    А. \(В – 16 = 35\)

    В. \(В + 16 = 35\)

    В. \(А + 16 = 35\)

    Г. \(А – 16 = 35 \)

    8- Какое число представлено \(A\)?
    \(9 × А = 108\) 9\circ\)F

    12- Число 47,06 можно представить как __ _______

    A. \((4 × 10) + (7 × 1) + (6 × 0,01)\)

    B \((4 × 10) + (7 × 1) + (6 × 0,1)\)

    C. \((4 × 1) + (7 × 1) + (0 × 1) + (6 × 1) )\)

    D. \((4 × 10) + (7 × 1) + (0 × 10) + (6 × 100)\)

    13- В году 365 дней, а в году 24 часа. день. Сколько часов в году?

    А. 2190

    Б. 7440

    В. 7679

    Д. 8760

    14- В субботу Лили была судьей на 3 футбольных матчах. Она прибыла на футбольное поле за 15 минут до первой игры. Каждая игра длилась \(\frac{11}{2}\) часов. Между играми было 5 минут. Лили ушла через 10 минут после последней игры. Сколько времени в минутах провела Лили на футбольном поле?

    A. 300 минут

    B. 305 минут

    C. 480 минут

    D. 485 минут

    15- Какая фигура представляет собой линию симметрии?

    А.

    B.

    C.

    D.

    16- Какая дробь имеет наименьшее значение?

    A. \(\frac{1}{2}\)

    B. \(\frac{3}{8}\)

    C. \(\frac{3}{4}\)

    D. \(\frac{9}{16}\)

    17- У Лизы 336 пастилок. Она хочет положить их в коробки по 12 пастилок. Сколько ящиков ему нужно?

    A. 20

    B. 22

    C. 24

    D. 28

    18- Каков объем куба?

    19- На какое число указывает стрелка?

    A. 26

    B. 28

    C. 30

    D. 33

    20- Какое смешанное число показано заштрихованными прямоугольниками?

    A. \(3\frac{1}{2}\)

    B. \(4\frac{1}{2}\)

    C. 3

    D. 4

    Лучший

    4-й КАРТА Класса Ресурс для подготовки к математике на 2021 год

    Ответы:

    1- C
    \(6\) четвертаков \(= 6 × 25\) пенни \(=150\) пенни
    \(9\) десятицентовиков \ (= 9 × 10 \)пенни \(= 90\) пенни
    Всего у Николь 251 пенни

    2- A
    \(1\) выпуск\(= $4\)
    Сколько выпусков? \(= $72\)
    \($72 ÷ $4= 18\) issue

    3- C
    Используйте формулу периметра треугольника. \circ\), треугольник не может иметь более одного тупого угла. 9\circ\)f), близкое к выбору B

    12- A
    \((4 × 10) + (7 × 1) + (6 × 0,01) = 40 + 7 + 0,06 = 47,06\)

    13- D
    1 год \(=\) 365 дней, 1 день \(=\) 24 часа
    1 год \(=\) 365 × 24
    1 год \(=\) 8 760

    14- B
    Каждая игра = \(\frac{11}{2}\) часов = 90 минут, поэтому 3 игры = \(\frac{41}{2}\) часов = 270 минут
    5 минут между каждой игрой, поэтому между 3 играми всего 10 минут.
    Она приходит за 15 минут до первой игры и уходит через 10 минут после последней игры.
    Всего она провела \(270+10+15+10=305\) минут на футбольном поле.

    15- A
    Вы можете определить, имеет ли фигура линию симметрии, сложив ее. Когда сложенная часть идеально лежит сверху (все края совпадают), линия сгиба является линией симметрии. Здесь первая фигура показывает линию симметрии.

    16- B
    Найдите наименьший общий знаменатель (НОД), а затем запишите каждый член как эквивалент дроби с помощью НКО. Затем мы сравниваем числители каждой дроби и располагаем их в правильном порядке от меньшего к большему или от большего к меньшему. 93 =64\)

    19- C
    Стрелка показывает ровно середину двух чисел 25 и 35, поэтому ответ равен 30

    20- A
    Эта фигура показывает 3 полных заштрихованных треугольника с половиной треугольник, равный \(3\frac{1}{2}\)

    Ищете лучший ресурс, который поможет вам успешно сдать тест MAP по математике в 4-м классе?

    Лучшие книги

    , чтобы стать лучшим Карта четвертого класса Математика Тест

    Реза

    Реза — опытный преподаватель математики и эксперт по подготовке к экзаменам, который обучает студентов с 2008 года. Он помог многим учащимся поднять свои баллы по стандартным тестам и поступить в колледжи своей мечты. Он работает со студентами индивидуально и в группах, ведет как живые, так и онлайн-курсы по математике, а также математическую часть стандартизированных тестов.

    Задачи простые по математике: 9 простых задач на математику

    5 логико-математических задач, которые сложно решить в уме

    А давайте отвлечёмся от кода и перещёлкаем 5 логико-математических задач. Попробуйте решить их в уме и напишите свои ответы в комментариях.

    1. Возраст мальчика

    Обилечивая человека, кондуктор поинтересовался, сколько лет его сыну. Человек ответил уклончиво:

    – Моя дочь в пять раз младше моего сына, а моя жена — в 5 раз его старше. Я, в свою очередь, вдвое старше своей жены. Моя мама сегодня отмечает день рождения — ей исполнился 81 год — столько, сколько мне, жене, дочери и сыну вместе взятым.

    Так сколько же лет мальчику?

    Решение

    х + 5х + 25х + 50х = 81
    
    81х = 81
    
    х=1

    Получается, что дочери один год, тогда мальчику 1 * 5 = 5 лет.

    2. Вёдра с водой

    Стоит два ведра ёмкостью 5 л и 9 л. Из реки необходимо набрать 3 литра воды. Как это сделать, если в распоряжении есть только эти два ведра?

    Решение

    Сначала заполним водой из реки девятилитровое ведро, и выльем из него воду в пятилитровое. Выходит, что в девятилитровом останется 4 литра. Выливаем всё из пятилитрового обратно в реку и переливаем в него из девятилитрового оставшиеся 4 литра. Снова наполняем водой из реки девятилитровое ведро и доливаем в меньшее литр воды. Итого в большом ведре остаётся 8 литров. Из меньшего выливаем всю воду обратно в реку и переливаем из девятилитрового в пятилитровое 5 л, после чего в большом ведре останется как раз 3 л воды.

    3. Лампочки и переключатели

    Есть две комнаты с низкими потолками. В первой висит три лампы накаливания, а в другой установлено три переключателя. Можно сколько угодно раз щёлкать переключатели, но в комнату с лампочками разрешено перейти только один раз.

    Как узнать, к какому переключателю подсоединена каждая из лампочек?

    Решение

    В условии сказано, что комнаты с низкими потолками, а перед нами лампы накаливания — то есть они нагреваются. Нам достаточно включить любую из них на некоторое время, затем выключить её и включить любую другую. После этого переходим в комнату с лампочками:

    • выключенная тёплая соединена с первым переключателем;
    • горящая лампочка связана со вторым;
    • та лампочка, которая не горит, соединена с выключателем, который мы не трогали.

    4. Время по верёвкам

    А как насчёт такой логико-математической задачи? Предположим, у нас есть две верёвки и бесконечное множество спичек. Каждая из этих верёвок сгорает за один час. Но вот беда — горят они неравномерно, поэтому невозможно узнать наверняка, за какое время сгорит какая-то часть веревки.

    Можно ли отмерить этими двумя верёвками 45 минут, и если да, то как это сделать?

    Решение

    Отмерить можно. Пусть верёвки и горят неравномерно, но сгорают они точно за 1 час. В этом случае можно:

    1. Поджечь одну верёвку с двух концов.
    2. На второй верёвке поджечь только 1 конец.
    3. Первая верёвка сгорит за 30 минут, и в этот момент поджигаем второй конец второй верёвки: на это уйдут оставшиеся 15 минут.

    5. Баночки с таблетками

    Есть двадцать баночек с таблетками. Почти во всех таблетки весят по 1 г, и только в одной — по 1,1 г. У нас есть точные кухонные весы, с помощью которых нужно определить баночку, каждая таблетка которой весит 1,1 г. Как это сделать, если можно взвесить только 1 раз?

    Решение

    Представим, что у нас 2 баночки, в одной из которых таблетки более тяжёлые. Даже если мы поставим их обе на весы, мы ничего не узнаем. Но если мы достанем из одной баночки одну таблетку, а также одну таблетку из другой, и положим их на весы — вот тогда-то и откроется истина. В данном случае вес будет 2,1 г или 2 г (в зависимости от того, какие по весу таблетки мы взяли). Так и определяем нашу баночку.

    Вернёмся к задаче. Из каждой баночки нужно доставать разное количество таблеток. То есть из первой баночки 1 таблетку, из второй — 2, из третьей — 3 и так далее. Если бы каждая таблетка весила по 1 г, общий вес составил бы 210 г. Но поскольку в одной из баночек таблетки тяжелее, вес будет больше. Для определения нужной баночки просто воспользуемся формулой:

    № тяжелой баночки = (вес - 210) * 10

    Понравилось решать логико-математические задачи? Тогда вас могут заинтересовать хитрые задания на логику с собеседований.

    5 самых старых нерешенных задач Математики о простых числах / Хабр

    Математика была предметом, который веками бросал вызов величайшим умам в истории человечества. Пожалуй, одной из наиболее исследуемых областей Математики является изучение простых чисел.

    Наши размышления о закономерностях в простых числах привели к некоторым сложнейшим проблемам, нерешенным даже величайшими математическими гениями. Сегодня мы рассмотрим 5 старейших математических задач о простых числах, которые интуитивно понятны старшекласснику, но все еще не доказаны даже после упорных попыток в течение 500-2000 лет.

    1. Совершенные числа: существуют ли нечетные совершенные числа? Бесконечны ли четные совершенные числа?

    Рассмотрим числа 6, 28, 496, 8128…

    Что в них особенного? Если вы не знаете, то я бы посоветовал сделать небольшую паузу и попытаться найти красивое свойство, которым обладают эти числа.  

    Двигаемся дальше….

    Если посмотреть на собственные делители этих чисел, то нетрудно заметить то самое «красивое» свойство:

    Числа, для которых сумма собственных делителей равна самому числу, называются совершенными числами. Самое раннее исследование совершенных чисел затеряно в истории. Однако, мы знаем, что пифагорейцы 525годдон.э. изучали совершенные числа. 

    Что мы знаем о таких числах?
    • Евклид доказал, что для данного n, если — простое число, то — совершенное число. В качестве упражнения попробуйте доказать это самостоятельно.

    Окей, краткий экскурс.

    Простые числа Мерсенна: простые числа вида для некоторого n. Мерсенн предположил, что все числа вида простые, когда n простое. (Мы знаем, что это неправда. Например, ).

    Открытый вопрос: существует ли бесконечно много простых чисел Мерсенна? На данный момент нам известно 47 простых чисел Мерсенна. 

    Как видите, мы знаем о четных совершенных числах и способах их получения еще со времен Евклида около300годдон. э.. Но нам неизвестно, существую ли нечетные совершенные числа!!! насамомделе,прогрессврешенииэтойпроблемыпрактическиотсутствует.

    Подводя итог, можно сказать, что изучение совершенных чисел ставит две давние открытые проблемы, а именно «существование нечетных совершенных чисел» и «существование бесконечно большого числа простых чисел Мерсенна».

    Евклид (ок. 300 г. до. н. э.) первым доказал то, что простых чисел бесконечно много.

    2. Гипотеза о близнецах: простых чисел-близнецов бесконечно много

    Простые числа-близнецы — это пара вида (p, p + 2), где p и p + 2 являются простыми числами.

    Точное происхождение гипотезы о простых числах-близнецах не установлено. Первая формулировка гипотезы о простых числах-близнецах была дана в 1846 году французским математиком Альфонсом де Полиньяком. Однако греческий математик Евклид дал старейшее из известных доказательств существования бесконечного числа простых чисел. Но он не предполагал, что существует бесконечное число простых чисел-близнецов.

    На протяжении 2000 лет в доказательстве этого утверждения практически не было прогресса. 

    Что мы знаем!
    1. Существует бесконечно много простых пар вида (p, p + k), где k <= 246.

    2. Если допустить истинность гипотезы Эллиота — Халберстама (которая, по нашему мнению, верна), то существует бесконечно много простых пар вида (p, p + k), где k <= 6. Это означает, что множество пар простых чисел, отличающихся на 2 (twin-primes), на 4 (cousin-primes) и на 6 (sexy-primes) бесконечно.

    Возможно, величайший из ныне живущих математиков, Теренс Тао, активно работает над этой проблемой. Посмотрите это видео, чтобы познакомиться с этим математическим гением и его работой над простыми числами-близнецами. 

    3. Какие правильные n-угольники построимы?

    Правильный многоугольник считается построимым, если его можно построить с помощью линейки и циркуля. Например, правильный пятиугольник можно построить с помощью линейки и циркуля, а правильный семиугольник нет.

    Древние греки знали, как построить правильный многоугольник с 3, 4 и 5 сторонами. Также они умели строить правильные многоугольники с удвоенным числом сторон для данного правильного многоугольника. 

    Таким образом, они могли построить правильный n-угольник для n = {3, 6, 12, 24… 4, 8, 16… 5, 10, 20…} и так далее.

    Естественно задать вопрос, для каких значений n можно построить правильный многоугольник. Первый реальный результат в решении этой проблемы был получен спустя 2000 лет после того, как древние греки впервые начали её изучать. В 1796 году 19-летний подросток построил правильный 17-угольник. Этим ребенком был не кто иной, как Карл Фридрих Гаусс. Несколько лет спустя Гаусс дал ответ на общую проблему.

    Что мы знаем!

    Гаусс показал, что правильный n-угольник может быть построен с помощью циркуля и линейки тогда и только тогда, когда n является произведением степени двойки и любого количества различных простых чисел Ферма (включая ни одного).

    Простое число Ферма — это простое число вида:

    Таким образом, проблема поиска всех построимых многоугольников сводится к нахождению всех простых чисел Ферма. Это отдельная нерешенная проблема. Несколько первых чисел Ферма: 3, 5, 17, 257, 65537, 4294967297…

    По состоянию на 2021 год единственными известными простыми числами Ферма являются F0=3, F1=5, F2=17, F3=257, F4=65537.

    Ферма предположил, что все числа Ферма являются простыми. В 1732 году Эйлер открыл, что F5 делится на 641. С тех пор мы выяснили, что для n = 5, 6…31 числа Ферма составные. Простое число Ферма после F4 неизвестно.

    Мы найдем ответ на вопрос о построимых правильных n-угольниках в тот же момент, как только найдем ответ на вопрос о существовании простых чисел Ферма.

    4. Гипотеза Гольдбаха (1742)

    Сильная гипотеза Гольдбаха:

    Каждое чётное число, большее двух, можно представить в виде суммы двух простых чисел.

    Слабая гипотеза Гольдбаха:

    Каждое нечётное число, большее 5, можно представить в виде суммы трёх простых чисел.

    Второе утверждение называется «слабым», потому что в случае истинности «сильной» гипотезы вторая также будет истинной. К сожалению, после значительных усилий поколений математиков, начиная с Эйлера, мы так и не смогли доказать ее.

    (Примечание — В 2013 году Харальд Хельфготт опубликовал доказательство слабой гипотезы Гольдбаха. По состоянию на 2018 год доказательство широко принято в математическом сообществе, однако оно еще не было опубликовано в рецензируемом журнале).

    В любом случае, все ждут доказательства сильной гипотезы.

    Что мы знаем!
    1. В 1930 году было доказано, что любое натуральное число больше 1 может быть записано в виде суммы не более чем C простых чисел, где C < 800 000 [Примечание — мы хотим, чтобы C = 2].

    2. В последнее десятилетие было показано, что каждое четное число n >= 4 на самом деле является суммой не более чем 6 простых чисел (т. е. С <= 6). Позже результат был улучшен до C <= 4.

    Забавный факт — гипотеза Гольдбаха является частью сюжета испанского фильма 2007 года «Западня Ферма«.

    Отказ от ответственности: название статьи вводит в заблуждение. После рассказа о 4 нерешенных задачах я хотел бы показать одну математическую проблему (пятая проблема), которая была недавно решена (в 2004 году).

    5. Тест простоты числа принадлежит классу P (2004)

    Допустим, вам дано число n = 10089886811898868001. Вас спрашивают, простое ли это число. Первое, что вам приходит на ум, так это, 

    Алгоритм A — проверить для каждого числа делится ли n на k. Вы можете оптимизировать этот алгоритм, понимая, что если n не является простым, то n будет иметь такой множитель k, что

    Алгоритм B — итак, вы проверяется только

    Хорошо, но погодите, что такое «P»?

    Говорят, что задача находится в «P», если существует «быстрый» алгоритм, который может решить задачу. В нашем случае задача заключается в том, чтобы определить, является ли заданное n простым числом.

    Итак, что такое быстрый алгоритм?

    Для любой заданной проблемы у нас имеется размер ввода (назовем его x). Для нашей задачи размер ввода — это количество цифр в числе n. Итак, x = 20 для указанного выше n. В общем случаем, при заданном n,

    Алгоритм называется быстрым (алгоритм с полиномиальным временем), если он решает задачу за f(x) шагов, где f — полиномиальная функция. 

    Если взглянуть на вышеупомянутые алгоритмы, то получим, что мы имеем n шагов в алгоритме А и шагов в алгоритме B. 

    Итак, размер ввода в нашем случае —

    Обозначим — количество шагов в алгоритме для данного размера ввода x.

    Для алгоритма А,

    Для алгоритма B,

    В обоих случаях алгоритмы имеют экспоненциальное время. В течение 400 лет математики пытались выяснить, можно ли решить задачу определения простоты числа за полиномиальное время. Оказывается, что да. Новость об этом распространилась в математическом сообщество (особенно среди теоретиков чисел) в 2004 году, когда об этом объявили профессор и двое его студентов из IITK.

    Алгоритм (известный как тест простоты AKS) был опубликован в статье под названием «Primes Is In P«, где показывается, что задача (независимо от того, является ли n простым или нет), может быть решена за ~ шагов. Позже были внесены некоторые улучшения, сократившие время до ~ шагов, также выдвигались предположения, что время можно уменьшить и вовсе до ~ шагов (прим. переводчика — предположение оказалось ложным).


    Дата-центр ITSOFT — размещение и аренда серверов и стоек в двух дата-центрах в Москве. За последние годы UPTIME 100%. Размещение GPU-ферм и ASIC-майнеров, аренда GPU-серверов, лицензии связи, SSL-сертификаты, администрирование серверов и поддержка сайтов.

    Ознакомьтесь с этими 50 задачами дня по математике для детского сада

    Начните свой ежедневный урок математики со словесной задачи дня — это отличный способ подготовить почву для обучения! Включите их в начале своего математического блока, чтобы укрепить уверенность, навыки критического мышления и обучающееся сообщество. Студенты привыкнут читать по смыслу, а также определять ключевую информацию. Предложите учащимся записывать уравнения и рисовать картинки, чтобы объяснить свое мышление, так как это помогает им увидеть свет, когда они застряли!

    Темы в этих математических задачах для детского сада включают сложение, вычитание, сравнение, чувство числа, сравнение чисел и измерение. Хотите весь этот набор задач по математике для детского сада в одном простом документе? Получите бесплатный пакет PowerPoint, отправив сообщение электронной почты здесь. Все, что вам нужно сделать, это опубликовать одну из задач на доске или экране проектора. Тогда пусть дети взять его оттуда.

    1. У Сью 2 маркера. У Тома 3 маркера. Сколько маркеров у них всех вместе?

    2. На ковре 4 красных блока. На ковре 4 желтых блока. Сколько всего кубиков на ковре?

    3. У Сэма 3 шарика. Тим дал Сэму 1 шарик. Сколько шариков сейчас у Сэма?

    4. Одна кошка во дворе. Во двор заходят еще 4 кошки.

    Сколько кошек сейчас во дворе?

    5. У Джилл было 7 игрушечных машинок. Она получила еще 3 на свой день рождения. Сколько сейчас игрушечных машинок у Джилл?

    6. У Авы на 2 книги больше, чем у Люси. У Люси 6 книг. Сколько книг у Авы?

    7. За столом стоят 7 красных стульев и 2 зеленых стула. Сколько стульев стоит за столом вместе?

    8. Пит положил 2 карандаша в свой стол. Там уже было 3 карандаша. Сколько карандашей сейчас в столе Пита?

    9. У Тины 4 куртки дома и 2 куртки в школе. Сколько всего курток у Тины?

    10. У Боба на 5 марок больше, чем у Билла. На купюре 2 марки. Сколько марок у Боба?

    11. У Джеймса 5 цветов. Он дарит Лизе 3 цветка. Сколько цветов осталось у Джеймса?

    12. На детской площадке было 8 птиц. Улетело 5 птиц. Сколько птиц осталось на детской площадке?

    13. У Карен было 4 арахиса. Она съела 3 из них. Сколько арахиса осталось у Карен?

    14.

    На грузовике 6 ящиков. 4 коробки синие. Остальные зеленые. Сколько ящиков зеленых?

    15. У Фрэнка 7 пончиков. Если он отдаст 3, сколько пончиков у него останется?

    16. Дома у Тима было 5 мячей. Он взял 1 мяч в школу. Сколько мячей он оставил дома?

    17. В миске 10 фруктов. 2 — яблоки. Остальные фрукты — апельсины. Сколько апельсинов в тарелке?

    18. У Крис 5 наклеек. Она дает Дэйву 4 свои наклейки. Сколько наклеек осталось у Крис?

    19. Лиза нарисовала 4 картинки. Келли нарисовала 6 картинок. Кто нарисовал больше картинок? На сколько больше?

    20. У Малика 10 глиняных шариков. Он дает 10 глиняных шаров Робу. Сколько глиняных шариков сейчас у Малика?

    21. Сара хочет сделать 8 карт. Она уже сделала 4 из них. Сколько еще карт нужно сделать Саре?

    22. Класс исполнил 5 песен. Они хотят спеть 10 песен. Сколько еще песен хочет спеть класс?

    23.

    На браслете Розы были бусы. Достала еще 3 бусины и надела их. Теперь у нее 6 бусинок вместе. Сколько бусинок было у Розы вначале?

    24. У мистера Джонса было 4 ручки. Он купил еще несколько ручек. Теперь у него 6 ручек. Сколько ручек он купил?

    25. У Лилли 3 розовые шляпы, 2 белые шляпы и 1 фиолетовая шляпа. Сколько всего шляп у Лилли?

    26. У Пэт есть 4 ластика. У Кена есть 2 ластика. У Джейсона есть 2 ластика. Сколько ластиков у них всего вместе?

    27. На скотном дворе было несколько животных. Было 5 коз, 4 коровы и 1 овца. Сколько всего животных было на скотном дворе?

    28. На Южной улице есть несколько домов. 3 дома синие, 1 дом серый и 4 дома белые. Сколько домов на Южной улице?

    29. У Мэг есть кошка, 2 собаки и попугай. Сколько домашних животных у Мэг?

    30. Кен любит сажать деревья. Он посадил 7 сосен, 2 дуба и 0 елей. Сколько деревьев Кен посадил вместе?

    31.

    Мисс Мато дала своему классу эту схему и попросила закончить ее: 3, 4, 5, 6, __, __. Какими должны быть последние две цифры?

    32. У Яна есть рисунок на рубашке. Это была синяя полоса, красная полоса, синяя полоса, красная полоса. Какого цвета следующая полоса?

    33. Начался обратный отсчет. Класс крикнул: «10, 9, 8, 7, 6…» Какие следующие два числа они назвали?

    34. Джоан считала свои носки по два. У нее было 4 пары носков. Сколько всего носков вместе?

    35. На шляпе Фейт есть буквенный узор. Это идет А, В, С, А, В, С. Какая буква стоит после Б в шаблоне?

    36. У Ларри 12 марок. У Барри 11 марок. У кого больше марок?

    37. Было 20 желтых рюкзаков. Было 19 синих рюкзаков. Какого цвета было большинство рюкзаков?

    38. У Джессики были мелки. У Пэм было 15 мелков. У Джессики было на 2 мелка больше, чем у Пэм. Сколько мелков было у Джессики?

    39.

    У Стэна было 14 пенни. У Дэйва было 9 пенни. У кого было меньше копеек?

    40. Бет было 8 лет. Ее сестре Лори было 10 лет. Который старше? Насколько старше?

    41. Палка была 10 дюймов в длину. Он был окрашен в красный и белый цвета. 5 дюймов палки были красными. Какая часть палочки была белой?

    42. Кошка Резвая весит 6 фунтов. Варежки кот весит 9 килограммов. Какая кошка весит больше? На сколько больше?

    43. 17 — счастливое число Чана. Число, стоящее перед числом Чана, — счастливое число Мин. Какое счастливое число Мин?

    44. Все автобусы выстроены по порядку номеров. Автобус №12 — это автобус Хуана. Какой номер автобуса идет сразу после автобуса Хуана?

    45. На прошлой неделе Стейси занималась футболом 6 часов. Эмили занималась футболом 4 часа на прошлой неделе. Кто дольше занимался футболом? Сколько еще?

    46. Класс миссис Тан получил 18 новых книг с книжной ярмарки.

    Класс мистера Смита получил 15 новых книг с книжной ярмарки. Какой класс получил больше новых книг с книжной ярмарки?

    47. За обеденным столом сидят 10 детей. 4 — мальчики. Остальные — девушки. Сколько детей за обеденным столом девочек?

    48. Двери в школе расположены по порядку номеров. Кэрол в комнате №11. Делия в комнате прямо перед Кэрол. В какой комнате Делия?

    49. Есть 3 ящика. В розовой коробке 4 мяча. В оранжевой коробке 2 мяча. В черном ящике на 2 шара больше, чем в розовом. В какой коробке больше всего мячей? Сколько их в том ящике?

    50. Джон любит рисовать. Он рисует по 1 рисунку каждый день после школы. Сколько рисунков он делает каждую неделю?

    Наслаждаетесь этими задачками по математике для детского сада? Посетите наш центр детских садов, чтобы найти еще больше ресурсов.

    Получите версию этих текстовых задач в формате PPT.

    20 каверзных, но забавных вопросов по математике для начальной школы

    Если вы не стали инженером, банкиром или бухгалтером, математика в начальной и средней школе была проклятием вашего существования. Вы бы неделями неустанно готовились к этим дурацким стандартизированным тестам — и все же, наступая в день экзамена, вы все равно не имели бы ни малейшего представления о том, для чего нужны какие-либо уравнения или сложные математические задачи. Поверьте нам, мы поняли.

    Хотя логика может привести вас к мысли, что ваши математические способности естественным образом улучшились с возрастом, к сожалению, реальность такова, что, если вы не решаете задачи по алгебре и геометрии ежедневно, скорее всего, произойдет обратное. случай.

    Не верите нам? Затем испытайте свою мудрость в вычислении чисел с помощью этих хитрых математических вопросов, взятых прямо из школьных тестов и домашних заданий, и убедитесь в этом сами.

    1. Вопрос: Какой номер парковочного места занимает автомобиль?

    Эта сложная математическая задачка стала вирусной несколько лет назад после того, как появилась на вступительном экзамене в Гонконге… для шестилетних детей. Предположительно, у студентов было всего 20 секунд, чтобы решить задачу!

    Ответ: 87.

    Хотите верьте, хотите нет, но этот «математический» вопрос вообще не требует математики. Если вы перевернете изображение вверх ногами, то увидите, что имеете дело с простой числовой последовательностью.

    2. Вопрос: Замените вопросительный знак в приведенной выше задаче соответствующим номером.

    Эта проблема не должна быть слишком сложной для решения, если вы много играете в судоку.

    Ответ: 6.

    Все числа в каждой строке и столбце в сумме дают 15! (Кроме того, 6 — единственное число, не представленное среди чисел от 1 до 9.)

    3. Вопрос: Найдите эквивалентное число.

    Эта задача взята прямо из стандартного теста, проведенного в Нью-Йорке в 2014 году.

    Ответ: 9.

    Shutterstock

    Простите, если вы не помните, как именно работают экспоненты. Чтобы решить эту задачу, вам просто нужно вычесть показатели степени (4-2) и решить для 3 2 , что расширяется до 3 x 3 и равняется 9.

    4.

    Вопрос: Сколько маленьких собак заявлено для участия в выставке? Изображение взято с Imgur/zakiamon

    Этот вопрос взят непосредственно из домашнего задания по математике второклассника. Угу.

    Ответ: 42,5 собаки.

    Чтобы вычислить, сколько маленьких собак участвует в соревнованиях, вы должны вычесть 36 из 49, а затем разделить результат, 13, на 2, чтобы получить 6,5 собак, или количество соревнующихся больших собак. Но вы еще не закончили! Затем вам нужно прибавить 6,5 к 36, чтобы получить количество соревнующихся маленьких собак, которое равно 42,5. Конечно, на самом деле половина собаки не может участвовать в выставках собак, но ради решения этой математической задачи давайте предположим, что это возможно.

    5. Вопрос: Найдите площадь красного треугольника.

    Изображение с YouTube

    Этот вопрос использовался в Китае для выявления одаренных пятиклассников. Предположительно, некоторые из сообразительных студентов смогли решить это менее чем за одну минуту.

    Ответ: 9.

    Чтобы решить эту задачу, вам нужно понять, как работает площадь параллелограмма. Если вы уже знаете, как связаны площадь параллелограмма и площадь треугольника, то сложите 79 и 10, а затем вычтите 72 и 8, чтобы получить 9.должно иметь смысл, но если вы все еще запутались, посмотрите это видео на YouTube для более подробного объяснения.

    6. Вопрос: Какой высоты стол?

    Изображение с YouTube

    YouTuber MindYourDecisions адаптировал этот ошеломляющий математический вопрос из похожего на домашнее задание ученика начальной школы в Китае.

    Ответ: 150 см.

    Изображение с YouTube

    Поскольку одно измерение включает рост кошки и вычитает рост черепахи, а другое делает противоположное, вы можете просто вести себя так, как будто двух животных здесь нет. Поэтому все, что вам нужно сделать, это сложить два измерения — 170 см и 130 см — вместе и разделить их на 2, чтобы получить высоту стола, 150 см.

    7. Вопрос: Если стоимость биты и бейсбольного мяча вместе взятых составляет 1,10 доллара, а бита стоит на 1 доллар больше, чем мяч, сколько стоит мяч?

    Shutterstock

    Эта задача с математической точки зрения очень похожа на одну из других в этом списке.

    Ответ: 0,05 доллара США.

    Вспомните ту задачу о собаках на выставке собак и используйте ту же логику для решения этой задачи. Все, что вам нужно сделать, это вычесть 1,00 доллара из 1,10 доллара, а затем разделить этот результат, 0,10 доллара, на 2, чтобы получить окончательный ответ, 0,05 доллара.

    8. Вопрос: Когда у Шерил день рождения?

    Изображение через Facebook/Kenneth Kong

    Если вам трудно это прочитать, см. здесь:

    «Альберт и Бернард только что подружились с Шерил, и они хотят знать, когда у нее день рождения. Шерил дает им список из 10 возможных даты. 14       15 августа       17 августа

    Затем Шерил сообщает Альберту и Бернарду по отдельности месяц и день своего дня рождения соответственно.0003

    Альберт: Я не знаю, когда день рождения Шерил, но я знаю, что Бернард тоже не знает.

    Бернард: Сначала я не знал, когда день рождения Шерил, но теперь я знаю.

    Альберт: Тогда я также знаю, когда у Шерил день рождения.

    Итак, когда у Шерил день рождения?»

    Непонятно, почему Шерил не могла просто сказать Альберту и Бернарду месяц и день своего рождения, но это не имеет отношения к решению этой проблемы.

    Ответ: 16 июля.

    Запутался о том, как можно найти какой-либо ответ на этот вопрос? Не волнуйтесь, так было в большинстве стран мира, когда несколько лет назад этот вопрос, взятый из математической олимпиады в Сингапуре и Азии, стал вирусным. New York Times  шаг за шагом объясняет, как добраться до 16 июля, и вы можете прочитать их подробный вывод здесь.

    9. Вопрос: Найдите пропущенную букву.

    Изображение из Facebook/The Holderness Family

    Это взято из домашнего задания первоклассника .

    Ответ: пропущена буква J.

    Если сложить вместе значения, указанные для S, B и G, сумма получится равной 40, а добавление пропущенной буквы J (значение которой равно 14) дает сумма других диагоналей такая же.

    10. Вопрос: Решите уравнение.

    Изображение с YouTube

    Эта задача может показаться простой, но удивительное количество взрослых не могут решить ее правильно.

    Ответ: 1.

    Начните с решения части уравнения, относящейся к делению. Для этого, если вы забыли, вам нужно перевернуть дробь и переключиться с деления на умножение, получив таким образом 3 x 3 = 9. Теперь у вас есть 9 – 9 + 1, и оттуда вы можете просто работать слева направо. вправо и получите окончательный ответ: 1.

    11. Вопрос: Где следует провести линию, чтобы приведенное ниже уравнение было точным?

    5 + 5 + 5 + 5 = 555.

    Ответ: Над знаком «+» следует провести черту.

    ae0fcc31ae342fd3a1346ebb1f342fcb

    Когда вы проводите наклонную линию в верхнем левом квадранте знака «+», она становится числом 4, и, таким образом, уравнение принимает вид 5 + 545 + 5 = 555.

    12. Вопрос: Решите незавершенное уравнение .

    Попробуйте выяснить, что общего у всех уравнений.

    Ответ: 4 = 256.

    В каждом уравнении используется следующая формула: 4 x = Y. Таким образом, 4 = 4, 4 = 16, 4 3 = 64 и 4 4  = 256.

    13. Вопрос: Сколько треугольников на изображении выше?

    Когда Best Life впервые написали об этом обманчивом вопросе, нам пришлось просить математика объяснить ответ!

    Ответ: 18.

    Некоторых людей смущают треугольники, спрятанные внутри треугольников, а другие забывают включить гигантский треугольник, в котором заключены все остальные. В любом случае, очень немногим людям — даже учителям математики — удалось найти правильный ответ на эту проблему. А чтобы получить дополнительные вопросы, которые проверят ваше предыдущее образование, ознакомьтесь с этими 30 вопросами, которые вам нужно будет ответить на высший балл, чтобы сдать экзамен по географии в 6-м классе.

    14. Вопрос: Добавьте 8,563 и 4,8292.

    Сложить два десятичных знака проще, чем кажется.

    Ответ: 13.3922.

    Пусть вас не смущает тот факт, что 8.563 имеет меньше номеров, чем 4.8292. Все, что вам нужно сделать, это добавить 0 в конце 8,563, а затем добавить, как обычно.

    15. Вопрос: На озере есть кувшинки. Каждый день патч удваивается в размере…

    Shutterstock

    … Если патчу требуется 48 дней, чтобы покрыть все озеро, сколько времени потребуется, чтобы патч покрыл половину озера?

    Ответ: 47 дней.

    Большинство людей автоматически предполагают, что половина озера будет покрыта за половину времени, но это предположение неверно. Поскольку участок подушек удваивается в размере каждый день, озеро будет покрыто наполовину всего за один день до того, как оно покроется полностью.

    16. Вопрос: Сколько футов в миле?

    Эта задачка для начальной школы требует не столько решения проблем, сколько запоминания.

    Ответ: 5280.

    Это был один из вопросов в популярном шоу  Вы умнее пятиклассника?

    17.

    Вопрос: При каком значении «x» приведенное ниже уравнение верно? Shutterstock

    -15 + (-5x) = 0

    Ответ: -3.

    Вам простительно думать, что ответ равен 3. Однако, поскольку число рядом с x отрицательное, нам нужно, чтобы x также было отрицательным, чтобы получить 0. Следовательно, x должно быть -3.

    18. Вопрос: Сколько 1,92 разделить на 3?

    Возможно, вам придется попросить помощи у детей.

    Ответ: 0,64.

    Чтобы решить эту, казалось бы, простую задачу, нужно убрать десятичную дробь из числа 1,92 и вести себя так, будто ее нет. После того, как вы разделили 192 на 3, чтобы получить 64, вы можете вернуть десятичную запятую на место и получить окончательный ответ 0,64.

    19. Вопрос: Решите приведенное выше математическое уравнение.

    Изображение с YouTube

    Не забывайте о PEMDAS!

    Ответ: 9.

    Используя PEMDAS (аббревиатура, указывающая порядок, в котором вы решаете: «круглые скобки, показатели степени, умножение, деление, сложение, вычитание»), вы должны сначала решить сложение внутри скобок (1 + 2 = 3), и оттуда закончите уравнение, как оно написано слева направо.